Conflicts Cases Complete

April 13, 2018 | Author: inayauan | Category: Will And Testament, Probate, Evidence (Law), Citizenship, Justice
Share Embed Donate


Short Description

Download Conflicts Cases Complete...

Description

Aznar vs. Garcia, G.R. No. L-16749, Jan. 3, 1963 FACTS: Edward Christensen, who at his death was a US citizen but domiciled in the Philippines, left a will, devising unto Maria Helen a certain amount of money and giving the rest of his estate to Maria Lucy. Helen opposed the partition on the ground that she is deprived of her legitime. Her contention is that the law of California directs that the law of the domicile (Philippines) should govern the will. ISSUE: Whether or not the national law or the domiciliary law should apply HELD: The intrinsic validity of wills is governed by the national law of the decedent. In the present case, the national law of Edward is the laws of California. However, there were two conflicting California laws regarding succession. One is enunciated in In Re Kaufman (which does not provide for legitimes) and another is Art. 946 of the California Civil Code (which provides that the law of the domicile applies). SC held that the national law is Art. 946, which is the conflict of laws rule of California. The

Renvoi doctrine is not applicable because there is no conflict as to the nationality and domicile of Bellis. He is both a citizen and a resident of Texas. So even if assuming the law of Texas applies the domiciliary rule, it is still Texas law that governs because his domicile is Texas.

Gibbs v. Government of Philippine Islands Facts: Gibbs and his wife were American nationals, domiciled in California. They acquired lands in the Philippines. The wife died in California. Gibbs was appointed administrator of the intestate proceedings instituted in Manila. Gibbs asked the cour t to adjudicate to him lands acquired in the Philippines not under our laws on succession but because in accordance with the law of California, the community property of spouses who are citizens of California, upon the death of the wife previous to that of the husband, belongs absolutely to the surviving husband without administration. CFI granted such to Gibbs upon proof of California law. The register of deeds refused to transfer such properties on the ground of nonpayment of inheritance tax. Gibbs argued that the conjugal right of a California wife in a community property is a personal right and

reason is that In Re Kaufman applies only to residents while Art. 946 is specific to nonresidents. Thus, since Art. 946 contains a referback to Philippine laws (the law of the domicile), then Maria Helen is entitled to her legitime.

even if this was a case of succession, California law would still apply.

Bellis vs. Bellis, G.R. No. L-23678, June 6, 1967

The appellee contends that the law of California should determine the nature and extent of the title, if any, that was vested in Eva Johnson Gibbs citing article 9 of the Civil Code. But that, even if the nature and extent of her title under said certificates be governed by the law of the Philippine Islands, the laws of California govern the succession to such title, citing the second paragraph of article 10 of the Civil Code. It is argued that the conjugal right of the California wife in community real estate

FACTS: Amos Bellis, a US citizen, died a resident of Texas. He left two wills -- one devising a certain amount of money to his first wife and three illegitimate children and another, leaving the rest of his estate to his seven legitimate children. Before partition, the illegitimate children who are Filipinos opposed on the ground that they are deprived of their legitimes. ISSUE: Whether the applicable law is Texas law or Philippine laws HELD: Applying the nationality rule, the law of Texas should govern the intrinsic validity of the will and therefore answer the question on entitlement to legitimes. But since the law of Texas was never proven, the doctrine of processual presumption was applied. Hence, SC assumed that Texas law is the same as Philippine laws, which upholds the nationality rule.

Private International Law

Issue: is Gibbs exempt from inheritance tax? Held:

in the Philippine Islands is a personal right and must, therefore, be settled by the law governing her personal status, that is, the law of California. But our attention has not been called to any law of California that incapacitates a married woman from acquiring or holding land in a foreign jurisdiction in accordance with the lex rei sitae. The trial court found that under the law of California, upon the death of the wife, the entire community property without administration belongs to the surviving husband; that he is the absolute owner of all the community property from the moment of the death of his wife, not by

Page 1

virtue of succession or by virtue of her death, but by virtue of the fact that when the death of the wife precedes that of the husband he acquires the community property, not as an heir or as the beneficiary of his deceased wife, but because she never had more than an inchoate interest or expentancy which is extinguished upon her death. However, the argument of the appellee apparently leads to this dilemma: If he takes nothing by succession from his deceased wife, how can the second

paragraph of article 10 be invoked? Can the appellee be heard to say that there is a legal succession under the law of the Philippine Islands and no legal succession under the law of California? It seems clear that the second paragraph of article 10 applies only when a legal or testamentary succession has taken place in the Philippines and in accordance with the law of the Philippine Islands; and the foreign law is consulted only in regard to the order of succession or the extent of the successional rights; in other words, the second paragraph of article 10 can be invoked only when the dece ased was vested with a descendible interest in property within the jurisdiction of the Philippine Islands. However, the court held that it is principle firmly established that to the law of the state in which the land is situated we must look for the rules which govern its descent, alienation, and transfer, and fo r the effect and construction of wills and other conveyances. This fundamental principle is stated in the first paragraph of article 10 of our Civil Code as follows: "Personal property is subject to the laws of the nation of the owner thereof; real property to the laws of the country in which it is situated. Under the provisions of the Civil Code and the jurisprudence prevailing here, the wife, upon the acquisition of any conjugal property, becomes immediately vested with an interest and title therein equal to that of her husband, subject to the power of management and disposition which the law vests in the husband. It results that the wife of the appellee was, by the law of the Philippine Islands, vested of a descendible interest, equal to that of her husband and the descendible interest of Eva Johnson Gibbs in the lands aforesaid was transmitted to her heirs by virtue of inheritance and this transmission plainly falls within the language of section 1536 of Article XI of Chapter 40 of the Administrative Code which levies a tax on inheritances. Fleumer vs. Hix 54 Phil 610

Facts:

Private International Law

The petitioner is a special administrator of the estate of Edward Hix. He alleged that the latter’s will was execute d in Elk in s, West Virginia on Novembe r 3, 1925 by Hix who had his residence in that jurisdiction, and that the laws of that state govern. To thi s en d, th e petit ioner su bmitt ed a cop y of Sect ion 38 68 of Act s 1882 , c.84 as found in West Virginia Code, annotated by Hogg, Charles E., vol.2 1914, p. 1690 an d as cert ified to by th e Director of National Library. The Judge of the First Insta nce however denied the probate of the will on the grounds that the will did not show the following: acknowledgment by Hix in the presence of 2 competent witnesses Witnesses subscribed to will in presence of the testator, and of each other •



Hence, this appeal. Issue: Is it necessary to prove in this jurisdiction the existence of such law in West Virginia as a prere quisi te to the allowance and recording of said will? Held: Yes. The laws of the foreign jurisdiction do not prove themsel ves in our court s. The courts of th e Philippine Islands are not authorized to take judicial notice of the laws of the various states of the Am er ican Union. Suc h law s mus t be prov ed as facts . Here the requirements of the law were not met. •







Ther e was no sho win g that the boo k from which an extract was taken was printed or published under the authority of the state of West Virginia, as provided in Sec 30 of the Code of Civil Procedure. Nor was the extract from the law attested by the certificate of the officer having of the theas seal of charge th e State of srcinal, Wes t under Virginia provided in Sec 301. No evi den ce was intr odu ced sho win g that the extract from the laws of West Virginia was in force at the time the alleged will was executed. Due execution of the will was not established: only showed testimony of the petitioner

The court therefore did not err in denying the probate of the will. The existence of such law in West Virginia must be proved.

Miciano v. Brimo

Page 2

Decedent is a Turkish citizen. He left a will stating that he wishes to dispose of his properties in accordance with Philippine Laws. His brother, Andre Brimo opposed the partition. The appellant's opposition is based on the fact that the partition in question puts into effect the provisions of Joseph G. Brimo's will which are not in accordance with the laws of his Turkish nationality, for which reason they are void as being in violation or article 10 of the Civil Code which states that testamentary successions shall be governed by the national law of the deceased. He was therefore excluded from the inheritance pursuant to a clause in the decedent’s will that anyone who would oppose the decedent’s wish to dispose his estate under Philippine law shall have his share annul or cancelled.

Impossible conditions and those contrary to law or good morals shall be considered as not imposed and shall not prejudice the heir or legatee in any manner whatsoever, even should the testator otherwise provide.

And said condition is contrary to law because it expressly ignores the testator's national law when, according to article 10 of the civil Code above quoted, such national law of the test ator is the o ne to govern dispositions.

his

testamentary

PCIB VS. ESCOLIN 56 SCRA 266

FACTS:

Issue: Whether or not the declaration that Turkish laws are impertinent to this case; Whether or not the appellant’s exclusion from the will is valid? Held: 1st issue The oppositor dispositions did not prove testamentary are that not said in accordance with the Turkish laws, inasmuch as he did not present any evidence showing what the Turkish laws are on the matter , and in the absence of evidence on such laws, they are presumed to be the same as those of the Philippines. The refusal to give the oppositor another opportunity to prove such laws does not constitute an error. It is discretionary with the trial court, and, taking into consideration that the oppositor was granted ample opportunity to introduce competent evidence, the Court finds no abuse of discretion on the part of the lower court in this particular. There is, therefore, no evidence in the record that the national law of the testator Joseph G. Brimo was violated in the testamenta ry dispositions in question which, not being contrary to our laws in force, must be complied with and executed.

2nd issue The institution of legatees in this will is conditional, and the condition is that the instituted legatees must respect the testator's will to distribute his property, not in accordance with the laws of his nationality, but in accordance with the laws of the Philippines. The fact is, however, that the said condition is void, being contrary to law, for article 792 of the civil Code provides the following:

Private International Law

Linnie Jane Hodges died giving her testamentary provisions to her husband. At the time of her death, she was citizen of Texas but, was, however domiciled in the Philippines. To see whether the testamentary provisions are valid, it is apparent and necessary to know what law should be applied. ISSUE: Whether or not laws of Texas is applicable. RULING: Prior evidence already presented to prove the existence of Texas Law. It is necessary that the Texas law be ascertained. Here it must be proven whether a renvoi will happen or whether Texas law makes the testamentary provisions valid. In line with Texas law, that which should be proven is the law enforced during the death of Hodges and not in any other time. The Supreme Court held that for what the Texas law is on the matter, is a question of fact to be resolved by the evidence that would be presented in the probate court. Texas law at the time of her death (and not said law at any other time). Article 16 of the Civil Code provides that “the national law of the perso n whose succession is under consideration, whatever may be the nature of the property and regardless of the country wherein said property may be found”,

Page 3

shall prevail. However, the Conflict of Law of Texas, which is the “national law” of the testatrix, Linnie Jane Hodges, provide that the domiciliary law (Philippine law — see paragraph 2, supra) should govern the testamentary dispositions and successional rights over movables (personal properties), and the law of the situs of the property (also Philippine law as to properties located in the Philippines) with regards immovable (real properties). Thus applying the “Renvoi Doctrine”, as approved and applied by our Supreme Court in the case of “In The Matter Of The Testate Estate of Eduard E. Christensen”, G.R. No. L-16749, promulgated January 31, 1963, Philippine law should apply to the Will of Linnie Jane Hodges and to the successional rights to her estate insofar as her movable and immovable assets in the Philippines are concerned. We shall not, at this stage, discuss what law should govern the assets of Linnie Jane Hodges located in Oklahoma and Texas, because the only assets in issue in this motion are those within the jurisdiction of this motion Court in the two above-captioned Special Proceedings. Nota bene: When can foreign law be given judicial notice

1. 2.

If the court of the forum is familiar of the law Or it is within the actual knowledge of the court

Suntay v. Suntay

Jose B. Suntay died intestate leaving properties in the Philippines and a house in China. He is survived by children from the 1 st marriage and a child and his widow from the 2 nd. Intestate proceedings were instituted. Thereafter the widow filed a petition for a probate of a will but was later denied when the will was lost after the filing of said petition. On appeal, the petition was granted since there was sufficiency to prove the loss of the will. In spite of the fact that a commission from the probate court was issued on 24 April 1937 for the taking of the deposition of Go Toh, an attesting witness to the will , on 7 February 1938 the probate court denied a motion for continuance of the hearing sent by cablegram from China by the surviving widow and dismissed the petition. In the meantime the Pacific War supervened. After liberation, Silvino claimed to have found a will by his father which was filed, recorded and probated in the Amoy district court, Province of Fookien, China and thus filed a petition in the intestate proceedings praying for the probate of the will. Issue: May a will filed, recorded, and probated in China be reprobated in the Philippines? Held: As to the will claimed to have been executed on 4 January 1931 in Amoy, China, the law on the point in Rule 78.

Private International Law

Section 1 of the rule provides: Wills proved and allowed in a foreign country, according to the laws of such country, may be allowed, filed, and recorded by the proper Court of First Instance in the Philippines.

Section 2 provides: When a copy of such will and the allowance thereof, duly authenticated, is filed with a petition for allowance in the Philippines, by the executor or other person interested, in the court having jurisdiction, such court shall fix a time and place for the hearing, and cause notice thereof to be given as in case of an original will presented for allowance.

Section 3 provides: If it appe ars at the hear ing that the will should be allowed in the Philippines, the court shall so allow it, and a certificate of its allowance, signed by the Judge, and attested by the seal of the courts, to which shall be attache d a copy of the will, shall be filed and recorded by the clerk, and the will shall

have effect as ifcourt. originally provedthe andsame allowed in such The fact that the municipal district court of Amoy, China, is a probate court must be proved. The law of China on procedure in the probate or allowance of wills must also be proved. The legal requirements for the execution of a valid will in China in 1931 should also be established by competent evidence. There is no proof on these points. The unverified answers to the questions propounded by counsel for the appellant to the Consul General of the Republic of China objected to by counsel for the appellee, are inadmissible, because apart from the fact that the office of Consul General does not quali fy and make the person who holds it an expert on the Chinese law on procedure in probate matters, if the same be admitted, the adverse party would be deprived of his right to confront and cross-examine the witness. Consuls are appointed to attend to trade matters. The order of the municipal district court of Amoy, China, does not purport to probate or allow the will which was the subject of the proceedings. In view thereof, the will and the alleged probate thereof cannot be said to have been done in accordance with the accepted basic and fundamental concepts and principles followed in the probate and allowance of wills. Consequently, the authenticated transcript of proceedings held in the municipal district court of Amoy, China, cannot be deemed and accepted as

Page 4

proceedings leading to the probate or allowance of a will and, therefore, the will referred to therein cannot be allowed, filed and recorded by a competent court of this country.

State of Illinois and the will was not in conformity with the laws of that State.

Likewise, the proceedings had in the municipal district court of Amoy were for the purpose of taking the testimony of two attes ting witnesses to the will and that the order of the municipal district court of Amoy does not purport to probate the will. In the absence of proof that the municipal district court of Amoy is a

Emil Johnson was a citizen of the State of Illinois. In the absence of clear proof to the contrary it should be presumed that a person naturalized in a court of a certain State thereby becomes a citizen of that State as well as of the United States.

probate and on matters, the Chinese law be of procedurecourt in probate it may presumed that the proceedings in the matter of probating or allowing a will in the Chinese courts are the a deposition or to a perpetuation of testimony, and even if it were so it does not measure same as those prov ided for in our laws on the subject. It is a proceedings in rem and for the validity of such proceedings personal notice or by publication or both to all interested parties must be made. The interested parties in the case were known to reside in the Philippines. The evidence shows that no such notice was received by the interested parties residing in the Philippines. The proceedings had in the municipal district court of Amoy, China, may be likened to or come up to the standard of such proceedings in the Philippines for lack of notice to all interested parties and the proceedings were held at the back of such interested parties.

In Section 625 of the Code of Civil Procedure it is declared that "the allowance by the court of a will of real or personal property shall be conclusive as to its due execution."

The decree appealed from is affirmed, without pronouncement as to costs. In re: Johnson

Emil H. Johnson, a native of Sweden and a naturalized citizen of the United States, died in the city of Manila, leaving a holographic will and is signed by himself and two witnesses only, instead of three witnesses required by section 618 of the Code of Civil Procedure. However, a petition was presented in the Court of First Instance of the city of Manila for the probate of this will, on the ground that Johnson was at the time of his death a citizen of the State of Illinois, United States of America; that the will was duly executed in accordance with the laws of that State; and hence could properly be probated here. Petitioner contends that the decedent is not a citizen of Illinois and prays to annul the decree of probate and put the estate into intestate administration, thus preparing the way for the establishment of the claim of the petitioner as the sole legitimate heir of her father. Issue: Whether or not judgment from which the petitioner seeks relief should be set aside because the testator was not a resident of the

Private International Law

Held:

The due execution of a will involves conditions relating to a number of matters, such as the age and mental capacity of the testator, the signing of the document by the testator, or by someone in his behalf, and the acknowledgment of the instrument by him in the presence of the required number of witnesses who affix their signatures to the will to attest the act. The proof of all these requisites is involved in the probate; and as to each and all of them the probate is conclusive. Our reported cases do not contain the slightest intimation that a will which has been probated according to law, and without fraud, can be annulled, in any other proceeding whatever, on account of any supposed irregularity or defect in the execution of the will or on account of any error in the action of the court upon the proof adduced before it. We are not unmindful of the fact that when a citizen of one State leaves it and takes up his abode in another State with no intention of returning, he immediately acquires citizenship in the State of his new domicile. This is in accordance with that provision of the Fourteenth Amendment to the Constitution of the United States which says that every citizen of the United States is a citizen of the State where in he resides. The effect of this provision necessarily is that a person transferring his domicile from one State to another loses his citizenship in the State of his srcinal above upon acquiring citizenship in the State of his new abode. The acquisition of the new State citizenship extinguishes the old. That situation, in our opinion, has no analogy to that which arises when a citizen of an American State comes to reside in the Phili ppine Islands. Here he cannot acquir e a new citizenship; nor by the mere change of domicile does he lose that which he brought with him. Proper rule in taking judicial notice: The proper rule is to require proof of the statutes of the States of the American Union whenever their provisions are determinative of the issues in any action litigated in the Philippine courts. Collector vs. Fisher

Page 5

Facts: This case relates to the determination and settlement of the hereditary estate left by the deceased Walter G. Stevenson, and the laws applicable thereto. Walter G. Stevenson (born in the Philippines on August 9, 1874 of British parents and married in the City of Manila on anuary J 23, 1909 to Beatrice Mauricia Stevenson another British subject) died on San Francisco, February 22, 1951 in California, U.S.A. whereto he and his wife moved and established their permanent residence since May 10, 1945. In his will executed in San Francisco on May 22, 1947, and which was duly probated in the Superior Court of California on April 11, 1951, Stevenson instituted his wife Beatrice as his sole heiress to the following real and personal properties acquired by the spouses while residing in the Philippines. Ancillary administration proceedings were instituted in the Court of First Instance of Manila for the settlement of the estate in the Philippines. In due time Stevenson's will was duly admitted to probate by our court and Ian Murray Statt was appointed ancillary administrator of the estate, filed a preliminary estate and inheritance tax return with the reservation of having the properties declared therein finally appraised at their values six months after the death of Stevenson. Preliminary return was made by the ancillary administrator in order to secure the waiver of the Collector of Internal Revenue on the inheritance tax due on the 210,000 shares of stock in the Mindanao Mother Lode Mines Inc. which the estate then desired to dispose in the United States. Acting upon said return, the Collector of Internal Revenue accepted the valuation of the personal properties declared therein, but increased the appraisal of the two parcels of land located in Baguio City by fixing their fair market value. After allowing the deductions claimed by the ancillary administrator for funeral expenses in the amount of P2,000.00 and for judicial and administration expenses in the sum of P5,500.00, the Collector assessed the state the amount of P5,147.98 for estate tax and P10,875,26 or inheritance tax, or a total of P16,023.23. Both of these assessments were paid by the estate. The ancillary administrator filed in amended estate and inheritance tax return in pursuance of his reservation made at the time of filing of the preliminary return and for the purpose of availing of the right granted by section 91 of the National Internal Revenue Code. Beatrice Mauricia Stevenson assigned all her rights and interests in the estate to the spouses, Douglas and Bettina Fisher, respondents herein. The ancillary administrator filed a second amended estate and inheritance tax return. This return declared the same assets of the estate stated in the amended return of September 22, 1952, except that it contained new claims for additional exemption and deduction to wit: (1) deduction in the amount of P4,000.00 from the gross estate of the decedent as provided for in Section 861 (4) of the U.S. Federal

Private International Law

Internal Revenue Code which the ancillary administrator averred was allowable by way of the reciprocity granted by Section 122 of the National Internal Revenue Code, as then held by the Board of Tax Appeals in case No. 71 entitled "Housman vs. Collector," August 14, 1952; and (2) exemption from the imposition of estate and inheritance taxes on the 210,000 shares of stock in the Mindanao Mother Lode Mines, Inc. also pursuant to the reciprocity proviso of Section 122 of the National Internal Revenue Code. In this last return, the estate claimed that it was liable only for the amount of P525.34 for estate tax and P238.06 for inheritance tax and that, as a consequence, it had overpaid the government. The refund of the amount of P15,259.83, allegedly overpaid, was accordingly requested by the estate. The Collector denied the claim. For this reason, action was comme nced in the Court of First Instance of Manila by respondents, as assignees of Beatrice Mauricia Stevenson, for the recovery of said amount. Pursuant to Republic Act No. 1125, the case was forwarded to the Court of Tax Appeals which court, after hearing, rendered decision : that: (a) the one-half (½) share of the surviving spouse in the conjugal partnership property as diminished by the obligations properly chargeable to such property should be deducted from the net estate of the deceased Walter G. Stevenson, pursuant to Section 89-C of the National Internal Revenue Code; (b) the intangible personal property belonging to the estate of said Stevenson is exempt from inheritance tax, pursuant to the provision of section 122 of the National Internal Revenue Code in relation to the California Inheritance Tax Law but decedent's estate is not entitled to an exemption of P4,000.00 in the computation of the estate tax; (c) for purposes of estate and inheritance taxation the Baguio real estate of the spouses should be valued at P52,200.00, and 210,000 shares of stock in the Mindanao Mother Lode Mines, Inc. should be appraised at P0.38 per share; and (d) the estate shall be entitled to a deduction of P2,000.00 for funeral expenses and judicial expenses of P8,604.39.

Issue: whether or not foreign law needs to be proved in our jurisdiction? Ruling: It is well-settled that foreign laws do not prove themselves in our jurisdiction and our courts are not authorized to take judicial notice of them. Like any other fact, they must be alleged and proved. Section 41, Rule 123 of our Rules of Court prescribes the manner of proving foreign laws before our tribunals. However, although we believe it desirable that these laws be proved in accordance with said rule, we held in the case of Willamette Iron and Steel Works v. Muzzal, 61 Phil. 471, that "a reading of sections 300 and 301 of our Code of Civil Procedure (now section 41, Rule 123) will convince one that these sections do not exclude the presentation of other competent evidence to prove the existence of a foreign law." In that

Page 6

case, we considered the testimony of an attorney-at-law of San Francisco, California who quoted verbatim a section of California Civil Code and who stated that the same was in force at the time the obligations were contracted, as sufficient evidence to establish the existence of said law. In line with this view, we find no error, therefore, on the part of the Tax Court in considering the pertinent California law as proved by respondents' witness.

the death of the testator. Since no right to share in the inheritance in favor of a divorced wife exists in the State of Nevada and since the court below had already found that there was no conjugal property between the testator and Magdalena C. Bohanan, the latter can now have no longer claim to pay portion of the estate left by the testator.

Phil Trust Company vs. Bohanan

whether the estementary dispositions, especially those for the children which are short of the legitime given them by the Civil Code of the Philippines, are valid?

Facts: Appeal against an order of the Court of First Instance of Manila, Hon. Ramon San Jose, presiding, dismissing the objections filed by Magdalena C. Bohanan, Mary Bohanan and Edward Bohanan to the project of partition submitted by the executor and approving the said project. The Court of First Instance of Manila, Hon. Rafael Amparo, presiding, admitted to probate a last will and test ament of C. O. Bohanan, executed by him in Manila. In the said order, the court made the following findings: According to the evidence of the opponents the testator was born in Nebraska and therefore a citizen of that state, or at least a citizen of California where some of his properties are located. This contention is untenable . Notwithstanding the long residence of the decedent in the Philippines, his stay here was merely temporary, and he continued and remained to be a citizen of the United States and of the state of his pertinent residence to spend the rest of his days in that state. His permanent residence or domicile in the United States depended upon his personal intent or desire, and he selected Nevada as his homicide and therefore at the time of his death, he was a citizen of that state. Nobody can choose his domicile or permanent residence for him. That is his exclus ive personal right. Wherefore, the court finds that the testator C. O. Bohanan was at the time of his death a citizen of the United States and of the State of Nevada and declares that his will and testament, is fully in accordance with the laws of the state of Nevada and admits the same to probate. Accordingly, the Philippine Trust Company, named as the executor of the will, is hereby appointed to such executor and upon the filing of a bond in the sum of P10,000.00. The executor filed a project of partition dated January 24, 1956, making adjudications, in accordance with the provisions of the will. The wife Magadalen a C. Bohanan and her two children question the validity of the testamentary provisions disposing of the estate in the manner above indicated, claiming that they have been deprived of the legitime that the laws of the forum concede to them. Moreover, the court below had found that the testator and Magdalena C. Bohanan were married on January 30, 1909, and that divorce was granted to him on May 20, 1922; that sometime in 1925, Magdalena C. Bohanan married Carl Aaron and this marriage was subsisting at the time of

Private International Law

Issue:

Ruling: The old Civil Code, which is applicable to this case because the testator died in 1944 , expressly provides that successional rights to personal property are to be earned by the national law of the person whose succession is in question. Says the law on this point: Nevertheless, legal and testamentary successions, in respect to the order of succession as well as to the extent of the successional rights and the intrinsic validity of their provisions, shall be regulated by the national law of the person whose succession is in question, whatever may be the nature of the property and the country in which it is found. (par. 2, Art. 10, old Civil Code, which is the same as par. 2 Art. 16, new Civil Code.) In the proceedings for the probate of the will, it was foun d out and it was decided that the testator was a citizen of the State of Nevada because he had selected this as his domicile and his permanent residence. It is not disputed that the laws of Nevada allo w a testator to dispose of all his properties by will. It does not appear that at time of the hearing of the project of partition, the above-quoted provision was introduced in evidence, as it was the executor's duly to do. The law of Nevada, being a foreign law can only be proved in our courts in the form and manner provided for by our Rules, which are as follows: SEC. 41. Proof of public or official record . — An official record or an entry therein, when admissible for any purpose, may be evidenced by an official publication thereof or by a copy attested by the officer having the legal custody of the record, or by his deputy, and accompanied, if the record is not kept in the Philippines, with a certificate that such officer has the custody. . . . (Rule 123).

We have, however, consulted the records of the case in the court below and we have found that during the hearing on October 4, 1954 of the motion of Magdalena C. Bohanan for withdrawal of P20,000 as her share, the foreign law, especially Section 9905, Compiled Nevada Laws was introduced in evidence by appellant's counsel. Again said laws presented by the counsel for the executor and admitted by the Court during the hearing of the case on before Judge Rafael Amparo.

Page 7

In addition, the other appellants, children of the testator, do not dispute the above-quoted provision of the laws of the State of Nevada. Under all the above circumstances, we are constrained to hold that the pertinent law of Nevada, especially Section 9905 of the Compiled Nevada Laws of 1925, can be taken judicial notice of by us, without proof of such law having been offered at the hearing of the project of partition. As in accordance with Article 10 of the old Civil Code, the validity of testamentary dispositions are to be governed by the national law of the testator, and as it has been decided and it is not disputed that the national law of the testator is that of the State of Nevada, already indicated above, which allows a testator to dispose of all his property according to his will, as in the case at bar, the order of the court approving the project of partition made in accordance with the testamentary provisions, must be, as it is hereby affirmed, with costs against appellants. Zalamea vs. CA

Facts: Petitioners-spouses Cesar C. Zalamea and Suthira Zalamea, and their daughter, Liana Zalamea, purchased three (3) airline tickets from the Manila agent of respondent TransWorld Airlines, Inc. for a flight to New York to Los Angeles. The tickets of petitioners-spouses were purchased at a discount of 75% while that of their daughter was a full fare ticket. All three tickets represented confirmed reservations. Petitioners received notice of the reconfirmation of their reservations for said flight. On the appointed date, however, petitioners checked in at 10:00 a.m., an hour earlier than the scheduled flight at 11:00 a.m. but were placed on the wait-list because the number of passengers who had checked in before them had already taken all the seats available on the flight. Liana Zalamea appeared as the No. 13 on the wait-list while the two other Zalameas were listed as "No. 34, showing a party of two." Out of the 42 names on the wait list, the first 22 names were eventually allowed to board the flight to Los Angeles, including petitioner Cesar Zalamea. The two others, on the other hand, at No. 34, being ranked lower than 22, were not able to fly. As it were, those holding full-fare tickets were given first priority among the wait-listed passengers. Mr. Zalamea, who was holding the full-fare ticket of his daughter, was allowed to board the plane; while his wife and daughter, who presented the discounted tickets were denied boarding. Even in the next TWA flight to Los Angeles Mrs. Zalamea and her daughter, could not be accommodated because it was also fully booked. Thus, they were constrained to book in another flight and purchased two tickets from American Airlines at a cost of Nine Hundred Eighteen ($918.00) Dollars. Upon their arrival in the Philippines, petitioners filed an action for damages based on breach of contract of air carriage before the Regional Trial Court of

Private International Law

Makati. As aforesaid, the lower court ruled in favor of petitioners. On appeal, the respondent Court of Appeals held that moral damages are recoverable in a damage suit predicated upon a breach of contract of carriage only where there is fraud or bad faith. Since it is a matter of record that overbooking of flights is a common and accepted practice of airlines in the United States and is specifically allowed under the Code of Federal Regulations by the Civil Aeronautics Board, no fraud nor bad faith could be imputed on respondent TransWorld Airlines.

Ruling: That there was fraud or bad faith on the part of respondent airline when it did not allow petitioners to board their flight for Los Angeles in spite of confirmed tickets cannot be disputed. The U.S. law or regulation allegedly authorizing overbooking has never been proved. Foreign laws do not prove themselves nor can the courts take judicial notice of them. Like any other fact, they must be alleged and proved. Written law may be evidenced by an official publication thereof or by a copy attested by the officer having the legal custody of the record, or by his deputy, and accompanied with a certificate that such officer has custody. The certificate may be made by a secretary of an embassy or legation, consul general, consul, vice-consul, or consular agent or by any officer in the foreign service of the Philippines stationed in the foreign country in which the record is kept, and authenticated by the seal of his office. 7 Respondent TWA relied solely on the statement of Ms. Gwendolyn Lather, its customer service agent, in her deposition dated January 27, 1986 that the Code of Federal Regulations of the Civil Aeronautics Board allows overbooking. Aside from said statement, no official publication of said code was presented as evidence. Thus, respondent court's finding that overbooking is specifically allowed by the US Code of Federal Regulations has no basis in fact. Even if the claimed U.S. Code of Federal Regulations does exist, the same is not applicable to the case at bar in accordance with the principle of lex loci contractus which require that the law of the place where the airline ticket was issued should be applied by the court where the passengers are residents and nationals of the forum and the ticket is issued in such State by the defendant airline. 8 Since the tickets were sold and issued in the Philippines, the applicable law in this case would be Philippine law. Existing jurisprudence explicitly states that overbooking amounts to bad faith, entitling the passengers concerned to an award of moral damages. In Alitalia Airways v. Court of Appeals, 9 where passengers with confirmed bookings were refused carriage on the last minute, this Court held that when an airline issues a ticket to a passenger confirmed on a particular flight, on a certain date, a contract of carriage arises, and the passenger has every

Page 8

right to expect that he would fly on that flight and on that date . If he doe s not, then the carrier opens itself to a suit for breach of contract of carriage. Where an airline had deliberately overbooked, it took the risk of having to deprive some passengers of their seats in case all of them would show up for the check in. For the indignity and inconvenience of being refused a confirmed seat on the last minute, said passenger is entitled to an award of moral damages. Wild Valley Shipping Co. Vs. CA

Facts: The Philippine Roxas, a vessel owned by Philippine President Lines, Inc., private respondent herein, arrived in Puerto Ordaz, Venezuela, to load iron ore. Upon the completion of the loading and when the vessel was ready to leave port, an official pilot of Venezuela, was designated by the harbour authorities in Puerto Ordaz to navigate the Philippine Roxas through the Orinoco River. The Philippine Roxas experienced some vibrations when it entered the San Roque Channel. The vessel proceeded on its way, with the pilot assuring the watch officer that the vibration was a result of the shallowness of the channel. The master (captain) checked the position of the vessel and verified that it was in the centre of the channel. The Philippine Roxas ran around in the Orinoco River, thus obstructing the ingress and egress of vessels. As a result of the blockage, the Malandrinon, a vessel owned by herein petitioner Wildvalley Shipping Company, Ltd., was unable to sail out of Puerto Ordaz on that day. Subsequently, Wildvalley Shipping Company, Ltd. filed a suit with the Regional Trial Court of Manila, Branch III against Philippine President Lines, Inc. and Pioneer Insurance Company (the underwriter/insurer of Philippine Roxas) for damages in the form of unearned profits, and interest thereon amounting to US $400,000.00 plus attorney's fees, costs, and expenses of litigation. Issue: whether or not Venezuelan law is applicable to the case at bar? Ruling: It is well-settled that foreign laws do not prove themselves in our jurisdiction and our courts are not authorized to take judicial notice of them. Like any other fact, they must be alleged and proved. For a copy of a foreign public document to be admissible, the following requisites are mandatory: (1) It must be attested by the officer having legal custody of the records or by his deputy; and (2) It must be accompanied by a certificate by a secretary of the embassy or legation, consul general, consul, vice consular or consular agent or foreign service officer, and with the seal of his office. The latter requirement is not a mere technicality but is intended to justify the giving of full faith and

credit to the genuineness of a document in a foreign country. With respect to proof of written laws, parol proof is objectionable, for the written law itself is the best evidence. According to the weight of authority, when a foreign statute is involved, the best evidence rule requires that it be proved by a duly authenticated copy of the statute. At this juncture, we have to point out that the Venezuelan law was not pleaded before the lower court. A foreign law is considered to be pleaded if there is an allegation in the pleading about the existence of the foreign law, its import and legal consequence on the event or transaction in issue. A review of the Complaint revealed that it was never alleged or invoked despite the fact that the grounding of the M/V Philippine Roxas occurred within the territorial jurisdiction of Venezuela. We reiterate that under the rules of private international law, a foreign law must be properly pleaded and proved as a fact. In the absence of pleading and proof , the laws of a foreign country, or state, will be presumed to be the same as our own local or domestic law and this is known as processual presumption. Board of Commissioners v. Dela Rosa

Facts: On July 12, 1960, Santiago Gatchalian, grandfather of William Gatchalian, was recognized by the Bureau of Immigration as a native born Filipino citizen following the citizenship of natural mother Mariana Gatchalian. On June 27, 1961, Willian, then twelve years old, arrives in Manila from Hongkong together with a daughter and a son of Santiago. They had with them certificate of registration and identity issued by the Philippine consulate in Hongkong based on a cablegram bearing the signature of the secretary of foreign affairs, Felixberto Serrano, and sought admission as Filipino citizens. On July 6, 1961, the board of special inquiry admitted the Gatchalians as Filipino citizens and issued an identification certificate to William. The board of commissioners was directed by the Secretary of Justice to Review all cases where entry was granted on the ground that the entrant was a Filipino citizen such included the case of William. As a result of the decision of the board of special inquiry which recommended for the reversal of the decision of the Board of Commissioners. Acting commissioner issued an order affirming the decision of the Board of Special Inquiry. On August 15, 1990, the Commission on Immigration and Deportatiion ordered the arrest of William and was released upon posting P 200,000 cash bond. Thus on the 29th of the same month, he filed a petition for certiorari and prohibition before the RTC of Manila. A motion to dismiss was filed but denied. Issue:

Private International Law

Page 9

the law of the country where it is stipulated.

Whether or not William Gatchalian is to be declared as a Filipino citizen

William Gatchalian is declared as a Filipino Citizen. Having declared the assailed marriage as valid, respondent William Gatchalian follows the citizenship of his father, a Filipino as legitimate child. Respondent belongs to a class of Filipinos who are citizens of the Philippines at the time of the adoption of the constitution.

However, intestate and testamentary successions, both with respect to the order of succession and to the amount of successional rights and to the intrinsic validity of testamentary provisions, shall be regulated by the national law of the person whose succession is under

In Moy Ya Lim vs. Commissioner of Immigration (41 SCRA 292 [1971]) and in Lee vs. Commissioner of Immigration (supra), this Court declared that:

consideration, may be the nature ofwhatever the property and regardless of the country wherein said property may be found. (10a)

Held:

(e)verytime the citizenship of a person is material or indispensable in a judicial or administrative case, whatever the corresponding court or administrative authority decides therein as to such citizenship is generally not considered as res adjudicata, hence it has to be threshed out again and again as the occasion may demand.

Art. 17.

When the acts referred to are executed before the diplomatic or consular officials of the Republic of the Philippines in a foreign country, the solemnities established by Philippine laws shall be observed in their execution.

An exception to the above rule was laid by this Court in Burca vs. Republic (51 SCRA 248 [1973]), viz: We declare it to be a sound rule that where the citizenship of a party in a case is definitely resolved by a court or by an administrative agency, as a material issue in the controversy, after a full-blown hearing with the active participation of the Solicitor General or his authorized representative, and this finding or the citizenship of the party is affirmed by this Court, the decision on the matter shall constitute conclusive proof of such party's citizenship in any other case or proceeding. But it is made clear that in no instance will a decision on the question of citizenship in such cases be considered conclusive or binding in any other case or proceeding, unless obtained in accordance with the procedure herein stated.

Prohibitive laws concerning persons, their acts or property, and those which have, for their object, public order, public policy and good customs shall not be rendered ineffective by laws or judgments promulgated, or by determinations or conventions agreed upon in a foreign country. (11a) Art. 71.

Thus, in order that the doctrine of res judicata may be applied in cases of citizenship, the following must be present: 1) a person's citizenship must be raised as a material issue in a controversy where said person is a party; 2) the Solicitor General or his authorized representative took active part in the resolution thereof, and 3) the finding or citizenship is affirmed by this Court.

Art. 15.

Art. 16.

Art. 124. Laws relating to family rights and duties, or to the status, condition and legal capacity of persons are binding upon citizens of the Philippines, even though living abroad. (9a) Real property as well as personal property is subject to

Private International Law

All marriages performed outside the Philippines in accordance with the laws in force in the country where they were performed, and valid there as such, shall also be valid in this polygamous, country, except bigamous, or incestuous marriages as determined by Philippine law. (19a)

Doctrine of processual presumption

Civil Code

The forms and solemnities of contracts, wills, and other public instruments shall be governed by the laws of the country in which they are executed.

If the marriage is between a citizen of the Philippines and a foreigner, whether celebrated in the Philippines or abroad, the following rules shall prevail:

1. If the husband is a citizen of the Philippines while the wife is a foreigner, the provisions of this Code shall govern their relations;

Page 10

2.

Art. 815.

If the husband is a foreigner and the wife is a citizen of the Philippines, the laws of the husband's country shall be followed, without prejudice to the provisions of this Code with regard to immovable property. (1325a)

When a Filipino is in a foreign country, he is authorized to make a will in any of the forms established by thehe lawmay of be. the country in which Such will may be probated in the Philippines. (n)

Art. 816.

Art. 818.

Art. 819.

Art. 829.

The will of an alien who is abroad produces effect in the Philippines if made with the formalities prescribed by the law of the place in which he resides, or according to the formalities observed in his country, or in conformity with those which this Code prescribes. (n) Two or more persons cannot make a will jointly, or in the same instrument, either for their reciprocal benefit or for the benef it of a third person. (669) Wills, prohibited by the preceding article, executed by Filipinos in a foreign country shall not be valid in the Philippines, even though authorized by the laws of the country where they may have been executed. (733a) A revocation done outside the Philippines, by a person who does not have his domicile in this country, is valid when it is done according to the law of the plac e where the will was made, or according to the law of the place in which the testator had his domicile at the time; and takes place if inthe thisrevocation country, when it is in accordance with the provisions of this Code. (n)

Art. 1039.

Capacity to succeed is governed by the law of the nation of the decedent. (n)

Rule 132 Sec. 25 What attestation of copy must state . — Whenever a copy of a document or record is attested for the purpose of evidence, the attestation must state, in substance, that the copy is a correct copy of the original, or a specific part thereof, as the case may be. The

Private International Law

attestation must be under the official seal of the attesting officer, if there be any, or if he be the clerk of a court having a seal, under the seal of such court. (26a) Rule 130 Section 45. Commercial lists and the like . — Evidence of statements of matters of interest to persons engaged in an occupation contained in a list, register, periodical, or other published compilation is admissible as tending to prove

the of any relevant matter so if that truth compilation is published for stated use by persons engaged in that occupation and is generally used and relied upon by them therein. (39) Section 46. Learned treatises. — A published treatise, periodical or pamphlet on a subject of history, law, science, or art is admissible as tending to prove the truth of a matter stated therein if the court takes judicial notice, or a witness expert in the subject testifies, that the writer of the statement in the treatise, periodical or pamphlet is recognized in his profession or calling as expert in the subject. (40a) How to acquire nationality 1. by birth

2. 3. by by repatriation naturalization 4. by subrogation and cession how to lose a nationality 1. release 2. deprivation 3. expiration 4. renunciation domicile 1. intent to stay 2. physical presence 3. conduct indicative of such intention Rep. Act No. 9225 signed into law by President Gloria M. Arroyo on August 29, 2003

SECTION 1. Short Title.-This Act shall be known as the "Citizenship Retention and Reacquisition Act of 2003." SEC. 2. Declaration of Policy.-It is hereby declared the policy of the State that all Philippine citizens who become citizens of another country shall be deemed not to have lost their Philippine citizenship under the conditions of this Act. SEC. 3. Retention of Philippine Citizenship.-Any provision of law to the contrary notwithstanding, natural-born citizens of the Philippines who have lost their Philipp ine citizenship by reason of their naturalization as citizens of a foreign country are hereby deemed to have

Page 11

reacquired Philippine citizenship upon taking the following oath of allegiance to the Republic:

"I ___________________________, solemnly swear (or affirm) that I will support and defend the Constitution of the Republic of the Philippines and obey the laws and legal orders promulgated by the duly constituted authorities of the Philippines; and I hereby declare that I recognize and accept the supreme authority of the Philippines and will

4. Those intending to practice their profession in the Philippines shall apply with the proper authority for a license or permit to engage in such practice; and 5. That right to vo te or be elected or appointed to any public office in the Philippines cannot be exercised by, or extended to, those who: a.

are candidates for or are occupying any public office in the country of which they are naturalized citizens; and/or

b.

are in the commissioned officers in the country which citizens.

maintain true faith and allegiance thereto; and that I impose this obligation upon myself voluntarily without mental reservation or purpose of evasion." Natural-born citizens of the Philippines who, after the effectivity of this Act, become citizens of a foreign country shall retain their Philippine citizenship upon taking the aforesaid oath.

active service as or noncommissioned armed forces of the they are naturalized

The SEC. 4. Derivative Citizenship. unmarried child, whether legitimate, illegitimate or adopted, below eighteen (18) years of age, of those who reacquire Philippine citizenship upon effectivity of this Act shall be deemed citizens of the Philippines.

SEC. 6. Separability Clause. - If any section or provision of this Act is held unconstitutional or invalid, any other section or provision not affected thereby shall remain valid and effective.

SEC. 5. Civil and Political Rights and Liabilities. - Those who retain or reacquire Philippine citizenship under this Act shall enjoy full civil and political rights and be subject to all attendant liabilities and responsibilities under existing laws of the Philippines and the following conditions:

the provisions of this Act are hereby repealed or modified accordingly.

1. Those intending to exercise their right of suffrage must meet the requirements under Section 1, Article V of the Constitution, Republic Act No. 9189, otherwise known as "The Overseas Absentee Voting Act of 2003" and other existing laws;

2. Those seeking elective public office in the Philippines shall meet the qualifications for holding such public office as required by the Constitution and existing laws and, at the time of the filing of the certificate of candidacy, make a personal and sworn renunciation of any and all foreign citizenship before any public officer authorized to administer an oath;

3. Those appointed to any public office shall subscribe and swear to an oath of allegiance to the Repub lic of the Philippines and its duly constituted authorities prior to their assumpti on of office: Provided, That they renounce their oath of allegiance to the country where they took that oath;

Private International Law

SEC. 7. Repealing Clause. - All laws, decrees, orders, rules and regulations inconsistent with

SEC. 8. Effectivity Clause. - This Act shall take effect after fifteen (15) days following its publication in the Official Gazette or two (2) newspapers of general circulation. ARTICLE IV CITIZENSHIP

Section 1. Philippines:

The following are citizens of the

1. Those who are citizens of the Philippines at the time of the adoption of this Constitution; 2. Those whose fathers or mothers are citizens of the Philippines; 3. Those born before January 17, 1973, of Filipino mothers, who elect Philippine citizenship upon reaching the age of majority; and

Page 12

4. Those who are naturalized in accordance with law. Section 2.

Natural-born citizens are those who are citizens of the Philippines from birth without having to perform any act to acquire or perfect their Philippine citizenship. Those who elect Philippine citizenship in

themselves with a certificate of legal capacity to contract marriage, to be issued by their respective diplomatic or consular officials. (13a) Art. 1039.

Capacity to succeed is governed by the law of the nation of the decedent. (n)

Art. 50.

For the exercise of civil rights and the fulfillment of civil iciplace le of obligations, the dom natural persons is the of their habitual residence . (40a)

accordance with paragraph (3), Section 1 hereof shall be deemed natural-born citizens. Section 3. Philippine citizenship may be lost or reacquired in the manner provided by law. Section 4.

Citizens of the Philippines who marry aliens shall retain their citizenship, unless by their act or omission, they are deemed, under the law, to have renounced it.

Section 5.

Dual allegiance of citizens is inimical to the national interest and shall be dealt with by law.

NCC Art. 15.

Laws relating to family rights and duties, or to the status, condition and legal capacity of persons are binding upon citizens of the Philippines, even though living abroad. (9a)

Art. 16.

Real property as well as personal property is subject to the law of the country where it is stipulated.

Art. 51.

When the law creating or recognizing them, or any other provision does not fix the domicile of juridical persons, the same shall be understood to be the place where their legal representation is established or where they exercise their principal functions. (41a)

Art. 99.

No person shall be entitled to a legal separation who has not resided in the Philippines for one year prior to the filing of the petition, unless the cause for the legal separation has taken place within the territory of this Republic. (Sec. 2a, Act No. 2710)

Art. 1251.

However, intestate and testamentary successions, both with respect to the order of succession and to the amount of successional rights and to the intrinsic validity of testamentary provisions, shall be regulated by the national law of the person whose succession is under consideration, whatever may be the nature of the property and regardless of the country wherein said property may be found. (10a) Art. 66.

When either or both of the contracting parties are citizens or subjects of a foreign country, it shall be necessary, before a marriage license can be obtained, to provide

Private International Law

Payment shall be made in the place designated in the obligation. There being no express stipulation and if the undertaking is to deliver a determinate thing, the payment shall be made wherever the thing might be at the moment the obligation was constituted. In any other case the place of payment shall be the domicile of the debtor. If the debtor changes his domicile in bad faith or after he has incurred in delay, the additional expenses shall be borne by him. These provisions are without prejudice to venue under the Rules of Court. (1171a)

Page 13

Art. 58.

Save marriages of an exceptional character authorized in Chapter 2 of this Title, but not those under Article 75, no marriage shall be solemnized without a license first being issued by the local civil registrar of the municipality where either contracting party habitually resides. (7a)

taking the oath recognizes and accepts the supreme authority of the Philippines is

an

unmistakable

and

categorical

affirmation of his undivided loyalty to the Republic.

Issue: Whether R.A. 9225 is unconstitutional and whether the court Held:

Art. 829.

A revocation done outside the Philippines, by a person who does not have his domicile in this country , is valid when it is done according to the law of the place where the will was made , or according to the law of the place in which the testator had his domicile at the time; and if the revocation takes place in this country, when it is in accordance with the provisions of this Code. (n)

R.A. 9225 is constitutional and that the Court has no jurisdiction yet to pass upon the issue of dual allegiance. The court held that that the intent of the legislature in drafting Rep. Act No. 9225 is to do away with the provision in Commonwealth Act No. 635 which takes away Philippi ne citizenship from natural-born Filipinos who become naturalized citizens of other countries.

What Rep. Act No. 9225 does is allow dual citizenship to natural-born Filipino citizens who

Cases:

have lost Philippine citizenship by reason of AASJS vs. Datumanong

their naturalization as citizens of a foreign country. On its face, it does not recognize

G.R. No. 160869, May 11, 2007

dual

allegiance.

By

swearing

to

the

Sec. 3, RA 9225 stayed clear of the problem of dual allegiance and shifted the burden of confronting the issue of whether or not there is dual allegiance to the concerned foreign country.

supreme authority of the Republic, the

FACTS:

the burden of confronting the issue of

person implicitly renounces his foreign citizenship. Plainly, from Section 3, Rep. Act No. 9225 stayed clear out of the problem of dual allegiance and shifted whether or not there is dual allegiance to concerned

foreign

country.

What

Petitioner filed the instant petition against

the

respondent, then Secretary of Justice Simeon

happens to the other citizenship was not made

Datumanong, the official tasked to implement

a concern of Rep. Act No. 9225.

laws governing citizenship in order to prevent the Justice Secretary from implementing R. A.

For its part, the OSG counters that pursuant to

9225. Petitioner argues that RA 9225 is

Section 5, Article IV of the 1987 Constitution,

unconstitutional as it violates Sec. 5, Article VI

dual allegiance shall be dealt with by law.

of the Constitution which states that dual allegiance of citizens is inimical to national

Thus, until a law on dual allegiance is enacted by Congress, the Supreme Court is without any

interest and shall be dealt with by law. The

jurisdiction to entertain issues regarding dual

Office of the Solicitor General (OSG) claims that

allegiance.

Section 2 merely declares as a state policy that "Philippine citizens who become citizens of

Moreover, Section 5, Article IV of the

another country shall be deemed not to have

Constitution is a declaration of a policy

lost their Philippine citizenship." The OSG

and it is not a self-executing provision.

further claims that the oath in Section 3 does

The legislature still has to enact the law

not allow dual allegiance since the oath taken

on dual allegiance. In Sections 2 and 3 of

by the former Filipino citizen is an effective

Rep. Act No. 9225, the framers were not

renunciation and repudiation of his foreign

concerned with dual citizenship per se,

citizenship. The fact that the applicant

but with the status of naturalized citizens

Private International Law

Page 14

who maintain their allegiance to their countries

of

naturalization.

origin

even

Congress

after

was

their

given

a

mandate to draft a law that would set specific parameters of what really constitutes dual allegiance. Until this is done, it would be premature including

for this

the Court,

judicial to

rule

department, on

but also over the res, which is the personal status of Baby Rose as well as that of petitioners herein.

issues

pertaining to dual allegiance. ELLIS V. REPUBLIC 7 SCRA 962

Petitioner Marvin G. Ellis, a native of San Fransisco, California and is married to Gloria G. Ellis in Banger, Maine, United States. Both are citizens of the United States. Baby Rose was born on September 26, 1959. Four or five days later, the moth er of Rose left her with the Heart of Mary Villa — an institution for unwed mothers and their babies — stating that she (the mother) could not take of Rose without bringing disgrace upon her (the mother's family.). Being without issue, on November 22, 1959, Mr. and Mrs. Ellis filed a petition with the Court of First Instance of Pampanga for the adoption of the aforementioned baby. At the time of the hearing of the petition on January 14, 1960, petitioner Marvin G. Ellis and his wife had been in the Philippines for three (3) years, he being assigned thereto as staff sergeant in the United States Air Force Base, in Angeles, Pampanga where both lived at that time. They had been in the Philippines before, or, to exact, in 1953.

Art. 15 adheres to the theory that jurisdiction over the status of a natural person is determined by the latters' nationality. Pursuant to this theory, the Court has jurisdiction over the status of Baby Rose, she being a citizen of the Philippines, but not over the statu s of the petitioners, who are foreigners. Under the Philippine’s political law, which is patterned after the Anglo-American legal system, the Court have, likewise, adopted the latter's view to the effect that personal status, in general, is determined by and/or subject to the ur j isdiction of the domiciliary law (Restatement of the Law of Conflict of Laws, p. 86; The Conflict of Laws by Beale, Vol. I, p. 305, Vol. II, pp. 713-714). This, perhaps, is the reason why our Civil Code does not permi t adoption by nonresident aliens, and we have consistently refused to recognize the validity of foreign decrees of divorce — regardless of the grounds upon which the same are based — involving citizens of the Philippines who are not bona fide residents of the forum, even when our laws authorized absolute divorce in the Philippines (Ramirez v. Gmur, 42 Phil. 855; Gonayeb v. Hashim, 30 Phil. 22; Cousine Hix v. Fleumer, 55 Phil. 851; Barretto Gonzales v. Gonzales, 58 Phil. 67; Recto v. Harden, L-6897, Nov. 29, 1955)".

Issue: Republic v. Maddela

Whether or not being permanent residents in the Philippines, petitioners are qualified to adopt Baby Rose. Held: Inasmuch as petitioners herein are not domiciled in the Philippines — and, hence, nonresident aliens – the Court cannot assume and exercise jurisdiction over the status, under either the nationality theory or the domiciliary theory. Article 335 of the Civil Code of the Philippines, provides that: "The following cannot adopt: xxx xxx xxx (4) Non-resident aliens;". xxx xxx xxx This legal provisions is too clear to require interpretation. Since adopt ion is a proceedin gs in rem, no court may entertain unless it has jurisdiction, not only over the subject matter of the case and over the parties,

Private International Law

27 SCRA 702

This is a petition to have the petitioners Miguela Tan Suat, and Chan Po Lan, all Chinese Nationals, to be declared a Filipino citizens. That sometime in the year 1937 Miguela was legally married to Sy Ing Seng, a Filipino citizen; likewise, in the year 1961, Chan Po Lan was legally married to Cu Bon Piao, a Filipino citizen; and that the petitioners have all the qualifications and none of the disqualifications to become Filipino citizens. The court inquired from Fiscal Veluz, who represents the Solicitor General, if he has any opposition to the petition to which the Fiscal answered that he has no opposition. The Court had it announced to the public if there is any opposition to the petitions of both to be declared a Filipino citizen and nobody in the crowded courtroom registered his opposition.

Page 15

As such, the petitioners Miguela Tan Suat and Chan Po Lan were declared a Filipino citizens by marriage and the Commissioner of Immigration is hereby ordered to cancel the necessary alien certificate of registration and immigrant certificate of residence of the petitioner and to issue the corresponding identification card.

Petitioner seeks reconsideration of the decision in this case which reversed that of the Court of First Instance of Leyte declaring her a citizen of the Philippines, the said court have found her to be married to a Filipino citizen and to possess all the qualifications and none of the disqualifications to become Filipino citizen enumerated in the Naturalization Law.

The Solicitor General filed the instant petitions instead, including the Commissioner of Immigration as co-petitioner in view of the fact

Issue:

that the dispositive parts of the decisions of the lower court are addressed to him for compliance.

declaration of citizenship.

Issue:

The Court cannot grant petitioner-appellee's prayer for the affirmance of the trial court's judgment declaring her a Filipino citizen. It must be noted that the sole and only purpose of the petition is to have petitioner declared a Filipino citizen. Under Philippine laws there can be no judicial action or proceeding for the declaration of the citizenship of an individual. It is as an incident only of the adjudication of the rights of the parties to a controversy, that the courts may pass upon, and make a pronouncement relative to, their status.

Whether or not a person claiming to be a citizen may get a judicial declaration of citizenship. Held: Under Philippine laws, there can be no action or proceeding for the judicial declaration of the citizenship of an individual. Courts of justice exist for the settlement of justiciable controversies, which imply a given right, legally demandable and enforceable, an act or omission violative of said

right, and a remedy, granted or sanctioned by law, for said breach of right. As an incident only of the adjudication of the right of the parties to a controversy, the court may pass upon, and make a pronouncement relative to, their status. Otherwise, such a pronouncement is beyond judicial power. Thus, for instance, no action or proceeding may be instituted for a declaration to the effect that plaintiff or petitioner is married, or single, or a legitimate child, although a finding thereon may be made as a necessary premise to justify a given relief available only to one enjoying said status. At times, the law permits the acquisition of a given status, such as naturalization by judicial decree. But there is no similar legislation authorizing the institution of a judicial

proceeding to declare that a given person is a Filipino Citizen. (Tan v. Republic, L-14159, April 18, 1960). Burca v. Republic

51 SCRA 248 1st case: Burca not granted citizenship because such power is granted under the executive branch. Moya Lim Yao: alien wife deemed ipso facto Filipino citizen as long as no disqualifications, no need to prove the qualifications; Burca motion for reconsideration:

Private International Law

Whether or not a court may gran t a judicial

Held:

In Moy Ya Lim Yao, the Court emphasized the administrative procedur e that needs to be followed in the Bureau Immigration regarding the steps to be taken by an alien woman married to a Filipino for the cancellation of her alien certificate of registration, and thus secure recognition of her status Filipino citizen. Such a procedure could be availed of Petitioner. u J dicial recourse would be avoidable to Petitioner in case of an adverse action by the Immigration Commissioner. However, if the decision of an administrative agency on the question of citizenship, is affirmed by this Court on the ground that the same is supported by substantial evidence on the whole record, there appears to be no valid reason why such finding should have no conclusive effect in other cases, where the same issue is involved. The same observation holds true with respect to a decision of a court on the matter of citizenship as a material matter in issue in the case before it, which is affirmed by this Court. For the "effective operation of courts in the social and economic scheme requires that their decision have the respect of and be observed by the parties, the general public and the courts themselves. According insufficient weight to prior decisions encourages disrespect and disregard of courts and their decisions and invites litigation" (Clear, Res Judicata Reexamined, 57 Yale Law Journal, 345).

Page 16

Wherefore, the Court declared it to be a sound rule, that where citizenship of a party in a case is definitely resolved by a court or by an administrative agency, as a material issue in controversy, after a fullblown hearing, with the act participation of the Solicitor Genera l or his authority representative, and this finding on the Citizenship of the party is affirmed by this Court, the decision on the matter shows constitute conclusive proof of such person's citizenship, in another case or

name except one, Rosa. She did not know the names of her brothers-in-law, or sisters-in-law. The Court of First Instance of Manila (Civil Case 49705) denied the prayer for preliminary injunction. Moya Lim Yao and Lau Yuen Yeung appealed.

proceeding. But it is made clear that in instance will a decision on the quest ion of citizenship in such cases be considered conclusive or binding in any other case proceeding, unless obtained in accordance with the procedure herein stated.

Held:

Moy Ya Lim Yao v. Commissioner

41 SCRA 292 Facts: On 8 February 1961 , Lau Yuen Yeung applied for a passport visa to enter the Philippines as a non-immigrant. In the interrogation made in connection with her application for a temporary visitor's visa to enter the Philippines, she state d that she was a Chinese residing at Kowloon, Hongkong, and that she desired to take a pleasure trip to the Philippines to visit her great-grand-uncle Lau Ching Ping for a period of one month. She was permitted to come into the Phili ppines on 13 March 1961, and was permitted to stay for a period of one month which would expire on 13 April 1961. On the date of her arrival, Asher Y, Cheng filed a bond in the amount of P1,000.00 to undertake, among others, that said Lau Yuen Yeung would actually depart from the Philippines on or before the expiration of her authorized period of stay in this country or within the period as in his disc retion the Commissioner of Immigration or his authorized representative might properly allow. After repeated extensions, Lau Yuen Yeung was allowed to stay in the Philippines up to 13 February 1962. On 25 January 1962, she contracted marriage with Moy Ya Lim Yao alias Edilb erto Aguinald o Lim an alleged Filipino citizen. Because of the contemplated action of the Commissioner of Immigration to confiscate her bond and order her arrest and immediate deportation, after the expiration of her authorized stay, she brought an action for injunction with preliminary injunction. At the hearing which took place one and a half years after her arrival, it was admitted that Lau Yuen Yeung could not write either English or Tagalog. Except for a few words, she could not speak either English or Tagalog. She could not name any Filipino neighbor, with a Filipino

Private International Law

Issue: Whether Lau Yuen Yeung ipso facto became a Filipino citizen upon her marriage to a Filipino citizen.

Lau Yuen Yeung, was declared to have become a Filipino citizen from and by virtue of her marriage to Moy Ya Lim Yao al as Edilberto Aguinaldo Lim, a Filipino citizen of 25 January 1962. Under Section 15 of Commonwealth Act 473, an alien woman marrying a Filipino, native born or naturalized, becomes ipso facto a Filipina provided she is not disqualified to be a citizen of the Philippines under Section 4 of the same law. Likewise, an alien woman married to an alien who is subsequently naturalized here follows the Philippine citizenship of her husband the moment he takes his oath as Filipino citizen, provided that she does not suffer from any of the disqualifications under said Section 4. Whether the alien woman requires to undergo the naturalization proceedings, Section 15 is a parallel provision to Section 16. Thus, if the widow of an applicant for naturalization as Filipino, who dies during the proceedings, is not required to go through a naturalization proceedings, in order to be considered as a Filipino citizen hereof, it should follow that the wife of a living Filipino cannot be denied the same privilege. This is plain common sense and there is absolutely no evidence that the Legislature intended to treat them differently. As the laws of our country, both substantive and procedural, stand today, there is no such procedure (a substitute for naturalization proceeding to enable the alien wife of a Philippine citizen to have the matter of her own citiz enship settled and established so that she may not have to be call ed upon to prove it everytime she has to perform an act or enter into a transaction or business or exercise a right reserved only to Filipinos), but such is no proof that the citizenship is not vested as of the date of marriage or the husband's acquisition of citizenship, as the case may be, for the truth is that the situation obtains even as to nativeborn Filipinos. Everytime the citizenship of a person is material or indispensible in a judicial or administrative case, Whatever the corresponding court or administrative authority

Page 17

decides therein as to such citizenship is generally not considered as res adjudicata, hence it has to be threshed out again and again as the occasion may demand.

oath of allegiance incident thereto, states that he does so only in connection with his service to said foreign country: And provided, finally, That any Filipino citizen who is rendering service to, or is commissioned in, the armed forces of a foreign country under any of the circumstances mentioned in paragraph (a) or (b), shall not be permitted to participate

Nota bene:

There are two laws, which govern the Loss of Philippine citizenship. These are Commonwealth Act No. 63 and Commonwealth Act No. 473. The former applies to both natural-born and naturalized citizens and the latter naturalized citizens.

applies

only

to

nor vote in any election of the Republic of the Philippines during the period of his service to, or commission in, the armed forces of said foreign country. Upon his discharge from the service of the said foreign country, he shall be automatically entitled to the full enjoyment of his civil and political rights as a Filipino citizen;

As stated in Commonwealth Act. No. 63, A Filipino citizen may lose his citizenship in any of the following ways and/or events:

1. By naturalization in a foreign country; 2. By

express citizenship;

renunciation

of

3. By

subscribing to an oath of allegiance to support the constitution or laws of a foreign country upon attaining twenty-one years of age or more: Provided, however, That a Filipino may not divest himself of Philippine citizenship in

any mann er while the Republic of the Philippines is at war with any country;

4. By

rendering services to, or accepting commissio n in, the armed forces of a foreign country : Provided, That the rend ering of service to, or the acceptance of such commission in, the armed forces of a foreign country, and the taking of an oath of allegiance incident thereto, with the consent of the Republic of the Philippines, shall not divest a Filipino of his Philippine citizenship if either of the following circumstances is present:

a. The

Republic of the Philippines has a defensive and/or offensive pact of alliance with the said foreign country; or

b.

T he said foreign country maintains armed forces on Philippine territory with the consent of the Repub lic of the Philippines : Provided, That the Filipino citizen concerned, at the time of rendering said service, or acceptance of said commission, and taking the

Private International Law

5. By

cancellation of the certificates of naturalization;

of

the

6. By having been declared by competent authority, a deserter of the Philippine armed forces in time of war , unless subsequently, a plenary pardon or amnesty has been granted; and

7. In the

case of a woman, upon her marriage to a foreigner if, by virtue of the laws in force in her husband's country, she acquires his nationality. Commonwealth Act No. 473, Section 18 of said law provides that:

A naturalization certificate may be cancelled by a competent judge on any of the following grounds:

1. If it is shown that said

naturalization

certificate was fraudulently or illegally;

obtained

2. If the person naturalized shall,

within five years next following the issuance of said naturalization certificated, return to his native country or to some foreign country and establish his permanent residence there: Provided, That the fact of the person naturalized remaining for more than one year in his native country or the country of his former nationality, or two years in any other foreign country, shall be considered prima facie evidence of his

Page 18

intention of taking up his permanent residence in the same;

taking of said oath. On that same date, petitioner took it and the certificate of naturalization was issued to him.

3. If the

petition was made on an invalid declaration of intention

4. If it is shown that the

minor children of the person naturalized failed to graduate from a public high school recognized by the Office of Private Education of the Philippines, where Philippine history, government and

civics are taught as part of the school curriculum, through the fault of their parents either by neglecting to support them or by transferring them to another school or schools. A certified copy of the decree of naturalization certificate shall be forwarded by the Clerk of Court of the Depa rtment of Interior and the Bureau of Justice.

5. If it is shown that the

naturalized citizen has allowed himself to be used as a dummy in violation of the constitutional provisions requiring Philippine citizenship as a requisite for the exercise, use or enjoyment of a right, franchise or privilege.

The main reason why a decision in a naturalization proceeding is not res judicata is because such is not a judicial adversarial proceeding. Similarly, estoppel or laches cannot apply to the government in action for the cancellation of a certificate of naturalization, since it is a known principle that the government is never estopped by the mistakes on the part of its agents. However, according to the Constitution, marriage to an alien would not automatically divest a person of his citizenship, unless he or she performs certain acts or omission which would result to the loss of his or her citizenship. This provision, however, is not retroactive; thus, does not repatriate those who lost their

Philippine citizenship by marriage under the 1935 and 1973 Constitutions. Oh Hek How v. Republic

29 SCRA 94 Petitioner Oh Hek How having been granted naturalization through his petition filed a motion alleging that he had complied with the requirements of Republic Act No. 530 and praying that he be allowed to take his oath of allegiance as such citizen and issued the corresponding certificate of naturalization. The Court of First Instance of Zamboanga del Norte issued forthwith an order authorizing the

Private International Law

The Government seasonably gave notice of its intention to appeal from said order of February 9, 1966 and filed its record on appeal among the grounds that the oath was taken prior to judgment having been final and executor. Issue: Is the oath valid? Whether or not a permission to renounce citizenship is necessary from the Minister of the Interior of Nationalist China. Held: First issue: The order of February 9, 1966 (oathtaking) had not — and up to the present has not — become final and executory in view of the appeal duly taken by the Government.

2nd Issue: It is argue d that the perm ission is not required by our laws and that the naturalization of an alien, as a citizen of the Philippines, is governed exclusively by such laws and cannot be controlled by any foreign law. However, the question of how a Chinese citizen may strip himself of that status is necessarily governed — pursuant to Articles 15 and 16 of our Civil Code — by the laws of China, not by those of the Philippines.

As a consequence, a Chinese national cannot be naturalized as a citizen of the Philippines, unless he has complied with the laws of Nationalist China requiring previous permission of its Minister of the Interior for the renunciation of nationality. •

Section 12 of Commonwealth Act No. 473 provides, however, that before the naturalization certificate is issued, the petitioner shall "solemnly swear," inter alia, that he renounces "absolutely and forever all allegiance and fidelity to any foreign prince, potentate" and particularly to the state "of which" he is "a subject or citizen." The obvious purpose of this requirement is to divest him of his former nationality, before acquiring Philippine citizenship, because, otherwise, he would have two nationalities and owe allegiance to two (2) distinct sovereignties, which our laws do not permit, except that, pursuant to Republic Act No. 2639, "the acquisition of citizenship by a natural-

Page 19

born Filipino citizen from one of the Iberian and any friendly democratic Ibero-American countries shall not produce loss or forfeiture of his Philippine citizenship, if the law of that country grants the same privilege to its citizens and such had been agreed upon by treaty between the Philippines and the foreign country from which citizenship is acquired." Tecson v. Comelec

424 SCRA 277 Facts: On 31 December 2003, Ronald Allan Kelly Poe, also known as Fernando Poe, Jr. (FPJ), filed his certificate of candidacy for the position of President of the Republic of the Philippines under the Koalisyon ng Nagkakaisang Pilipino (KNP) Party, in the 2004 national elections. In his certificate of candidacy, FPJ, representing himself to be a natural-born citizen of the Philippines, stated his name to be "Fernando Jr.," or "Ronald Allan" Poe, his date of birth to be 20 August 1939 and his place of birth to be Manila. Victorino X. Fornier, initiated, on 9 January 2004, a petition before the Commission on Elections (COMELEC) to disqualify FPJ and to deny due course or to cancel his certificate of candidacy upon the thesis that FPJ made a material misrepresentation in his certificate of candidacy by claiming to be a natural-born Filipino citizen when in truth, according to Fornier, his parents were foreigners; his mother, Bessie Kelley Poe, was an American, and his father, Allan Poe, was a Spanish national, being the son of Lorenzo Pou, a Spanish subject. And even if Allan F. Poe was a Filipino citizen, he could not have transmitted his Filipino citizenship to FPJ, the latter being an illegitimate child of an alien mother. Fornier based the allegation of the illegitimate birth of FPJ on two assertions: (1) Allan F. Poe contracted a prior marriage to a certain Paulita Gomez before his marriage to Bessie Kelley and, (2) even if no such prior marr iage had existed, Allan F. Poe, married Bessie Kelly only a year after the birth of FPJ. Issue: Whether FPJ was a natural born citizen, so as to be allowed to run for the offcie of the President of the Philippines. Held: Any conclusion on the Filipino citizenship of Lorenzo Pou coul d only be drawn from the presumption that having died in 1954 at 84 years old, Lorenzo would have been born

Private International Law

sometime in the year 1870, when the Philippines was under Spanish rule , and that San Carlos, Pangasinan, his place of residence upon his death in 1954, in the absence of any other evidence, could have well been his place of residence before death, such that Lorenzo Pou would have benefited from the "en masse Filipinization" that the Philippine Bill had effected in 1902 . That citizenship (of Lorenzo Pou), if acquired, would thereby extend to his son, Allan F. Poe, father of respondent FPJ. The 1935 Constitution, during which regime respondent FPJ has seen first light, confers citizenship to all persons whose fathers are Filipino citizens regardless of whether such children are legitimate or illegitimate.

Board of Commissioners v. Dela Rosa Facts:

On July 12, 1960, Santiago Gatchalian, grandfather of William Gatchalian, was recognized by the Bureau of Immigration as a native born Filipino citizen following the citizenship of natural mother Mariana Gatchalian. On June 27, 1961, Willian, then twelve years old, arrives in Manila from Hongkong together with a daughter and a son of Santiago. They had with them certificate of registration and identity issued by onthe Philippine consulate in Hongkong based a cablegram bearing the signature of the secretary of foreign affairs, Felixberto Serrano, and sought admission as Filipino citizens. On August 15, 1990, the Commission on Immigration and Deportatiion ordered the arrest of William and was released upon posting P 200,000 cash bond. Thus on the 29th of the same month, he filed a petition for certiorari and prohibition before the RTC of Manila. A motion to dismiss was filed but denied. Petitioners, claim that respondent is an alien. In support of their position, petitioners point out that Santiago Gatchalian's marriage with Chu Gim Tee in China as well as the marriage of Francisco (father of William) Gatchalian to Ong Chiu Kiok, likewise in China, were not supported by any evidencenor other own self-serving testimony was than theretheir any showing what the laws of China were. It is the postulate advanced by petitioners that for the said marriages to be valid in this country, it should have been shown that they were valid by the laws of China wherein the same were contracted. There being none, petitione rs conclude that the aforesaid marriages cannot be considered valid. Hence, Santiago's children, including Francisco, followed the citizenship of their mother, having been born outside of a valid marriage. Similarly, the validity of the Francisco's marriage not having been demonstrated, William and Johnson followed the citizenship of their mother, a Chinese national.

Page 20

Issue: Whether or not William Gatchalian is to be declared as a Filipino citizen Held: In Miciano vs. Brimo (50 Phil. 867 [1924]; Lim and Lim vs. Collector of Customs, 36 Phil. 472; Yam Ka Lim vs. Collector of Customs, 30 Phil. 46 [1915]), this Court held that in the absence of evidence to the contrary, foreign laws on a particular subject are presumed to be the same as those of the Philippines. In the case at bar, there being no proof of Chinese law relating to marriage, there arises the presumption that it is the same as that of Philippine law. The lack of proof of Chinese law on the matter cannot be blamed on Santiago Gatchalian much more on respondent William Gatchalian who was then a twelve-year old minor. The fact is, as records indicate, Santiago was not pressed by the Citizenship Investiga tion Board to prove the laws of China relating to marriage, having been content with the testimony of Santiago that the Marriage Certificate was lost or destroyed during the Japanese occupation of China. The testimonies of Santiago Gatchalian and Francisco Gatchalian before the Philippine consular and immigration authorities regarding their marriages, birth and relationship to each other are not self-serving but are admissible in evidence as statements or declarations regarding family reputation or tradition in matters of pedigree (Sec. 34, Rule 130). Philippine law, following the lex loci celebrationis, adheres to the rule that a marriage formally valid where celebrated is valid everywhere. Referring to marriages contracted abroad, Art. 71 of the Civil Code (now Art. 26 of the Family Code) provides that "all marriages performed outside of the Philippines in accordance with the laws in force in the country where they were performed, and valid there as such, shall also be valid in this country . . ."

And any doubt as to the validity of the matrimonial unity and the extent as to how far the validity of such marriage may be extended to the consequences of the coverture is answered by Art. 220 of the Civil Code in this "In case of doubt, all manner: presumptions favor the solidari ty of the family. Thus, every intendment of law or facts leans toward the validity of marriage, the indissolubility of the marriage bonds, the legitimacy of children, the community of property during marriage, the authority of parents over their children, and the validity of defense for any member of the family in case of unlawful aggression." (Emphasis supplied). Bearing in mind the "processual presumption" enunciated in Miciano and other cases, he who asserts that the marriage is not valid under our law bears the burden of proof to present the foreign law.

Private International Law

Having declare d the assailed marriages as valid, respondent William Gatchalian follows the citizenship of his father Francisco, a Filipino, as a legitimate child of the latter. Francisco, in turn is likewise a Filipino being the legitimate child of Santiago Gatchalian who (the latter) is admittedly a Filipino citizen whose Philippine citizenship was recognized by the Bureau of Immigration in an order dated July 12, 1960.

Finally, respondent William Gatchalian belongs to the class of Filipino citizens who became as such at the time of the adoption of the Constitution. . . . Nota bene: for Gatchalian

In Moy Ya Lim vs. Commissioner of Immigration (41 SCRA 292 [1971]) and in Lee vs. Commissioner of Immigration (supra), this Court declared that: Everytime the citizenship of a person is material or indispensable in a judicial or administrative case, whatever the corresponding court or administrative authority decides therein as to such citizenship is generally not considered as r es adjudicata, hence it has to be threshed out again and again as the occasion may demand. An exception to the above rule was laid by this Court in Burca vs. Republic (51 SCRA 248 [1973]), viz: We declare it to be a sound rule that where the citizenship of a party in a case is definitely resolved by a court or by an administrative agency, as a material issue in the controversy, after a full-blown hearing with the active participation of the Solicitor General or his authorized representative, and this finding or the citizenship of the party is affirmed by this Court, the decision on the matter shall constitute conclusive proof of such party's citizenship in any other case or proceeding. But it is made clear that in no instance will a decision on the question of citizenship in such cases be considered conclusive or binding in any other case or proceeding, unless obtained in accordance with the procedure herein stated.

Thus, in order that the doctrine of res judicata may be applied in cases of citizenship, the following must be present: 1) a person's citizenship must be raised as a material issue in a controversy where said person is a party; 2) the Solicitor General or his authorized representative took active part in the resolution thereof, and 3) the finding or citizenship is affirmed by this Court. NUVAL VS. GURRAY

Facts: •

This appeal was taken by the petitioner Gregorio Nuval from the judgment of the Court of First Instance of La Union,

Page 21

upholding the defense of res judicata and dismissing the quo warranto proceedings instituted by the said Gregorio Nuval against Norbeto Guray and others, with costs against the petitioner. •

is Poblacion, Balaoan, La Union; " and in order to be registered in the subscribed affidavit Exhibit F-1 before the board of election inspectors of precinct No. 1 of Balaoan, by virtue of which he was registered as an elector of the said precinct, having made use of the right of suffrage in said municipality in the general elections of 1925. In his cedula certificates issued by himself as municipal treasurer of Balaoan from the year 1923 to 1928,

Gregorio Nuval filed, in his dual capacity as a voter duly qualified and registered in the election list of the municipality of Luna and as a duly registered candidate for the office of municipal president of said municipality, a petition against Norberto Guray asking for the exclusion of his name from the election list of said municipality, not being a qualified voter of said municipality and he had not resided therein for six months as required by section 431 of the said Administrative Code.



Norbeto Guray was elected to the office of municipal president of Luna by a plurality of votes, Gregorio Nuval obtaining second place.



Gregorio Nuval filed the present action of quo warranto asking that Norberto

included, he made it appear that his residence was the residential district of Balaoan. In the year 1926, his wife and children who, up to that time, had lived in the municipality of Balaoan, went back to live in the town of Luna in the house of his wife's parents, due to the high cost of living in that municipality. Norberto Guray used to go home to Luna in the afternoons after office hours, and there he passed the nights with his family. His children studied in the public school of Luna. In January, commenced the 1927, he construction of a house of strong materials in Luna, which has not yet been completed, and neither be nor his family has lived in it. On

Guray be declared ineligible had a legal residence of one year previous to the election as required by section 2174 of the said Administrative Code in order to be eligible to an elective municipal office. •

Norberto Guray had resided in the municipality of Luna, his birthplace, where he had married and had held the office of municipal treasurer. On that date he was appointed municipal treasurer of Balaoan, Province of La Union. The rules of the provincial treasurer of La Union, to which Norberto Guray was subject as such municipal treasurer, require that municipality treasurers live continuously in the municipality where they perform they official duties, in order to be able to give an account of their acts as such treasurers at any time. In order to qualify and be in a position to vote as an elector in Balaoan in the gene ral election of 1925, Norberto Guray asked for the cancellation of his name in the election lists of Luna, where he had voted in the general elections of 1922, alleging as a ground therefore the following: "On the ground of transfer of any residence which took place on the 28th day of June, 1922. My correct and new address

Private International Law

February Norberto Guray applied for1, and1928, obtained vacation leave to be spent in Luna, and on the 16th of the same month he filed his resignation by telegraph, which was accepted on the same day, also by telegraph. Nothwithstanding that he was already provided with a cedula by himself as municipal treasurer of Balaoan on anuary J 31, 1928, declaring him resident of said town, he obtained another cedula from the municipality of Luna on February 20, 1928, which was dated January 15, 1928, in which it is presented that he resided in the barrio of Victoria, municipality of Luna, Province of La Union. On February 23, 1928, Norberto Guray applied for and obtained the cancellation of his name in the election list of the municipality of Balaoan, and on April 14, 1928, he applied for regi stration as a voter in Luna, alleging that he had been residing in said municipality for thirty years. For this purpose he made of the cedula certificate antedated.

Issue: Whether or not Norberto Guray had the legal residence of one year immediately prior to the

Page 22

general elections of June 5, 1928, in order to be eligible to the office of municipal president of Luna, Province of La Union?



Ruling: •

It is an established rule that"where a voter abandons his residence in a state and acquires one in another state, he cannot again vote in the state of his former residence until he has qualified by a new period of residence" (20 Corpus Juris, p. 71, par. 28). "The term 'residence' as so used is synonymous with 'domicile,' which imports not only intention to reside in a fixed place, but also personal presence in that place, coupled with conduct indicative of such intention." (People vs. Bender, 144 N. Y. S., 145.) Since Norberto Guray abandoned his first residence in the municipality of Luna and acquired another in Balaoan, in order to vote and be a candidate in the municipality of Luna, he needed to reacquire residence in the latter municipality for the length of time prescribed by the law, and for such purpose,he needed not only the intention to do so, but his personal presence in said municipality.









VELILLA VS. POSADA •



Facts: •







That Arthur Graydon Moody died in Calcutta, India, on February 18, 1931. That Arthur Graydon Moody executed in the Philippine Islands a will, by virtue of which will, he bequeathed all his property to his only sister , Ida M. Palmer, who then was and still is a citizen and resident of the State of New York, United States of America. That on February 24,1931, a petition for appointment of special administrator of the estate of the deceased Arthur Graydon Moody was filed by W. Maxwell Thebaut with the Court of First Instance of Manila. That subsequently or on April 10, 1931, a petition to the will of the deceased Arthur Graydon Moody, and the same was, after hearing, duly probated by





That on July 14, 1931, Ida M. Palmer was decl ared to be the sole and only heiress of the deceased Arthur Graydon Moody That the property left by the late Arthur Graydon Moody consisted principally of bonds and shares of stock of corporations organized under the laws of the Philippine Islands, bank deposits and other personal properties. That on July 22, 1931, the Bureau of Internal Revenue prepared for the estate of the late Arthur Graydon Moody an inheritance tax return.

Private International Law

attorney forof Ida M. marked Palmer Exhibit another letter, copy which NN is hereto attached and made a part hereof. That the estate of the late Arthur Graydon Moody paid under protest the sum of P50,000 on July 22, 1931, and the other sum of P40,019.75 on January 19, 1932, making assessment for inheritance tax and the sum of P13,001.41 covers the assessment for income tax against said estate. That on January 21, 1932, the Collector of Internal Revenue overruled the protest made by Ida M. Palmer through her attorney. The parties reserve their right to introduce additional evidence at the hearing of the present case. Manila, August 15, 1933. In addition to the foregoing agreed statement of facts, both parties introduced oral and documentary evidence from which it appears that Arthur G. Moody, an American citizen, came to the Philippine Islands in 1902 or 1903 and engaged actively in business in these Islands up to the time of his death in Calcutta, India, on February 18, 1931. He had no business elsewhere and at the time of his death left an estate consisting principally of bonds and shares of stock of corporations organized under the laws of the Philippine Islands, bank deposits and other intangibles and personal property valued by the commissioners of appraisal and claims at P609,767.58 and by the Collector of Internal Revenue for the purposes of inheritance tax at P653,657.47. All of said property at the time of his death was located and had its situs within the Philippine Islands. So far as this record shows, he left no property of any kind located anywhere else. In his will,he made a statement that: Arthur G. Moody, a citizen of the United States of America, residing in the Philippine Islands, hereby publish and declare the following as my last Will and Testament . . ..

the court in a decree dated May 5, 1931. •

That on September 9, 1931, an income tax return for the fractional period from January 1, 1931 to June 30, 1931, was also prepared by the Bureau of Internal Revenue for the estate of the said deceased Arthur Graydon Moody. That on December 3, 1931, the committee on claims and appraisals filed with the court its report. That on November 4, 1931, and in answer to the letter mentioned in the preceding paragraph, the Bureau of Internal Revenue addressed to the

Ruling: To effect the abandonment of one's domicile, there must be a deliberate and provable choice of a new domicile, coupled with actual residence in the place

Page 23

chosen, with a declared or provable intent that it should be one's fixed and permanent place of abode, one's home. There is a complete dearth of evidence in the record that Moody ever established a new domicile in a foreign country.

Finding no merit in any of the assignments of error of the appe llant, the court affirm the judgment of the trial court, first, because the property in the estate of Arthur G. Moody at the time of his death was located and had its situs within the Philippine Islands and, second , because his legal the time of his de ath wasdomicile within up thetoPh ilip pine Islands.

UJANO VS. REPUBLIC

Facts:

Petitioner seeks to reacquire his Philippine citizenship in a petition filed before the Court of First Instance of Ilocos Sur. Petitioner was born 66 years ago of Filipino parents in Magsingal Ilocos Sur. He is married to Maxima O. Ujano with whom he has one son, Prospero, who is now of legal age. He left the Philippines for the United States of America in 1927 where after a residence of more than 20 years he acquired American citizenship by naturalization. He returned to the Philippines on November 10, 1960 to which he was admitted merely for a temporary stay. He owns an agricultural land and a residential house situated in Magsingal, Ilocos Sur. He receives a monthly pension from the Social Security Administration of the United States of America. He has no record of conviction and it is his intention to renounce his allegiance to the U.S.A. After hearing, the court a quo rendered decision denying the petition on the ground that petitioner did not have the residence required by

'residence' requirement in cases of been naturalization, has already interpreted to mean the actual or constructive permanent home otherwise known as legal residence or domicile (Wilfredo Uytengsu vs. Republic of the Philippines, 95 Phil. 890). A place in a country or state where he lives and stays permanently, and to which he intends to return after a temporary absence, no matter how long, is his domicile. In other words domicile is characterized by animus manendi. So an alien who has been admitted into this country as a temporary visitor, either for business or pleasure, or for reasons of health, though actually present in this country cannot be said to have established his domicile here because the period of his stay is only temporary in nature and must leave when the purpose of his coming is accomplished. In the present case, petitioner, who is presently a citizen of the United States of America, was admitted into this country as a temporary visitor, a status he has maintained at the time of the filing of the present petition for reacquisition of Philippine citizenship and which continues up to the present. Such being the case, he has not

complied with six themonths specificresidence requirement of law regarding before filing his present petition." CAASI VS. COURT OF APPEALS

Facts: •

law six months before he filed his petition for reacquisition of Philippine citizenship.

America, not of Bolinao. •

Ruling:

The court a quo , in denying the petition, made the following comment: "One of the qualifications fo r reacquiring Philippine citizenship is that the applicant 'shall have resided in the Philippines at least six months before he applies for naturalization' [Section 3(1), Commonwealth Act No. 63]. This

Private International Law

These two cases were consolidated because they have the same objective; the disqualification under Section 68 of the Omnibus Election Code of the private respondent, Merito Miguel for the position of municipal mayor of Bolinao, Pangasinan, to which he was elected in the local elections of January 18, 1988, on the ground that he is a green card holder, hence, a permanent resident of the United States of

In his answer to both petitions, Miguel admitted that he holds a green card issued to him by the US Immigration Service, but he denied that he is a permanent resident of the United States. He allegedly obtained the green card for convenience in order that he may freely enter the United States for his periodic medical examination and to visit his children there. He alleged that he is a permanent resident of Bolinao, Pangasinan, that he voted in all previous elections, including the plebiscite on February 2,1987 for the

Page 24

ratification of the 1987 Constitution, and the congressional elections on May 18,1987. •

After hearing the consolidated petitions before it, the COMELEC dismissed the petitions on the ground that: The possession of a green card by the respondent (Miguel) does not sufficiently establish that he has abandoned his residence in the Philippines. On the contrary, inspite (sic) of his green card, Respondent has sufficiently to continuouslyindicated reside his in intention Bolinao as shown by his having voted in successive elections in said municipality. As the respondent meets the basic requirements of citizenship and residence for candidates to elective local officials (sic) as provided for in Section 42 of the Local Government Code, there is no legal obstacle to his candidacy for mayor of Bolinao, Pangasinan. (p. 12, Rollo, G.R. No. 84508).

Issues: (1) whether or not a green card is proof that the holder is a permanent resident of the United States, and

(residence and domicile, for purposes of election laws are synonymous…)

The mischief which this provision — reproduced verbatim from the 1973 Constitution — seeks to prevent is the possibility of a "stranger or newcomer unacquainted with the conditions and needs of a community and not identified with the latter, from an elective office to serve that community." Petitioner Romualdez-Marcos filed her Certificate Imelda of Candidacy for the position of Representative of the First District of Leyte with the Provincial Election Supervisor on March 8, 1995. Private respondent Cirilo Roy Montejo, the incumbent Representative of the First District of Leyte and a candidate for the same position, filed a "Petition for Cancellation and Disqualification" with the Commission on Elections alleging that petitioner did not meet the constitutional requirement for residency. In his petition, private respondent contended that Mrs. Marcos lacked the Constitution's one year residency requirement for candidates for the House of Representatives. Held:

(2) whether respondentresident Miguel of had his status as a permanent orwaived immigrant to the U.S.A . prior to the loca l elections on January 18, 1988. Ruling: Despite his vigorous disclaimer, Miguel's immigration to the United States in 1984 constituted an abandonment of his domicile and residence in the Philippines. For he did not go to the United States merely to visit his children or his doctor there; he entered the limited States with the intention to have there permanently as evidenced by his application for an immigrant's (not a visitor's or tourist's) visa. Based on that application of his, he was issued by the U.S. Government the requisite green card or authority to reside there permanently. To be "qualified to run for elective office" in the Philippines, the law requires that the candidate who is a green card holder must have "waived his status as a permanent resident or immigrant of a foreign country." Therefore, his act of filing a certificate of candidacy for elective office in the Philippines, did not of itself cons titute a waiver of his statu s as a permanent resident or immigrant of the United States. The waiver of his green card should be manifested by some act or acts independent of and done prior to filing his candidacy for elective office in this country. Without such prior waiver, he was "disqualified to run for any elective office" (Sec. 68, Omnibus Election Code). MARCOS vs COMELEC

Private International Law

So settled is the concept (of domicile) in our election law that in these and other election law cases, this Court has stated that the mere absence of an individual from his permanent residence without the intention to abandon it does not result in a loss or change of domicile. It stands to reason therefore, that petitioner merely committed an honest mistake in jotting the word "seven" in the space provided for the residency qualification requirement. It would be plainly ridiculous for a candidate to deliberately and knowingly make a statement in a certificate of candidacy which would lead to his or her disqualification. Residence in the civil law is a material fact, referring to the physical presence of a person in a place. A person can have two or more residences, such as a country residence and a city residence. Residence is acquired by living in place; on the other hand, domicile can exist without actually living in the place. The important thing for domicile is that, once residence has been established in one place, there be an intention to stay there permanently, even if residence is also established in some other place. For political purposes the concepts of residence and domicile are dictated by the peculiar criteria of political laws. As these concepts have evolv ed in our elect ion law, what

Page 25

has clearly and unequivocally emerged is the fact that residence for election purposes is used synonymously with domicile. In Nuval vs. Guray, the Court held that "the term residence. . . is synonymous with domicile which imports not only intention to reside in a fixed place, but also personal presence in that place, coupled with conduct indicati ve of such intention." Larena vs. Teves reiterated the

same doctrine in a case involving the qualifications of the respondent therein to the post of Municipal President of Dumaguete, Negros Oriental. Faypon vs. Quirino, held that the absence from residence to pursue studies or practice a profession or registration as a voter other than in the plac e where one is elected does not constitute loss of residence. So settled is the concept (of domicile) in our election law that in these and other election law cases, this Court has stated that the mere absence of an individual from his permanent residence without the intention to abandon it does not result in a loss or change of domicile. In Co vs. Elect oral Tribunal of the House of Representatives, this Court concluded that the framers of the 1987 Constitution obviously adhered to the definition given to the term residence in election law, regarding it as having the same meaning as domicile. A Person cannot have 2 domiciles. As long as the Domicile was not lost, it continues to be the same until replaced by a new one. Marcos did not overtedly abandon her domicile since even if living in Malakanyang, she constantly goes home to her domicile.

Prior to this: The civil code provides that the wife follows the domicile of her husband. JIMENEZ V. REPUBLIC 109 PHIL 273 FACTS:

1. Plaintiff Joel Jimenez filed a complaint praying of a decree annulling his marriage with Remedios Canizares. 2. He claimed that the orifice of her genitals was too small to allow the penetration of a male organ or penis for copulation. 3. He also claimed that the condition of her genitals existed at the time of marriage and continues to exist. 4. The wife was summoned and served with a copy of the complaint but she did not file an answer. 5. The court entered an order requiring defendant to submit to a physical

Private International Law

examination by a competent lady physician to determine her physical capacity for copulation. 6. Defendant did not submit herself to the examination and the court entered a decree annulling the marriage. 7. The City Attorney filed a Motion for Reconsideration, among the grounds that the defendant’s impotency has not been satisfactorily established as required by law; that she had not been physically examined because she refused to be examined. ISSUE:

Whether or not the marriage may be annulled on the strength only of the lone testimony of the husband who claimed and testified that his wife is impotent. HELD:

The law specifically enumerates the legal grounds that must be proved to exist by indubitable evidence to annul a marriage. In the case at bar, the annulment of the marriage in question was decreed upon the sole testimony of the husband who was expected to give testimony tending or aiming at securing the annulment of his marriage he sought and seeks. Whether the wife is really impotent cannot be deemed to have been satisfactorily established because from the commencement of the proceedings until the entry of the decree she had abstained from taking part therein. Although her refusal to be examined or failure to appear in court show indifference on her part, yet from such attitude the presumption arising out of the suppression of evidence could not arise or be inferred because women of this country are by nature coy, bashful and shy and would not submit to a physical examination unless compelled to by competent authority. A physical examination in this case is not selfincriminating. She is not charged with any offense . She is not being compelled to be a witness against herself. “Impotency being an abnormal condition should not be presumed. The presumption is in favor of potency.” The lone testimony of the husband that his wife is physically incapable of sexual intercourse is insufficient to tear asunder the ties that have bound them together as husband and wife. Recto v. Harden 100 Phil 427

Facts: Recto and Harden entered into a contract for professional services wherein the latter

Page 26

engaged the servi ces of the form er as her counsel against her husband for a claim in their conjugal property. Mr. Harden previously filed for divorce against the Mrs. The Court awarded Mrs. Harden an amount totaling to almost 4 million pesos plus litis expensae. Subsequently however, Mrs. Harden ordered her counsel to vacate all orders and judgments rendered therein, and abandon and nullify all her claims to the conjugal partnership existing between her and Mr. Harden. Later, she entered into an amicable settlement with Mr. Harden agreeing to a share of a lesser amount. Appellee counsel for Mrs. Harden alleged that the purpose of the said instruments, executed by Mr. and Mrs. Harden, was to defeat the claim of the former for attorney’s fees, for which reason, he prayed that the court grant him the necessary fees.

Appellants assail the contract for professional services as void, mainly, upon the grounds that:

1. that Mrs. Harden cannot bind the 2. 3.

4.

conjugal partnership without her husband’s consent;c that Article 1491 of the Civil Code of the Philippines in effect prohibits contingent fees; c that the contract in question has for its purpose a decree of divorce, allegedlyto insecure violation of Articles 1305, 1352 and 1409 of the Civil Code of the Philippines; that the terms of said contract are harsh, inequitable and oppressive.

Held: The first objection has no foundation in fact, for the contract in dispute does not seek to bind the conjugal partnership . By virtue of said contract, Mrs. Harden merely bound herself — or assumed the pers onal obligation — to pay, by way of contingent fees, 20% of her share in said partnership. The contract neither gives, nor purports to give, to the Appellee any right whatsoever, personal or real, in and to her aforesaid share. The amount thereof is simply a basis for the computation of said fees.

For the same reason, the second objection is, likewise, untenable. Moreover, it has already been held that contingent fees are not prohibited in the Philippines and are impliedly sanctioned by our Cannons (No. 13) of Professional Ethics. (see, also, Ulanday vs. Manila Railroad Co., 45 Phil., 540, 554.) Such is, likewise, the rule in the United States (Legal Ethics by Henry S. Drinker, p. 176). “in the United States, the great weight of authority recognizes the validity of contracts for contingent fees, provided such contracts are not in contravention of public policy, and it is only when the attorney has taken an unfair or unreasonable advantage of his client that such a claim is condemned .”

Private International Law

(See 5 Am. Jur. 359 et seq; Ballentine, Law Dictionary, 2nd ed., p. 276.)

The third objection is not borne out, either by the language of the contract between them, or by the intent of the parties thereto. Its purpose was not to secure a divorce, or to facilitate or promote the procurement of a divorce. It merely soug ht to protect the interest of Mrs. Harden in the conjugal partnership, during the pendency of a divorce suit she intended to file in the United States. What is more, inasmuch as Mr. and Mrs. Harden are admittedly citizens of the United States, their status and the dissolution thereof are governed — pursuant to Article 9 of the Civil Code of Spain (which was in force in the Philippines at the time of the execution of the contract in question) and Article 15 of the Civil Code of the Philippines — by the laws of the United States, which sanction divorce. In short, the contract of services, between Mrs. Harden and herein Appellee, is not contrary to law, morals, good customs, public order or public policy. It is a basic principle that status, once established by the perso nal law of the party, is given universal recognition. Therefore, aliens can sue and be sued in our courts subject to Philippine procedural law even on matters relating to their status and capacity. However, the law to be applied by Philippine courts in determining their capacity and status is their personal law.

The last objection is based upon principles of equity, but, pursuant thereto, one who seeks equity must come with clean hands (Bastida, et al., vs. Dy Buncio & Co., 93 Phil., 195; 30 C.J. S. 475), and Appellants have not done so, for the circumstances surrounding the case show, to our satisfaction, that their aforementioned agreements, ostensibly for the settlement of the differences between husband and wife, were made for the purpose of circumventing or defeating the rights of herein Appellee, under his above-quoted contract of services with Mrs. Harden. BARNUEVO V. FUSTER 29 PHIL 606

FACTS: Gabriel and Constanza were married in Spain. Thereafter, Gabriel went to the Philippines, settled, and acquired real and personal properties. Constanza later followed. A few years however, both parties executed a contract for their separation wherein the wife returned to Spain and has agreed to be supported by the husband to be paid in Madrid, Spain. Eventually, the wife instituted a petition for divorce here in the Philippines against the husband. The husband opposed the petition on the grounds that:

a. neither the trial court nor any other court in the Philippine Islands has jurisdiction over the subject matter of

Page 27

b.

the complaint, because, as to the allowance for support, since neither the plaintiff nor the defendant are residents of Manila, or of any other place in the Philippine Islands, the agreement upon the subject was neither celebrated, nor was it to be fulfilled, in the Philippine Islands; and as to th e divorce, because th e action therefore ought to be tried by the ecclesiastical courts.

In deciding the case, the Court of First Instance of the city of Manila held itself to have jurisdiction, decreed the suspension of life in common between the plaintiff and defendant; Issue: Do Philippine Courts have jurisdiction over the petition for divorce? What law should apply in construing the term pesetas?

That by the express provision of article 80 of the Civil Code of Spain, "jurisdiction in actions for divorce and nullification of canonical marriages lies with ecclesiastical courts," while that of civil tribunals is limited to civil marriages; that this being so, the action for divorce brought by the plaintiff in the cause does not fall within the jurisdiction of the civil courts , according to his own law of persons, because these courts ought to apply the Spanish law in accordance with the said article 9 of the Civil Cod of Spain, and this Spanish law grants the jurisdiction over the present cause to the ecclesiastical courts, in the place of which no tribunal of these Islands con subrogate itself. However, husband was unable to prove by any law or legal doctrine whatever that the personal statute of a foreigner carries with it, to whether he transfers his domicile, the authority established by the law of his nation to decree his divorce, which was what he had to demonstrate.

Held: As the Court upheld: The lower court did not commit this error attributed to him. The defendant had not proved that he had elsewhere a legal domicile other than that which he manifestly had in the Philippi nes during the seventeen years preceding the date of the complaint . On the contrary, it plainly appears, without proof to the contrary, that during this not inconsiderable period, extending from the year 1892 until a month prior to the arrival of his wife in the Philippines in March, 1909, he had constantly resided in the said Islands, had kept open house, and had acquired in the city of Manila quite a little real property which is now the object of the division of the conjugal society. It has been established that defendant is domiciled in the Philippines. Defendant, although a Spanish subject, was a resident of these Islands. Article 26 of the Civil Code that he cites itself provides that "Spaniards who change their domicile to a foreign country, where they may be considered as natives without other conditions than that of residents therein, shall be required, in order to preserve the Spanish nationality, to state that such is their wish before the Spanish diplomatic or consular agent, who must record them in the registry of Spanish residents, as well as their spouses, should they be married, and any children they may have." From this provision, which is the exclusive and irrefutable law governing the defendant, we are to conclude that the domicile of the defendant and the plaintiff is fully proven, irrespective of the Treaty of Paris. Section 377 of the Code of Civil Procedure leaves to the election of the plaintiff the bringing of a personal action like the one at bar either in the place where the defendant may reside or be found, or in that where the plaintiff resides. As held by the Husband:

Private International Law

"The jurisdiction of courts and other questions relating to procedure are considered to be of a public nature and consequently are generally submitted to the territorial principle. . . . All persons that have to demand justice in a case in which foreigners intervene, since they can gain nothing by a simple declaration, should endeavor to apply to the tribunales of the state which have coercive means (property situated in the territory) to enforce any decision they may render. Otherwise, one would expose himself in the suit to making useless expenditures which, although he won his case, would not contribute to secure his rights because of the court's lack of means to enforce them." (Torres Campos, "Elementos de Derecho International Privado," p. 108.) The provi sions of artic le 80 of the Civil Law of Spain is only binding within the dominions of Spain. It does not accompany the persons of the Spanish subject wherever he may go. He could not successfully invoke it if he resided in Japan, in China, in Hongkong or in any other territory not subject to the dominion of Spain. Foreign Catholics domiciled in Spain, subject to the ecclesiastical courts in actions for divorce

according to the said article 80 of the Civi l Code, could not allege lack of jurisdiction by invoking, as the law of their personal statute, a law of their nation which gives jurisdiction in such a case to territorial courts, or to a certain court within or without the territory of their nation.

(Benedicto vs. De la Rama, 3 Phil. Rep., 34, and Ibañez vs. Ortiz, 5 Phil. Rep., 325). In the present action for divorce the Court of First Instance of the city of Manila did not lack jurisdiction over the persons of the litigants, for, although Spanish Catholic subjects, they were residents of this city and had their domicile herein.

Page 28

The Courts of First Instance of the Philippine Islands have the power and jurisdiction to try actions for divorce. That of the city of Manila did not lack jurisdiction by reason of the subject matter of the litigation. With respect to their property regime, the Foral Law presented by the husband in an affidavit, with which conjugal partnership is known to be inexistent, cannot apply since this affidavit was never presente d in proof, was never received by the trial judge, and cannot seriously be considered as an ef fort to estaSections blish th300, e law a foreign jurisdiction. 301ofand 302 of the Code of Civil Procedure, now in force in these islands, indicate the method by which the law of a foreign country may be proved. The Court maintains that the affidavit of a person not versed in the law, which was never submitted as proof, never received by the trial court , and which has never been subjected to any cross-examination, is not a means of proving a foreign law on which the defendant relies. Thus, since no proof has been submitted to this effect, all the property of the marriage, says article 1407 of the Civil Code, shall be considered as conjugal property until it is proven that it belongs exclusively to the husband or to the wife. ON PLAINTIFF’S APPEAL

The court did not commit it in applying the rule contained in article 1287 of the Civil Code. "The usages or customs of the country shall be taken into consideration in interpreting ambiguity in contracts. . . ." If in the contract the word " pesetas," not being specific, was ambiguous, then it was in harmony with this precept to interpret it as being the peseta then in use or current when and where the agreement was made , Mexican being then the usual and current money in the Philippines. QUITA V. PADLAN GR NO. 124371, DECEMBER 22, 1998

FE D. QUITA and Arturo T. Padlan, both Filipinos, were married in the Philippines on 18 May 1941. They were not however blessed with children. Somewhere along the way their relationship soured. Eventually Fe sued Arturo for divorce in San Francisco, California, U.S.A. and obtained a final judgment of divorce. She married thrice thereafter. On 1972 Arturo died. He left no will. Respondent Blandina Padlan claiming to be the surviving spouse of Arturo Padlan, and Claro, Alexis, Ricardo, Emmanuel, Zenaida and Yolanda, all surnamed Padlan, named in the children of Arturo Padlan opposed the petition. Ruperto T. Padlan, claiming to be the sole surviving brother of the deceased Arturo, also intervened. The court held that no dispute exists as to the right of the six (6) Padlan children to inherit from the decedent because there are proofs that they have been duly acknowledged by him

Private International Law

and petitioner herself even recognizes them as heirs of Arturo Padlan; Issue: Whether or not petitioner could inherit as the surviving spouse of Arturo? Held: Case remanded subject to determination of petitioner’s citizenship. However, Private respondent's claim heirship She was and already Arturo resolved by the trial to court. were married on 22 April 1947 while the prior marriage of petitioner and Arturo was subsisting thereby resulting in a bigamous marriage considered void from the beginning under Arts. 80 and 83 of the Civil Code. Consequently, she is not a surviving spouse that can inherit from him as this status presupposes a legitimate relationship. Case remanded to the court a quo for further proceedings since the trial court was not able to completely ascertain petitioner’s citizenship. The trial court must have overlooked the materiality of this aspect. Once proved that she was no longer a Filipino citizen at the time of their divorce, Van Dorn would become applicable and petitioner could very well lose her right to inherit from Arturo. Nota bene: Van dorn: ang bana ga laot sa wife since under Philippine law, they’re still married. LLORENTE V. COURT OF APPEALS GR No. 124371, November 23, 2000

FACTS:

Lorenzo Llorente and petitioner Paula Llorente were married in 1937 in the Philippines. Lorenzo was an enlisted serviceman of the US Navy. Soon after, he left fo r the US where through naturalization, he became a US Citizen. Upon his visit to his wife, he discovered that she was living with his brother and a child was born. The child was registered as illegitimate but the name of the father was left blank. Llorente filed a divorce in California in which Paula was represented by counsel, John Riley, and actively participated in the proceedings, which later on became final. He married Alicia and they lived together for 25 years bringing 3 children. He made his last will and testament stating that all his properties will be given to his second marriage. He filed a petition of probate that made or appointed Alicia his special administrator of his estate. Before the proceeding could be terminated, Lorenzo died. Paula filed a letter of administration over Llorente’s estate. The trial granted the letter and denied the motion for reconsideration. An appeal was made to the

Page 29

Court of Appeals, which affirmed and modified the judgment of the Trial Court that she be declared co-owner of whatever properties, she and the deceased, may have acquired in their 25 years of cohabitation. ISSUE:

Whether or not national law shall apply?

Nota bene: The following are the requisites of res judicata: 1. the former judgment must be final; 2. the court that rendered it had jurisdiction over the subject matter and the parties; 3. it is a judgment on the merits; and 4. there is — between the first and the second actions — an identity of parties, subject matter and cause of action.

RULING: “Art. 15. Laws relating to family rights and duties, or to the status, condition and legal capacity of persons are binding upon citizens of the Philippines , even though living abroad. “Art. 16. Real property as well as personal property is subject to the law of the country where it is situated. First, there is no such thing as one American law. The "national law" indicated in Article 16 of the Civil Code cannot possibly apply to general American law. There is no such law governing the validity of testamentary provisions in the United States. Each State of the unio n has its own law applicable to its citizens and in forc e only within the State. It can therefore refer to no other than the law of the State of which the decedent was a resident. Second, there is no showing that the application of the renvoi doctrine is called for or required by New York State law.

It has been held that in order that a judgment in one action can be conclusive as to a particular matter in another action between the same parties or their privies, it is essential that:

1. the issue be identical; 2. If a particular po int or qu estion is in issue in the second action, 3. and the ju dgment will depend on th e determination of that particular point or question, 4. a former judgment between th e same parties or their privies will be final and conclusive in the second if that same point or question was in issue and adjudicated in the first suit (Nabus v. Court of Appeals, 193 SCRA 732 [1991]). 5. Identity of cause of action is not required but merely identity of issue. Art. 15.

Laws relating to family rights and duties, or to the status, condition and legal capacity of persons are binding upon citizens of the Philippines, even though living abroad. (FRDSCL)

“However, intestate and testamentary succession, both with respect to the order of succession and to the amount of successional rights and to the intrinsic validity of testamentary provisions, shall be regulated by the national law of the person whose succession is under consideration , whatever may be the nature of the property and regardless of the country wherein said property may be found.” (emphasis ours)

Personal status – legal position of an individual in a society Capacity – power to acquire and exercise rights Incidental to personal status

Likewise, Lorenzo Llorente was already an American citizen when he divorced Paula . Such was also the situation when he married

Legislative jurisdiction – authority of the state of his nationality or domicile or where he may be physically present to promulgate laws

stat in Alicia his code, will. As Art icleand 15 executed of the civil alien sed may obtain divorces abroad, provided that they are valid in their National Law . Thus the divorce obtained by Llorente is valid because the law that governs him is not Philippine Law but his National Law since the divorce was contracted after he became an American citizen. Furthermore, his National Law allowed divorce.

affecting his status; Judicial jurisdiction – the authority of the court to hear and determine the cause of action

Foreign laws regulating the person’s status and capacity are to be disregarded where they are political or penal in character;

Beginning of personality – conception provided subsequent birth End – death Other questions of status

1. The case was remanded to the court of srcin for determination of the intrinsic validity of Lorenzo Llorente’s will and determination of the parties’ successional rights allowing proof of foreign law.

Private International Law

Absence – a special legal status, if one is not in his domicile and his whereabouts is unknown; When can be declared: under civil code – a. Lapse of 2 years wit hout ne ws or since the receipt of last news

Page 30

b.

Lapse of 5 years if abs entee has left someone in charge of the administration of his properties

Who may declare: a. Spouse b. Heirs c. Relatives who will benefit in intestacy d. Those who have rights over the properties

2.

Art. XV, Sec. 2, 1987 Constitution Marriage, as an inviolable social institution, is the foundation of the family and shall be protected by the State. NCC Art. 15.

Laws relating to family rights and duties, or to the status, condition and legal capacity of persons are binding upon citizens of the Philippines, even though living abroad. (9a)

Art. 17.

The forms and solemnities of contracts, wills, and other public instruments shall be governed by the laws of the country in which they are executed.

Conflicts problems respecting an individual’s name and extent of protection against abuse of his name

Right to use a title of nobility is determined in accordance with his national law 3. Minority CAPACITY

1.

2.

When the acts referred to are executed before the diplomatic or consular officials of the Republic of the Philippines in a foreign country, the solemnities established by Philippine laws shall be observed in their execution.

Juridical capacity – the f itness to be the subject of legal relations; considered inherent in every natural person and is lost only through death; Capacity to ac t – the p ower to d o acts with legal effects; it is acquired and may be lost;

Nota bene:

Prohibitive laws concerning persons, their acts or property, and those which have, for their

Under Philippine law, the capacity to dispose of real property is governed by the lex situs while the capa city to succeed is governed by the national law of the deceased.

object, public order, public policy and good customs shall not be rendered ineffective by laws or judgments promulgated, or by determinations or conventions agreed upon in a foreign country. (11a)

Question: A 16-year old Cambodian inherited real properties from the Philippines; he died and left a will which disposes of his properties in the Philippines; supposing, Cambodian law provides that age of majority is 16, is the will now if probated in the Philippines valid?

Art. 66.

When either or both of the contracting parties are citizens or subjects of a foreign country, it shall be necessary, before a marriage license can be obtained, to provide themselves with a certificate of legal capacity to contract marriage, to be issued by their respective diplomatic or consular officials. (13a)

Art. 71.

All marriages performed outside the Philippines in accordance with the laws in force in the country where they were performed, and valid there as such, shall also be valid in this country, except bigamous, polygamous, or incestuous marriages as determined by Philippine law. (19a)

Art. 75.

Marriages between Filipino citizens abroad may be solemnized by consuls and vice-consuls of the Republic of the Philippines. The duties of

If an 18-year old foreigner, where in his country, the age of majority is 21 goes to the Philippines and gets married, is such marriage valid? Family rights, duties, status, condition and legal capacity of persons accompany a person even when he moves to a foreign country;

MARRIAGE

Art. II Section 12, 1987 Constitution The State recognizes the sanctity of family life and shall protect and strengthen the family as a basic autonomous social institution. It shall equally protect the life of the moth er and the life of the unborn from conception. The natural and primary right and duty of parents in the rearing of the youth for civic efficiency and the development of moral character shall receive the support of the Government.

Private International Law

Page 31

the local civil registrar and of a judge or justice of the peace or mayor with regard to the celebration of marriage shall be performed by such consuls and vice-consuls. (n) FC Article 1.

Marriage is a special contract of permanent union between a man and a woman entered into in accordance with law for the establishment of conjugal and family life. It is the foundation

Article 29 of this Code, or where both of the parties request the solemnizing officer in writing in which case the marriage may be solemnized at a house or place designated by them in a sworn statement to that effect. (57a) FC Art. 10.

Marriages between Filipino citizens abroad may be solemnized by a consulgeneral, consul or vice-consul of the Republic of the Philippines. The issuance of the marriage license and the duties of the local civil registrar and of the solemnizing officer with regard to the celebration of marriage shall be performed by said consular official. (75a)

Art. 26.

All marriages solemnized outside the Philippines, in accordance with the laws in force in the country where they were solemnized, and valid there as such, shall also be valid in this country, except those prohibited under Articles 35 (1), (4), (5) and (6), 3637 and 38. (17a)

of the institution family andwhose an inviolable social nature, consequences, and incidents are gove rned by law and not subject to stipulation, except that marriage settlements may fix the property relations during the marriage within the limits provided by this Code. (52a) Art. 7. by:

Marriage may be solemnized

1. 2.

3.

4.

5.

Article. 8.

Any incumbent me mber of the judiciary within the court's jurisdiction; Any priest, rabbi, imam, or minister of any church or religious sect duly authorized by his church or religious sect and registered with the civil registrar general, acting within the limits of the written authority granted by his church or religious sect and provided that at least one of the contracting parties belongs to the solemnizing officer's church or religious sect; Any ship captain or airplane chief only in the case mentioned in Article 31; Any military commander of a unit to which a chaplain is assigned, in the absence of the latter, during a military operation, likewise only in the cases

Where a marriage between a Filipino citizen and a foreigner is validly celebrated and a divorce is thereafter validly obtained abroad by the alien spouse capacitating him or her to remarry, the Filipino spouse shall have capacity to remarry under Philippine law. (As amended by Executive Order 227)

FC Art. 35. The following marriages shall be void from the beginning: 1. Those contracted by any party below eighteen years of age even with the consent of parents or guardians; 2. Those sol emnized by any

mentioned in Article 32; Any consul-general, consul or vice-consul in the case provided in Article 10. (56a)

The marriage shall be solemnized publicly in the chambers of the judge or in open court, in the church, chapel or temple, or in the office the consul-general, consul or vice-consul, as the case may be, and not elsewhere, except in cases of marriages contracted on the point of death or in remote places in accordance with

Private International Law

3.

4. 5.

person not legally authorized to perform marriages unless such marriages were contracted with either or both parties believing in good faith that the solemnizing officer had the legal authority to do so; Those solemnized without license, except those covered the preceding Chapter; Those bigamous or polygamous marriages not failing under Article 41; Those contracted through mistake of one contracting

Page 32

6.

Art. 36.

party as to the identity of the other; and Those subsequent marriages that are void under Article 53.

apply to marriage and divorce wherein both parties are Muslims, or wherein only the male party is a Muslim and the marriage is solemnized in accordance with Muslim law or this Code in any part of the Philippines. (2) In case of marriage between a Muslim and a non-Muslim, solemnized not in accordance with Muslim law or this Code, the Civil Code of the Philippines shall apply. .chan robles virtual law library (3) Subject to the provisions of the preceding paragraphs, the essential requisites and legal impediments to marriage, divorce, paternity and filiation, guardianship and custody of minors, support and maintenance, claims for customary dower (mahr), betrothal, breach of contract to marry, solemnization and registration of marriage and divorce, rights and obligations between husband and wife parental authority, and the properly relations between husband and wife shall be governed by this Code and other applicable Muslim laws.

A marriage contracted by any party who, at the time of the celebration, was psychologically incapacitated to comply with the essential marital obligations of marriage, shall likewise be void even if such becomes manifestincapacity only after its solemnization. (As amended by Executive Order 227)

Art. 37.

Marriages between the following are incestuous and void from the beginning, whether relationship between the parties be legitimate or illegitimate: 1. 2.

Between ascendants and descendants of any degree; and Between brothers and sisters, whether of the full or half blood. (81a)

Art. 38. The following marriages shall be void from the beginning for reasons of public policy: 1.

2. 3. 4. 5.

6. 7. 8. 9.

Muslim Code 13. Application. —

Between collateral blood relatives whether legitimate or illegitimate, up to the fourth civil degree; Between step-parents and step-children; Between parents-in-law and children-in-law; Between the adopting parent and the adopted child; Between the surviving spouse of the adopting parent and the adopted child; Between the surviving spouse of the adopted child and the adopter; Between an adopted child and a legitimate child of the adopter; Between adopted children of the same adopter; and Between parties where one, with the intention to marry the other, killed that other person's spouse, or his or her own spouse. (82) PD

108 3,

Nature. — Marriage is not only a civil contract but a social institution. Its nature, consequences and incidents are governed by this Code and the Shari'a and not subject to stipulation, except that the marriage settlements may to a certain extent fix the property relations of the spouses.

Art. 180.

The Law applicable. — provisions of the Revised

Art.

(1) The provisions of this Title shall

Private International Law

Art. 14.

Page 33

3.

Penal Code relative to the crime of bigamy shall not apply to a person married in accordance with the provisions of this Code or, before its effectivity, under Muslim law.

4. 5.

Essential and Formal Requisites FC Art. 2. No marriage shall be valid, unless these essential requisites are present: 1.

2.

Art. 3. The marriage are:

Legal capacity of the contracting parties who must be a male and a female; and Consent freely given in the presence of the solemnizing officer. (53a) formal

1. 2.

3.

requisites

6.

Art. 4. The absence of any of the essential or formal requisites shall render the marriage void ab initio , except as stated in Article 35 (2). A defect in any of the essential requisites shall not affect the validity of the marriage but the party or parties responsible for the irregularity shall be civilly, criminally and administratively liable. (n)

of

Authority of the solemnizing officer; A valid marriage license except in the cases provided for in Chapter 2 of this Title; and A marriage ceremony which takes place with the appearance of the contracting parties before the solemnizing officer and their personal declaration that they take each other as husband and wife in the presence of not less than two witnesses of legal age. (53a, 55a)

Art. 35 (2) supra Muslim Code, requisites. —

The

Art.

15. Essential

No marriage contract shall be perfected unless the following essential requisites are compiled with:

a. Legal

capacity of the contracting parties; b. Mutual consent of the parties freely given; c. Offer (ijab) and acceptance (qabul) duly witnessed by at least two competent persons after the proper guardian in marriage (wali) has given his consent; and d. Stipulation of customary dower (mahr) duly witnessed by two competent persons.

Art. 5. Any male or female of the age of eighteen years or upwards not under any of the impediments mentioned in Articles 37 and 38, may contract marriage. (54a) Art. 35.

Those solemnized without license, except those covered the preceding Chapter; Those bigamous or polygamous marriages not failing under Article 41; Those contracted through mistake of one contracting party as to the identity of the other; and Those subsequent marriages that are void under Article 53.

following

marriages shall be void from the beginning: 1.

Those contracted by any party below eighteen years of age even with the consent of parents or guardians; 2. Those solemnized by any person not legally authorized to perform marriages unless such marriages were contracted with either or both parties believing in good faith that the solemnizing officer had the legal authority to do so;

Private International Law

GOITIA VS. CAMPOS RUEDA Facts:

The parties were legally married in the city of Manila on January 7, 1915, and immediately thereafter established their residence at 115 Calle San Marcelino, where they lived together for about a month, when the plaintiff returned to the home of her parents. That the defendant, one month after he had contracted marriage with the plaintiff, demanded of her that she perform uncha ste

Page 34

and lascivious acts on his genital organs; that the plaintiff spurned the obscene demands of the defendant and refused to perform any act other than legal and valid cohabitation; that the defendant, since that date had continually on other successive dates, made similar lewd and indecorous demands on his wife, the plaintiff, who always spurned them, which just refusals of the plaintiff exasperated the defendant and induce him to maltreat her by word and deed and inflict injuries upon her lips, her face and different parts of her body; and that, as the plaintiff was unable by any means to induce the defen dant to desist from his repugnant desires and cease from maltreating her, she was obliged to leave the conjugal abode and take refuge in the home of her parents. Issue: whether or not that the wife may claim for support against her husband outside of their conjugal abode? Ruling: marriage partakes of the nature of an ordinary contract. But it is something more than a mere contract. It is a new relation, the rights, duties, and obligations of which rest not upon the agree ment of the parti es but upon the general law which defines and prescrib es those rights , duties, and obligations. Marriage is an institution, in the maintenance of which in its purity the public is deeply interested. It is a relation for life and the parties cannot terminate it at any shorter period by virtue of any contract they may make .The reciprocal rights arising from this relation, so long as it continues, are such as the law determines from time to time, and none other. When the legal existence of the parties is merged into one by marriage, the new relation is regulated and controlled by the state or government upon principles of public policy for the benefit of society as well as the parties. And when the object of a marriage is defeated by rendering its continuance intolerable to one of the parties and productive of no possible good to the community, relief in some way should be obtainable. With these principles to guide us, we will inquire into the status of the law touching and governing the question under consideration. The mere act of marriage creates an obligation on the part of the husband to support his wife. This obligation is founded not so much on the express or implied terms of the contract of marriage as on the natural and legal duty of the husband; an obligation, the enforcement of which is of such vital concern to the state itself that the laws will not permit him to terminate it by his own wrongful acts in driving his wife to seek protection in the parental home. A judgment for separate maintenance is not due and payable either as damages or as a penalty; nor is it a debt in the strict legal sense of the term, but rather a judgment calling for the performance of a duty made specific by the mandate of the sovereign. This is done from necessity and with a view to preserve the public peace and the puri ty of the wife ; as where the husband makes so base demands upon his wife and indulges in the habi t of assaulting her. The pro tanto separation

Private International Law

resulting from a decree for separate support is not an impeachment of that public policy by which marriage is regarded as so sacred and inviolable in its nature; it is merely a stronger policy overruling a weaker one; and except in so far only as such separation is tolerated as a means of preserving the public peace and morals may be considered, it does not in any respect whatever impair the marriage contract or for any purpose place the wife in the situation of a feme sole. Where the wife, who is forced to leave the conjugal abode by her husband without fault on her part, may maintain an action against the husband for separate maintenance when she has no other remedy, notwithstanding the provisions of article 149 of the Civil Code giving the person who is obliged to furnish support the option to satisfy it either by paying a fixed pension or by receiving and maintaining in his own home the one having the right to the same.

WONG WOO YIU VS. VIVO Facts:

The Board of Special Inquiry No. 3 rendered a decision finding petitioner to be legally married to Perfecto Blas and admitting her into the country as a non-quota immigrant. This decision was affirmed by the Board of Commissioners of which petitioner was duly informed in a letter sent on the same date by the Secretary of the Board. However, the same Board of Commissioners, but composed entirely of a new set of members, rendered a new decision reversing that of the Board of Special Inquiry No. 3 and ordering petitioner to be excluded from the country. Petitioner filed a motion for new trial requesting an opportunity to clarify certain points taken in the decision, but the same was denied for lack of merit. Whereupon, petitioner initiated the instant petition for mandamus with preliminary injunction before the Court of First Instance of Manila which incidentally was considered by it as a petition for certiorari. Petitioner declared that she came to the Philippines in 1961 for the first time to join her husband Perfecto Blas to whom she was married in Chingkang, China ;that they had several children all of whom are not in the Philippines; that their marriage was celebrated by one Chua Tio , a village leader; that on June 28, 1961 the Boar d of Special Inquiry No. 3 rendered a decision finding, among others, that petitioner is legally married to Perfecto Blas, a Filipino Citizen, and admitted her into the country as a non-quota immigrant; that this decision was affirmed by the Board of Commissioners of which petitioner was duly notified by the Secretary of said Board in a letter dated July 12, 1961; that in a motu proprio decision rendered by the Board of Commissioners composed of a new set of members the latter found that petitioner's claim that she is the lawf ul wife of Perfecto

Page 35

Blas was without basis in evidence as it was "bereft of substantial proof of husband-wife relationship"; that said Board further held that, it appearing that in the entry proceedings of Perfecto Blas had on January 23, 1947 he declared that he first visited China in 1935 and married petitioner in 1936, it could not possibly sustain her claim that she married Perfecto Blas in 1929; that in an affidavit dated August 9, 1962 Perfecto Blas claimed that he went to China in 1929, 1935 and 1941, although in his re-entry declaration he admitted that he first went to China in 1935, then in 1937, then in 1939, and lastly in 1941; and that Perfecto Blas in the same affidavit likewise claimed that he first went to China when he was merely four years old so that computed from his date of birth in 1908 it must have been in 1912. In view of the discrepancies found in the statements made by petitioner and her alleged husband Perfecto Blas in the several investigations conducted by the immigration authorities concerning their alleged marriage before a village leader in China in 1929, coupled with the fact that the only basis in support of petitioner's claim that she is the lawful wife of Perfecto Blas is "a mass of oral and documentary evidence bereft of substantial proof of husband-wife relationship," the Board of Commissioners motu proprio reviewed the record concerning the admission of petitioner into the country resulting in its finding that she was improperly admitted. Issue: whether or not that the marr iage is valid? Ruling: there is no documentary evidence to support the alleged marriage of petitioner to Perfecto Blas but the record is punctured with so many inconsistencies which cannot but lead one to doubt their veracity concerning the pretended marriage in China in 1929. This claim cannot also be entertained under our law on family relations. Thus, Article 15 of our new Civil Code provides that laws relating to family rights or to the status of persons are binding upon citizens of the Philippines, even though living abroad, and it is well-known that in 1929 in order that a marriage celebrated in the Philippines may be valid it must be solemnized either by a judge of any court inferior to the Supreme Court, a justice of the peace, or a priest or minister of the gospel of any denomination duly registered in the Philippine Library and Museum (Public Act 3412, Section 2). Even if we assume, therefore, that the marriage of petitioner to Perfecto Blas before a village leader is valid in China, the same is not one of those authorized in our country. A marriage contracted outside of the Philippines which is valid under the law of the country in which it was celebrated is also valid in the Philippines. But no validity can be given to this contention because no proof was presented relative to the law of marriage in China. Such being the case, we should apply the general rule that in the absence of proof of the law of a foreign

Private International Law

country it should be presumed that it is the same as our own. Since our law only recognizes a marriage celebrated before any of the officers mentioned therein, and a village leader is not one of them, it is clear that petitioner's marriage, even if true, cannot be recognized in this jurisdiction. ADONG VS. CHAONG SENG GEE Facts: Cheong Boo, a native of China died in Zamboanga, Philippine Islands on August 5, 1919 and left property worth nearly P100,000 which is now being claimed by two parties - (1) Cheong Seng Gee who alleged that he was a legitimate child by marriage contracted by Cheong Boo with Tan Bit in China in 1985, and (2) Mora Adong who alleged that she had been lawfully married to Cheong Boo in 1896 in Basilan, Philippine Islands and had two daughters with the deceased namely Payang and Rosalia. The conflicting claims to Cheong Boo’s estate were ventilated in the lower court that ruled that Cheong Seng Gee failed to sufficiently establish the Chinese marriage through a mere letter testifying that Cheong Boo and Tan Bit married each other but that because Cheong Seng Gee had been admitted to the Philippine Islands as the son of the decea sed, he should shar e in the estat e as a natural child. With reference to the allegations of Mora Adong and her daughters, the trial court reached the conclusion that the marriage between Adong and Cheo ng Boo had been adequately proved but that under the laws of the Philippine Islands it could not be held to be a lawful marriage and thus the daughter Payang and Rosalia would inherit as natural children. The lower court believes that Mohammedan marriages are not valid under the Philippine Island’s laws this as an Imam as a solemnizing officer and under Quaranic laws. ISSUES: Whether or not the Chinese marriage between Cheong Boo and Tan Dit is valid ?

Whether or not the Mohammedan marriage between Cheong Boo and Mora Adong is valid? RULING: The Supreme Court found the (1) Chinese marriage not proved and Chinaman Cheong Seng Gee has only the rights of a natural child while (2) it found the Mohammedan marriage to be proved and to be valid , thus giving to the widow Mora Adong and the legitimate children Payang and Rosalia the rights accruing to them under the law. (FOR STATCON) The Supreme Court held that marriage in this jurisdiction is not only a civil contract but it is a new relation, an instruction in the maintenance of which the public is deeply interested. The presumption as to marriage is that every intendment of the law leans toward legalizing matrimony. Persons

Page 36

dwelling together in apparent matrimony are presumed, in the absence of counterpresumption or evidence special to the case, to be in fact married. The reason is that such is the common order of society, and if the parties were not what they thus hold themselves out as being, they would be living in the constant violation of decency of the law. As to retroactive force, marriage laws is in the nature of a curative provision intended to safeguard society by legalizing prior marriages. Public policy should aid acts intended to validate marriages and should retard acts intended to invalidate marriages. This as for public policy, the courts can properly incline the scale s of their decision in favor of that solution which will most effectively promote the public policy. That is the true construction which will best carry legislative intention into effect. (FOR PERSONS) Sec. IV of the Marriage law provides that “all marriages contracted outside the islands, which would be valid by the laws of the country in which the same were contracted, are valid in these islands. To establish a valid foreign marriage pursuant to this comity provision, it is first necessary to prove before the courts ofthe Islands the existence of the foreign law as a question of fact, and it is then necessary to prove the alleged foreign marriage by convincing evidence. A Philippine marriage followed by 23 years of uninterrupted marital life, should not be impugned and discredited, after the death of the husband through an alleged prior Chinese marriage, “save upon proof so clear, strong and unequivocal as to produce a moral conviction of the existence of such impediment.” A marriage alleged to have been contracted in China and proven mainly by a socalled matrimonial letter held not to be valid in the Philippines.

CHING HUAT VS. CO HEONG Facts: It is alleged in the petition, that the said minor is his legitimate daughter ; that up to June 21, 1946, said minor had been living with and had under the custody of petitioner; that respondent, taking advantage of his confidential and spiritual relation with Maria Ching as her godfather , persuaded and induced her by means of trick, promises and cajolery, to leave the parental home and to elope with him in the night of June 21, 1946, to Plaridel, Bulacan, where they were married on the following day before the Justi ce of the Peace of said municipality, said Maria Ching being at the time 15 years old; and that ever since respondent has had the minor Maria Ching under his custody in Malolos, Bulacan, and has restrained her at her liberty. It is further alleged that respondent had been previously married in China to Gue Min, said marriage being said to be subsisting at the time respondent married Maria Ching. Petitioner further avers that Gue Min has never

Private International Law

been declared an absentee nor generally considered as dead and believed to be so by respondent at the time he married Maria Ching. Respondent, in his answer, among other things, asserts that on June 21, 1946, he and Maria Ching alias Avelina Ching were legally married before the Justice of the Peace of Plaridel, Bulacan, and alleges that the essential requisites for such marriage were complied with. Issue: whether or not petitioner still retains his

right to the custody of his minor daughter Maria Ching alias Avelina Ching? Ruling: the Philippine marriage between said respondent and Maria Ching before the Justice of the Peace of Plaridel, Bulacan, is undisputed. It is also bey ond question that marriage was contracted by a man much over 16 years old with a girl 15 years old (Act No. 3613, section 2), neither of whom was included in any of the exceptions mentioned in section 28 of the same Act; nor in those stated in section 29 thereof for the reason that the alleged prior Chinese marriage has not been established.

If the supposed prior Chinese marriage had been sufficiently proven, then in order that the subsequent Philippine marriage could be valid, it would have been necessary either ( a) that the Chinese marriage should have been previously annulled or dissolved: or ( b) that the first wife of respondent should have been absent for 7 consecutive years at the time of the second marriage without the respondent having news of the absentee being alive; or (c) that the absentee should have been generally considered as dead and believed to be so by respondent at the time of contracting the subsequent marriage, in either of which last two cases the subsequent marriage will be valid until declared null and void by a competent court, while in the first it will be valid without this limitation. (Act No. 3613, section 29 [ a] and [ b]; section 30 [ b].) But as already adverted to, the complete absence of proof of the supposed former Chinese marriage makes sections 29 and 30 of the Marriage Law inapplicable. Maria Ching having been validly married on June 21 , 1946, she became emancipated on that same date (arts. 314 [1] and 315, Civil Code). This emancipation brought about the loss by the fath er of the parental authority that he claims. On the other hand, by article 48 of Chapter V of the Spanish Marriage Law of 1870, whose articles 44 to 78 were, and are now partly, in force in the Philippines (Benedicto vs. De la Rama, 3 Phil., 34), the wife has the duty, among others, of living in her husband's company and of following him to wherever he transfer his domicile or residence. (Yañez de Barbuevo vs. Fuster, 29 Phil., 606, 612.)

Page 37

PEOPLE V. DUMPO 62 Phil 247

Facts: Moro Hassan and Mora Dupo have been legally married according to the rites and practice of the Mohammedan religion.Without this marriage being dissolved, it is alleged that Dumpo contracted another marriage with Moro Sabdapal after which the two lived together as husband and wife . Dumpo was prosecuted for and convicted of the crime of bigamy.

The accused interposed an appeal. It has been established by the defense, without the prosecution having presented any objection nor evidence to the contrary, that the alleged second marriage of the accused is null and void according to Mohammedan rites on the ground that her father had not given his consent thereto.

witnesses. Jan Peng, a Chinaman of 52 years of age, swore that he knew the forms of ceremonies of marriage in China, Dee Tim also presented a witness, Ty Cong Ting, a Chinaman, 32 years of age and a lawyer, who testified concerning the laws and customs in China with reference to the forms of marriage ceremony. He testified that he knew and was well acquainted with the customs and practices of Chinamen in China with reference to marriages and the manner and form in which they were celebr ated, and the form of proof issued for the purpose of proving that a marriage ceremony had been performed. Mr. Ty Cong Ting was, at the time he testified as a witness, the legal attorney of the Chinese Consul General in the City of Manila. Issue: Who is the legal wife? Held:

Held: Marriage among Moslems is a fact of which no judicial notice may be taken and must be subject to proof in every particular case. In the case at bar we have the uncontradicted testimony of Tahari, an Imam or Mohammedan priest authorized to solemnize marriages between Mohammedans, to the effe ct that the consent of the brid e's father or in the absence thereof, that of the chief of the tribe to which she belongs is an indispensable requisite for the validity of such contracts. It is an essential element of the crime of bigamy that the alleged second marriage, having all the essential requisit es, would be valid were it not for the subsistence of the first marriage. However, accused’s subsequent marriage was void for lack of requisites necessary under Moslem law, she must be acquitted.

LAO AND LAO V. DEE TIM 45 Phil 739 (1924)

It is perhaps true that Yap Siong did on various occasions, depending upon his interest and convenience at the particular time, state that Maria Lao was his querida and not his wife. It is also perhaps true, for the same reason, that he stated that Dee Tim was not his wife but his querida. Evidently he was attempting to keep the information, which he was quite able to do, until he had passed to that bourn from which none returns, and until a distribution of his large accumulated earnings among his heirs became necessary. Based on a preponderance of the evidence the Court was convinced that both Dee Tim and Maria Lao were legally married to Yap Siong in good faith, believing that each was his sole and separate wife, living in absolute ignorance of the fact of his double marriage. They were each married in good faith and in ignorance of the existence of the other marriage. Yap Siong up to the time of his death seems to have been successful in keeping each of his two wive s ignorant of the fact that he was married to the other.

Facts: Yap

Siong

died

intestate.

During

the

distribution of his estate, Maria Lao and Jose Lao appeared claiming to be the legitimate spouse and son of the deceased. Maria claims that they had been married in the Philippines on June 24, 1903. On the other hand, Dee Tim claims to be the legitimate widow of Yap Siong; that she and Yap Sion g were joined in holy matrimony on the 14th day of September, 1893, in accordance with the laws of China.

Maria Lao presented marriage certificates as proof. Dee Tim likewise presented a certificate of marriage and that it was positive proof of her marriage and that it complied with the custom and practice in China with reference to marriage ceremonies. To support her contention she presented a number of

Private International Law

Under the Leyes de Partidas (Law 1, title 13, partida 4), where two women innocently and in good faith are legal ly united in holy matrimony to the same man, their children born will be regarded as legitimate children and each family will be entitled to one-half of the estate of the husband upon distribution of his estate. That provision of the Leyes de Partidas is a very humane and wise law. It justly protects those who innocently have entered into the solemn relation of marriage and their descendants. The good faith of all the parties will be presumed until the contrary is positively proved. (Article 69, Civil Code; Las Leyes de Matrimonio, section 96; Gaines vs. Hennen, 65 U.S., 553.)c YAO KEE V. GONZALES 167 SCRA 736

Page 38

FACTS:

1.

2.

3. 4.

5.

6.

7.

Sy Ki at, a Chinese national, died in Calooocan City where he was then residing leaving behind real and personal properties here in the Philippines. Private respondents (Aida Sy-Gonzales et al.,) filed a petition for the grant of letters or administration alleging that they were the children of the deceased with Asuncion Petition was Gillego. opposed by herein petitioners (Yao Kee et al.,) alleging that they were the legitimate family. The probate cou rt found that Sy Kia t was lega lly marr ied to Yao Kee and that their 3 offsprings were the legitimate children. The court likewise ruled that respondents are the acknowledged illegitimate offspring of Sy Kiat with Asuncion Gillego. On appeal, the low er cou rt’s dec ision was set aside declaring petitioners as the acknowledge natural children of Sy Kiat and Asuncion Gillego. Oppostiors were declared the acknowelged natural children of the deceased since the legality of the alleged marriage of Sy Kiat and Yao Kee in China had not been proven to be valid to the laws of China.

ISSUE: •





custom on marriage not only because they are self serving evidence, but more importantly, there is no showing that they are competent to testify on the subject matter. For failure to prove the foreign law or custom, and consequently, the validity of the marriage in accordance with said law or custom, the marriage between Yao Kee and Sy Kiat cannot be recognized in this jurisdiction. However, as petitioners failed to establish the marriage of Yao Kee with Sy Kiat according to the laws of China, they cannot be accorded the status of legitimate children but only that of acknowledged natural children. Petitioners are natural children, it appearing that at the time of their conception Yao Kee and Sy Kiat were not disqualified by any impediment to marry one another. [See Art. 269, Civil Code] And they are acknowledged children of the deceased because of Sy Kiat’s recognition of Sze Sook Wah and its extension to Sze Lai Cho and Sy Chun Yen who are her sisters of the full blood. Private respondents on the other hand are also the deceased’s acknowledged natural children with Asuncion Gillego , a Filipina with whom he lived for 25 years without the benefit of marriage. They have in their favor their father’s acknowledgment, evidence by a compromise agreement entered into by and between their parents and approved by the CFI wherein Sy Kiat not only acknowledged them as his children by Asuncion Gillego but likewise made provisions for their support and future inheritance.

Was the fact of marriage of Sy Kiat and Yao Kee in China proven as a custom?

REPUBLIC V. ORBECIDO GR NO. 154380, October 5, 2005

HELD: •





Custom is defined as “a rule of conduct formed by repetition of acts, uniformly observed (practiced) as a social rule, legally binding and obligatory.” The law requires that “a custom must be proved as a fact, according to the rules of evidence. [Article 12, Civil Code] On this score the Court had occasion to state that “a local custom as a source of right cannot be considered by a court of justice unless such custom is properly established by competent evidence like any other fact. The same evidence, if not one of a higher degree, should be required of a foreign custom.

Construing this provision of law the Court has held that to establish a valid foreign marriage two things must be proven, namely 1) the existence of the foreign law as a question of fact; and 2) the alleged foreign marriage by convincing evidence. In the case at bar petitioners did not present any competent evidence relative to the law and custom of China on marriage. The testimonies of Yao and Gan Ching (brother) cannot be considered as proof of China’s law or

Private International Law

Facts: On May 24, 1981, Cipriano Orbecido III married Lady Myros M. Villanueva in the Philippines in Lam-an, Ozamis City. Their marriage was blessed with a son and a daughter. In 1986, Cipriano’s wife left for the United States bringing along their son Kristoffer. A few years later, Cipriano discovered that his wife had been naturalized as an American citizen.

Sometime in 2000, Cipriano learned from his son that his wife had obtained a divorce decree and then married a certain Innocent Stanley. She, Stanley and her child by him currently live in San Gabriel, California. Cipriano thereafter filed with the trial court a petition for authority to remarry invoking Paragraph 2 of Article 26 of the Family Code. No opposition was filed. Finding merit in the petition, the court granted the same. The Republic, herein petitioner, through the Office of the Solicitor General (OSG), sought reconsideration but it was denied.

Page 39

The OSG contends that Paragraph 2 of Article 26 of the Family Code is not applicable to the instant case because it only applies to a valid mixed marriage; that is, a marriage celebrated between a Filipino citizen and an alien. Furthermore, the OSG argues there is no law that governs respondent’s situation. The OSG posits that this is a matter of legislation and not of judicial determination.

and compelling reasons for the exemption. However, such exemption shall not apply if the same is not compatible with the solidarity of the family. (110a) Art. 70.

The spouses are jointly responsible for the support of the family. The expenses for such support and other conjugal obligations shall be paid from the community

Held: Taking into consideration the legislative intent and applying the rule of reason, we hold that Paragraph 2 of Article 26 should be interpreted to include cases involving parties who, at the time of the celebration of the marriage were Filipino citizens, but later on, one of them becomes naturalized as a foreign citizen and obtains a divorce decree. The Filipino spouse should likewise be allowed to remarry as if the other party were a foreigner at the time of the solemnization of the marriage . To rule otherwise would be to sanction absurdity and injustice. Where the interpretation of a statute according to its exact and literal import would lead to mischievous results or contravene the clear purpose of the legislature, it should be construed according to its spirit and reason, disregarding as far as necessary the letter of the law. A statute may therefore be extended to cases not within the literal meaning of its terms, so long as they come

property and, in absence thereof, from thethe income or fruits of their separate properties. In case of insufficiency or absence of said income or fruits, such obligations shall be satisfied from the separate properties. (111a) Art. 73.

Either spouse may exercise any legitimate profession, occupation, business or activity without the consent of the other. The latter may object only on valid, serious, and moral grounds. In case of disagreement, the court shall decide whether or not:

within its spirit or intent.

1.

In view of the fore going, we state the twin elements for the application of Paragraph 2 of Article 26 as follows:

2.

1. There is a vali d mar riage tha t has been celebrated between a Filipino citizen and a foreigner; and 2. A vali d div orce is obt ained ab road by the alien spouse capacitating him or her to remarry.

The reckoning point is not the citizenship of the parties at the time of the celebration of the marriage, but their citizenship at the time a valid divorce is obtained abroad by the alien spouse capacitating the latter to remarry. VI. CONSEQUENCES OF MARRIAGE

1.

The objection is proper; and Benefit has occurred to the family prior to the objection or thereafter. If the benefit accrued prior to the objection, the resulting obligation shall be enforced against the separate property of the spouse who has not obtained consent.

The foregoing provisions shall not prejudice the rights of creditors who acted in good faith. (117a) Muslim Code

Art. 34.

Mutual rights and obligations. —

PERSONAL RELATIONS

1. The husband and the wife Civil Code Art. 15, supra Family Code Art. 69.

The husband and wife shall fix the family domicile. In case of disagreement, the court shall decide. The court may exempt one spouse from living with the other if the latter shou ld live abroad or there are other valid

Private International Law

are obliged to live together, observe mutual respect and fidelity, and render mutual help and support in accordance with this Code. 2. When one of the spou ses neglects his or her duties to the conjugal union or brings danger, dishonor or material injury upon the other, the injured party may petition the court for relief. The court may

Page 40

counsel the offender to comply with his or her duties, and take such measures as may be proper. 3. The husband and the wife shall inherit from each other in accordance with this Code. 4. The husband and the wife shall have the right to divorce in accordance with this Code. Art. 35.

6. The wife shall be entitled to an equal and just treatment by the husband.

2. PROPERTY RELATIONS Civil Code: Art. 15, supra Art. 117.

engage in business. the husband may However, object, provided:

Rights and obligations of the husband. — The husband shall fix the residence of the family. The court may exempt the wife from living with her husband on any of the following grounds:

1.

His income is sufficient for the family, according to its social standing, and 2.His opposition is founded on serious and valid grounds.

a. Her

dower is not satisfied in accordance with the stipulations; or b. The conjugal dwelling is not in keeping with her social standing or is, for any reason, not safe for the members of the famil y or her property.

In case of disagreement on this question, the parents and grandparents as well as the family council, if any, shall be consulted. If no agreement is still arrived at, the court will decide whatever may be proper and in the best interest of the family. (n)

Art. 36. Rights and obligations of the wife. — 1. The wife shall dutifully manage the affairs of the household. She may purchase things necessary for the maintenance of the family, and the husband shall be bound to reimburse the expenses, if he has not delivered the proper sum. 2. The wife cannot, without the husband's consent, acquire any property by gratuitous title, except from her relatives who are within the prohibited degrees in marriage. 3. The wife may, with her husband's consent, exercise any profession or occupation or engage in lawful business which is in keeping with Islamic modesty and virtue. However,

if the husband refuses to give his consent on the ground that his income is sufficient for the family according to its social standing or his opposition is based on serious and valid grounds, the matter shall be referred to the Agama Arbitration Council. 4. The wife shall have the right to demand the satisfaction of her mahr. 5. Unless otherwise stipulated in the marriage settlements, the wife retain ownership and administration of her exclusive property.

Private International Law

The wife may exercise any profession or occupation or

Art. 118.

The property relations between husband and wife shall be governed in the following order: a. By contract executed before the marriage; b. By the provisions of this Code; and c. By custom. (1315a)

Art. 124.

If the marriage is between a citizen of the Philippines and a foreigner, whether celebrated in the Philippines or abroad, the following rules shall prevail:

1. If the husband is a citizen of the Philippines while the wife is a of foreigner, the provisions this Code shall govern their relations; 2. If the husband is a foreigner and the wife is a citizen of the Philippines, the laws of the husband's country shall be followed, without prejudice to the provisions of this Code with regard to immovable property. (1325a) Art. 144.

When a man and a woman live together as husband and wife, but they are not married, or their marriage is void from the beginning, the property

Page 41

acquired by either or both of them through their work or industry or their wages and salaries shall be governed by the rules on co-ownership. (n)

2.

Family Code:

3. Art. 74.

A marriage in articulo mortis may also be solemnized by the captain of a ship or chief of an airplane during a voyage, or by the commanding officer of a

4.

military unit, in the absence of a chaplain, during war. The duties mentioned in the two preceding articles shall be complied with by the ship captain, airplane chief or commanding officer. (n) Art. 75.

Art. 76.

Art. 77.

Marriages between Filipino citizens abroad may be solemnized by consuls and vice-consuls of the Republic of the Philippines. The duties of the local civil registrar and of a judge or justice of the peace or mayor with regard to the celebration of marriage shall be performed by such consuls and vice-consuls. (n) No marriage license shall be necessary when a man and a woman who have attained the age of majority and who, being unmarried, have lived together as husband and wife for at least five years, desire to marry each other. The contracting parties shall state the foregoing facts in an affidavit before any person authorized by law to administer oaths. The official, priest or minister who solemnized the marriage shall also state in an affidavit that he took steps to ascertain the ages and other qualifications of the contracting parties and that he found no legal impediment to the marriage. (n) In case two with persons accordance law married desire toin ratify their union in conformity with the regulations, rites, or practices of any church, sect, or religion it shall no longer be necessary to comply with the requirements of Chapter 1 of this Title and any ratification made shall merely be considered as a purely religious ceremony. (23)

Art. 80. The following marriages shall be void from the beginning: 1. Those contracted under the ages of sixteen and fourteen years by the male

Private International Law

and female respectively, even with the cons ent of the parents; Those solemnized by any person not legally authorized to perform marriages; Those solemnized without a marriage license, save marriages of exceptional character; Bigamous or polygamous marriages not falling under Article 83, Number 2; marriages mentioned in Article 81; Those where one or both contracting parties have been found guilty of the killing of the spouse of either of them; Those between stepbrothers and stepsisters and other marriages specified in Article 82. (n)

5. Incestuous 6.

7.

Art. 147.

When a man and a woman who are capacitated to marry each other, live exclusively with each other as husband and wife without the benefit of marriage or under a void marriage, theirbe wages salaries shall owned and by them in equal shares and the property acquired by both of them through their work or industry shall be governed by the rules on co-ownership. In the absence of proof to the contrary, properties acquired while they lived together shall be presumed to have been obtained by their joint efforts, work or industry, and shall be owned by them in equal shares. For purposes of this Article, a party who did not participate in the acquisition by the other party of any property shall be deemed to have contributed jointly in the acquisitionefforts thereof if the former's consisted in the care and maintenance of the family and of the household. Neither party can encumber or dispose by acts inter vivos of his or her share in the property acquired during cohabitation and owned in common, without the consent of the other, until after the termination of their cohabitation. When only one of the parties to a void marriage is in good faith, the share of the party in bad

Page 42

faith in the co-ownership shall be forfeited in favor of their common children. In case of default of or waiver by any or all of the common children or their descendants, each vacant share shall belong to the respective surviving descendants. In the absence of descendants, such share shall belong to the innocent party. In all cases, the forfeiture shall take place upon termination of the cohabitation. (144a)

contract, shall be governed by the regime of complete separation of property in accordance with this Code and, in a suppletory manner, by the general principles of Islamic law and the Civil Code of the Philippines. VII. Dissolution of Marriage

1.

Annulment

Civil Code: Art. 148.

In cases of cohabitation not falling under the preceding Article, only the properties acquired by both of the parties through their actual joint contribution of money, property, or industry shall be owned by them in common in proportion to their respective contributions. In the absence of proof to the contrary, their contributions and corresponding shares are presumed to be equal. The same rule and presumption shall apply to joint deposits of money and evidences of credit.

Art. 66.

When either or both of the contracting parties are citizens or subjects of a foreign country, it shall be necessary, before a marriage license can be obtained, to provide themselves with a certificate of legal capacity to contract marriage, to be issued by their respective diplomatic or consular officials. (13a)

Art. 71.

All marriages performed outside the Philippines in accordance with the laws in force in the country where they were performed, and valid there as such, shall also be

If one of the parties is validly married his or shall her share in to theanother, co-ownership accrue to the absolute community or conjugal partnership existing in such valid marriage. If the party who acted in bad faith is not validly married to another, his or her shall be forfeited in the manner provided in the last paragraph of the preceding Article. The foregoing rules on forfeiture shall likewise apply even if both parties are in bad faith. (144a)

valid in this polygamous, country, except bigamous, or incestuous marriages as determined by Philippine law. (19a) Family Code Art. 45.

A marriage may be annulled for any of the following causes, existing at the time of the marriage: 1.

Muslim Code: Art. 37.

How governed. — The property relations husband wife shallbetween be governed in and the following order:

a. By contract before or at the b. c.

time of the celebration of marriage; By the provisions of this Code; and By custom. 2.

Art. 38.

Regime of property relations. — The property relations between the spouses, in the absence of any stipulation to the contrary in the marriage settlements or any other

Private International Law

3.

That the party in whose behalf it is sought to have the marriage annulled was eighteen years of age or over but below twenty-one, and the marriage was solemnized without the consent or of person the parents, guardian having substitute parental authority over the party, in that order, unless after attaining the age of twenty-one, such party freely cohabited with the other and both lived together as husband and wife; That either party was of unsound mind, unless such party after coming to reason, freely cohabited with the other as husband and wife; That th e con sent of e ither party was obtained by

Page 43

4.

5.

6.

Art. 46.

fraud, unless such party afterwards, with full knowledge of the facts constituting the fraud, freely cohabited with the other as husband and wife; That th e con sent of e ither party was obtained by force, intimidation or undue influence, unless the same having disappeared or ceased, such party thereafter freely cohabited with the other as husband and wife; That either party was physically incapable of consummating the marriage with the other, and such incapacity continues and appears to be incurable; or That either party was afflicted with a sexuallytransmissible disease found to be serious and appears to be incurable. (85a)

3.

4.

Any of the following circumstances shall constitute fraud referred to in Number 3 of the preceding Article: 1.

2.

3.

4.

Non-disclosure of a previous conviction by final judgment of the other party of a crime involving moral turpitude; Concealment by the wife of the fact that at the time of the marriage, she was pregnant by a man othe r than her husband; Concealment of sexually transmissible disease, regardless of its nature, existing at the time of the marriage; or Concealment of drug addiction, habitual alcoholism or homosexuality or lesbianism existing at the time of the marriage.

No other misrepresentation or deceit as to character, health, rank, fortune or chastity shall constitute such fraud as will give grounds for action for the annulment of marriage. (86a) Art. 47.

2.

The action for annulment of marriage must be filed by the following persons and within the periods indicated herein: 1.

For causes mentioned in number 1 of Article 45 by the party whose parent or guardian did not give his or her consent, within five years after attaining the

Private International Law

5.

Art. 48.

age of twenty-one, or by the parent or guardian or person having legal charge of the minor, at any time before such party has reached the age of twentyone; For ca uses mentioned in number 2 of Article 45, by the same spouse, who had no knowledge of the other's insanity; or by any relative or guardian or person having legal charge of the insane, at any time before the death of either party, or by the insane spouse during a lucid interval or after regaining sanity; For ca uses mentioned in number 3 of Article 45, by the injured party, within five years after the discovery of the fraud; For ca uses mentioned in number 4 of Article 45, by the injured party, within five years from the time the force, intimidation or undue influence disappeared or ceased; For ca uses mentioned in number 5 and 6 of Article 45, by the injured party, within five years after the marriage. (87a)

In all cases of annulment or declaration of absolute nullity of marriage, the Court shall order the prosecuting attorney or fiscal assigned to it to appear on behalf of the State to take steps to prevent collusion between the parties and to take care that evidence is not fabricated or suppressed. In the cases referred to in the preceding paragraph, no judgment shall be based upon a stipulation of facts or confession of judgment. (88a)

Art. 49.

During the pendency of the action and in the abse nce of adequate provisions in a written agreement between the spouses, the Court shall provide for the support of the spouses and the custody and support of their common children. The Court shall give paramount consideration to the moral and material welfare of said children and their choice of the parent with whom they wish to remain as provided to in Title IX. It shall also provide for appropriate visitation rights of the other parent. (n)

Page 44

Art. 50.

The effects provided for by paragraphs (2), (3), (4) and (5) of Article 43 and by Article 44 shall also apply in the proper cases to marriages which are declared ab initio or annulled by final judgment under Articles 40 and 45.

the appropriate civil registry and registries of property; otherwise, the same shall not affect third persons. (n) Art. 53.

Either of the former spouses may marry again after compliance with the requirements of the immediately preceding Article; otherwise, the subsequent marriage shall be null and void.chan robles virtual law

Art. 54.

Children conceived or born before the judgment of annulment or absolute nullity of the marriage under Article 36 has become final and executory shall be considered legitimate. Children conceived or born of the subsequent marriage under Article 53 shall likewise be legitimate.

The final judgment in such cases shall provide for the liquidation, partition and distribution of the properties of the spouses, and support of the thecustody common children, and the delivery of third presumptive legitimes, unless such matters had been adjudicated in previous judicial proceedings. All creditors of the spouses as well as of the absolute community or the conjugal partnership shall be notified of the proceedings for liquidation. In the partition, the conjugal dwelling and the lot on which it is situated, shall be adjudicated in accordance with the provisions of Articles 102 and 129. Art. 51.

In said partition, the value of the presumptive legitimes of all common children, computed as of the date of the final judgment of the trial court, shall be delivered in cash, property or sound securities, unless the parties, by mutual agreement judicially approved, had already provided for such matters.

library

Art. 55. A petition for legal separation may be filed on any of the following grounds: 1.

2.

3.

The children or their guardian or the trustee of their property may ask for the enforcement of the judgment. 4. The delivery of the presumptive legitimes herein prescribed shall in no way prejudice the ultimate successional rights of the children accruing upon the death of either of both of the parents; but the valu e of the properties already received under the decree of annulment or absolute nullity shall be considered as advances on their legitime. (n) Art. 52.

The judgment of annulment or of absolute nullity of the marriage, the partition and distribution of the properties of the spouses and the delivery of the children's presumptive legitimes shall be recorded in

Private International Law

5. 6. 7.

8.

Repeated physical violence or grossly abusive conduct directed against the petitioner, a common child, or a child of the petitioner; Physical violence or moral pressure to compel the petitioner to change religious or political affiliation; Attempt of re spondent to corrupt or induce the petitioner, a common child, or a child of the petitioner, to engage in prostitution, or connivance in such corruption or inducement; Final judgment sentencing the respondent to imprisonment of more than six years, even if pardoned; Drug addiction or habitual alcoholism of the respondent; Lesbianism or homosexuality of the respondent; Contracting by the respondent of a subsequent bigamous marriage, whether in the Philippines or abroad; Sexual infidelity or perversion;

Page 45

9.

Attempt by the respondent against the life of the petitioner; or 10. Abandonment of petitioner by respondent without ustifiable j cause for more than one year. For purposes of this Article, the term "child" shall include a child by nature or by adoption. (9a) 2.

Absolute Divorce

ROEHR VS. RODRIGUEZ

Facts:

Petitioner Wolfgang Roehr, a German citizen, married a Filipina, Carmen Rodriguez in Germany. The marriage was ratified in Tayasan, Negros Oriental. Private respondent filed a petition for the declaration of nullity of marriage before the RTC of Makati. Petitioner filed a motion to dismiss but was denied by the trial court. The petitioner obtained a decree of divorce from the Court of First Instance of Hamburg Blankenese and granting the custody of the children to the father.

Civil Code:

Issue:

Art. 15, supra Art. 17, Supra

Whether or not the legal effects of a divorce obtained from a foreign country such as support and cust ody of the children can be determined in our courts?

Muslim Code:

Held:

45-55 supra

Yes. In order to take effect, a foreign judgement must clearly show that the opposing party has been given ample opportunity to do so under the Rules of Civil Procedure. Accordingly, the respondent was not given the opportunity to challenge the judgement of the German Court, therefore, legal effects of divorce must be determined in our courts. The court held that the trial court has jurisdiction over the issue between the parties as to who has the parental custody.

Art. 27. By a husband. — Notwithstanding the rule of Islamic law permitting a Muslim to have more than one wife but one wife unless he can deal with them with equal companionship and just treatment enjoined Islamic lawas and only exceptional cases. Art. 34.

by in

Mutual rights and obligations. —

RECTO VS. HARDEN (supra)

1. The husband and the wife

2.

3.

4.

are obliged to live together, observe mutual respect and fidelity, and render mutual help and support in accordance with this Code. When one of the spou ses neglects his or her duties to the conjugal union or brings danger, dishonor or material injury upon the other, the injured party may petition the court for relief. The court may counsel the offender to comply with his or her duties, and take such measures as may be proper. .chan robles virtual law library The husband and the wife shall inherit from each other in accordance with this Code. The husband and the wife shall have the right to divorce in accordance with this Code.

Private International Law

GONZALES VS. GONZALES

Facts:

Plaintiff and defendant are citizens of the Philippine Islands and at present residents of the City of Manila. They were married in the City of Manila on January 19, 1919, and lived together as man and wife in the Philippine Islands until the spring of 1926. They voluntarily separated since time not lived together asand man andthat wife. Of have this union four children were born. Negotiations between the parties, both being represented by attorneys, whereupon it was mutually agreed to allow the plaintiff for her support and that of her children, five hundred pesos (P500) monthly; this amount to be increased in case of illness or necessity, and the title of certain properties to be put in her name. Shortly after this agreement the husband left the Islands, betook himself to Reno, Nevada, and secured in that jurisdiction an absolute divorce on the ground of desertion. Shortly thereafter the defendant moved to California and returned to these Islands in August 1928, where he has since remained. On the same date that he secured a divorce in Nevada he went through the forms of marriage with another citizen of

Page 46

these Islands and now has three children as a result of that marriage. Defendant, after his departure from these Islands, reduced the amount he had agreed to pay monthly for the support of his wife and four minor children and has not made the payments fixed in the Reno divorce as alimony. Issue:

Whether or not that the divorced acquired in Nevada is valid here in the Philippines? Held

While the parties in this action are in dispute over financial matters they are in unity in trying to secure the courts of this jurisdiction to recognize and approve of the Reno divorce. On the record here presented this can not be done. The public policy in this jurisdiction on the question of divorce is clearly set forth in Act No. 2710, and the decisions of this court.

jurisdiction, except it be for a cause, and under conditions for which the courts of Philippine Islands would grant a divorce. The lower court in granting relief as prayed for frankly stated that the securing of the divorce, the contracting of another marriage and the bringing into the world of innocent children brings about such a condition that the court must grant relief. The hardships of the existing divorce laws of the Philippine Islands are well known to the members of the Legislature. It is of no moment in this litigation what he personal views of the writer on the subject of divorce may be. It is the duty of the courts to enforce the laws of divorce as written by the Legislature if they are constitutional. Courts have no right to say that such laws are too strict or too liberal. TENCHAVEZ V. ESCANO 15 SCRA 355

Facts: •

The entire conduct of the parties from the time of their separation until the case was submitted to this court, in which they all prayed that the Reno divorce be ratified and confirmed, clearly indicates a purpose to circumvent the laws of the Philippine Islands regarding divorce and to secure for themselves a change of status for reasons and under conditions not authorized by our law. At all times the matrimonial domicile of this couple has been within the Philippine Islands and the residence acquired in the State of Nevada by the husband of the purpose of securing a divorce was not a bona fide residence and did not confer jurisdiction upon the Court of that State to dissolve the bonds if matrimony in which he had entered in 1919. While the decisions of this court heretofore in refusing to recognize the validity of foreign divorce has usually been expressed in the negative and have been based upon lack of matrimonial domicile or fraud or collusion, we have not overlooked the provisions of the Civil Code now in force in these Islands. Article 9 thereof reads as follows: The laws relating to family rights and duties, or to the status, condition and legal capacity or persons, are binding upon Spaniards though they reside in a foreigneven country.

And article 11, the last part of which reads: . . . the prohibitive laws concerning persons, their acts and their property, and those intended to promote public order and good morals, shall nor be rendered without effect by any foreign laws or judgments or by anything done or any agreements entered into a foreign country. It is therefore a serious question whether any foreign divorce relating to citizens of the Philippine Islands, will be recognized in this

Private International Law



Pastor Tenchavez married Vicenta Escano on Feb. 24, 1948 in Cebu City. As of June 1948, the newly-weds were already estranged. Vicenta left for the US and filed a verified complaint for divorce against the Pastor in the State of Nevada on the ground of "extreme cruelty, entirely mental in character." A decree of divorce was issued by the Nevada Court. Later on, Vicenta married an American, Russell Leo Moran in Nevada. She now lives with him in California and has begotten children. She acquired American citizenship on August 8, 1958. On July 30, 1955, Pastor filed a complaint for legal separation and damages against Vicenta and her parents in the CFI-Cebu.

HELD: •

At the time the divor ce decree was issued, Vicenta, like her husband, was still a Filipino citizen. She was then subject to Philippine laws under Art. 15 of the New Civil Code. Philippine law, under the NCC then now in force, does not admit absolute divorce but only provides for legal separation.



For Philippine courts to recognize foreign divorce decrees between Filipino ci tize nsdeclared would policy be a of pate nt violation of the the State, especially in view of the 3rd par. of Art. 17, of the New Civil Code which reads: Prohibitive laws concerning persons, their acts or property, and those which have, for their object, public order, public policy and good customs shall not be rendered ineffective by laws or judgments promulgated, or by determinations or conventions agreed upon in a foreign country. Moreover, recognition would give rise to scandalous discrimination in favor of wealthy citizens to the detriment of those members of our society whose means do not permit them to sojourn abroad and obtain absolute divorce outside the Philippines.

Page 47



Therefore, a foreign divorce between Filipino citizens, sought and decreed after the effectivity of the NCC, is not entitled to recognition as valid in this jurisdiction.

enforced outside of the jurisdiction of the court. •

SIKAT V. CANSON 67 PHIL 207 •















Hilaria Sikat and John Canson contrac ted marriage and lived together as husband and wife until 1911 when they separated. During the same year the wife commenced divorce proceedings against her spouses, but the case was dismi ssed without the court passing upon the merits thereof. At the time of their marriage in 1904, John Canson was an Italian citizen but on February 27, 1922, he became a naturalized Filipino citizen. In 1929, he went to Reno, Nevada, United States of America, and on October 8, of that year, he obtained an absolute decree of divorce on the ground of desertion. Hilaria, in 1933, filed another action, civil case No. 5398 of the Court of First Instance of Rizal, wherein she sought to compel the defendant to pay her a monthly pension of P500 as alimony or support. Canson interposed three defenses: (1) adultery on the part of the plain tiff: (2) absolute divorce obtained by the defendant as decreed by the court in Reno, Nevada, United States of America; and (3) that the defendant did not have the means to pay the allowance sought. The lower court dismissed the complaint and declined to accord validity to the divorce obtained in Reno but found that Hilaria Sikat had forfeited her right to support because she had committed adultery. This judgment was not appealed and it became final. On June 1, 1934, the present action was instituted by the plaintiff-appellant to obtain the liquidation of the conjugal partnership. The action is predicated on the existence of a final decree of absolute divorce rendered by the court of Reno, Nevada, since 1929.

Held: •

It is not, however, the citizenship of the plaintiff for divorce which confers jurisdiction upon a court, but his legal residence within the State (Cousins Hix vs. Fluemer, 55 Phil. 851 ). And assuming that John Canson acquired legal residence in the State of Nevada through the approval of his citizenship papers, this did not confer jurisdiction on the Nevada court to grant a divorce that would be valid in this jurisdiction nor jurisdiction that could determine their matrimonial status, because the wife was still domiciled in the Philippines. The Nevada court never acquired jurisdiction over her person. (Gorayeb vs. Hashim, 50 Phil. 26 , and Cousins Hix vs. Fluemer, supra.) This was not a proceeding in rem to justify a court in entering a decree as to the res or marriage relation entitled to be

Private International Law



territorial

Plaintiff-appellant had made her choice of two inconsistent remedies afforded her by law: (1) to impugn the divorce and file an action for support, or (2) uphol d the validity of the divorce and sue for a liquidation of conjugal partnership. She chose the first remedy when she filed her action for support. She lost the case and should take the consequences. The courts in the Philippines can grant a divorce only on the ground of "adultery on the part of the wife or concubinage on the part of the husband" as provided for under section 1 of Act No. 2710. The divorce decree in question was granted on the ground of desertion, clearly not a cause for divorce under our laws.





That our divorce law, Act No. 2710, is too strict or too liberal is not for this court to decide. (Barretto Gonzalez vs. Gonzalez, supra.) The allotment of powers between the different governmental agencies restricts the judiciary within the confines of interpretation, not of legislation. The legislative policy on the matter of divorce in this jurisdiction is clearly set forth in Act No. 2710 and has been upheld by this court; ARCA V. JAVIER 95 PHIL 579

Dissatisfied with the decision of the Court of First Instance of Cavite ordering him to give a monthly allowance of P60 to plaintiffs beginning March 31, 1953, and to pay them attorney's fees in the amount of P150 defendant took the case directly to this Court attributing five errors to the court below. The facts are not disputed. •

Javier and Arca got married in Manila. Javier, an enlisted US Navy personnel left for the State s 7 years after the birth of their first born. At such time, Arca lived with Javier’s parents. However, due to strained relations with the latter, she left and transferred to her hometown. Thereafter, Javier filed a case for divorce in Alabama against Arca alleging abandonment. Arca made her reply claiming among other things that she never abandoned her husband and that their separation was due to a physical impossibility which justifies her separation if the husband moves to ultra-marine colonies. The divorce however was granted. • Javier got married the 2nd time but was subsequently divorced. After a few years, he went back to the Philippines, and believing that the first two divorces were valid, remarried the 3rd time. •

Issue: Did the Circuit Court of Mobile County acquire jurisdiction of both spouses and effectively

Page 48

rendered a judgment in rem when it granted divorce to Javier? Held: •





No, one of the essential conditions for the validity of a decree of divorce is that the court must have jurisdiction over the subject matter and in order that this may be acquired, plaintiff must be domiciled in good faith in the State in which it is granted (Cousins Hix vs. Fluemer, 55 Phil., 851, 856). While it is true that Salud R. Arca filed an answer in the divorce case instituted at the Mobile County in view of the summo ns served upon her in this jurisdiction, but this action cannot be interpreted as placing her under the jurisdiction of the court because its only purpose was to impugn the claim of appellant that his domicile or legal residence at that time was Mobile County, and to show that the ground of desertion imputed to her was baseless and false. Such answer should be considered as a special appearance the purpose of which is to impugn the jurisdiction of the court over the case. It is established by the great weight of authority that the court of a country in which neither of the spouses is domiciled and to which one or both of them may resort merely for the purpose of obtaining a divorce has no jurisdiction to determine their matrimonial status; and a divorce granted by such a court is not entitled to recognition elsewhere. ( See Note to Succession of Benton, 59 L. R. A., 143) The voluntary appearance of the defendant before such a tribunal does not invest the court with jurisdiction. (Andrews vs. Andrews, 188 U. S., 14; 47 L. ed., 366.)





It follows that, to give a court jurisdiction on the ground of the plaintiff's residence in the State or country of the judicial forum, his residence must be bona fide . If a spouse leaves the family domicile and goes to another State for the sole purpose of obtaining a divorce, and with no intention of remaining, his residence there is not sufficient to confer jurisdiction on the courts of the State. (Ramirez vs. Gmur, 82 Phil., 855.) But even if his residence had been taken up is good faith, and the court had acquired jurisdiction to take cognizance of the divorce suit, the decree issued in his favor is not binding upon the appellant; for the matrimonial domicile of the spouses being the City of Manila;

Ramirez v. Gmur 42 Phil 855

Leona Castro was the natural daughter of decedent Samuel Bischoff. Whereas Ana Ramirez was the latter's widow to which they had no children. Leona was married to Kauffman. They had 3 children, Elena,

Private International Law

Federico, and Ernesto. Later, Kauffman brought Leona to Switzerland to recuperate her health. A few years later, she fell for a Dr. Mory to whom she had a child, Leontina. She informed Kauffman that she no longer wished to stay with him to which the latter obtained a divorce in France where Leona was in default. Leona, after the divorce got married in London, and after which bore two children from which the last childbirth caused Leona her life. The heirs of Leona from the first marriage and the second now claims the Estate of Samuel to which Ana Ramirez opposed since Leona is not a recognized natural child. Held: The status of Leona Castro as recognized natural daughter of Samuel Bischoff is fully and satisfactorily shown. With reference to the right of the von Kauffman children, it is enough to say that they are legitimate children, born to their parents in lawful wedlock; and they are therefore entitled to participate in the inheritance which would have devolved upon their mother, if she had survived the testator. The Court is of the opinion that the decree of divorce upon which reliance is placed by the representation of the Mory children cannot be recognized as valid in the courts of the Philippine Islands. The French tribunal has no jurisdiction to entertain an action for the dissolution of a marriage contracted in these Islands by person domiciled here, such marriage being indissoluble under the laws then prevailing in this country.

The evidence shows that the decree was entered against the defendant in default, for failure to answer, and there is nothing to show that she had acquired, or had attempted to acquire, a permanent domicile in the City of Paris. It is evident of course that the presence of both the spouses in that city was due merely to the mutual desire to procure a divorce from each other. It is established by the great weight of authority that the court of a country in which neither of the spouses is domiciled and to which one or both of them may resort merely for the purpose of obtaining a divorce has no jurisdiction to determine their matrimonial status; and a divorce granted by such a court is not entitled to recognition elsewhere. It follows that, to give a court jurisdiction on the ground of the plaintiff's residence in the State or country of the judicial forum, his residence must be bona fide. If a spouse leaves the family domicile and goes to another State for the sole purpose of obtaining a divorce, and with no intention of remaining, his residence there is not sufficient to confer juri sdiction on the courts of that State. This is especially true where the cause of divorce is one not recognized by the laws of the State of his own domicile.

Page 49

reach of execution to satisfy the judgment debt of Corominas. VAN DORN V. ROMILLO 139 SCRA 139

MANILA SURETY & FIDELITY VS. TEODORO FACTS:

Jose Corominas, Jr. and Sonia Lizares were married in Iloilo on January 5, 1935. On November 29,1954, a decree of divorce was granted by the Court of the State of Nevada dissolving the bonds of matrimony between

Facts: •

Sonia Lizares and Jose Corominas, Jr. . . . Trinidad Teodoro met Jose Corominas, Jr. in Hongkong on October 30, 1955. . . . On March 26, 1956, they went through a Buddhist wedding ceremony in Hongkong. Upon their return to the Philippines they took up residence in a rented house at No. 2305 Agno Street . . . Manila. On September 5, 1961, plaintiff and Jose Corominas, Jr. were married for a second time on Washoe County, Nevada. U.S.A. Additional Pertinent facts, also mentioned in the decision under review and controverted by the parties, are that Sonia Lizares is still living and that the conjugal partnership formed by her marriage to Corominas was dissolved by the Juvenile and Domestic Relations Court of Manila upon their joint petition, the decree of dissolution having been issued on October 21, 1957. Trinidad questioned the levy on the property since the property in question was her paraphernal property.







RULING: There is no doubt that the decree of divorce granted by the Court of Nevada in 1954 is not valid under Philippine law, which has outlawed divorce altogether ; that the matrimonial bonds between Jose Corominas, Jr. and Sonia Lizares have not been dissolved, although their conjugal partnership was terminated in 1957; and that the former's subsequent marriage in Hongkong to Trinidad Teodoro is bigamous and void.

In the present case, however, we find no need to pass on this question. The particular properties involved here which were admittedly acquired by respondent Teodoro, cannot be deemed to belong to such co-ownership because, as found by the trial court and confirmed by the Court of Appeals, the funds used in acquiring said properties were fruits of respondent's paraphernal investments which accrued before her "marriage" to Corominas. In other words they were not acquired by either or both of the partners in the void marriage through their work or industry or their wages and salaries, and hence cannot be the subject of coownership under Article 144. They remain respondent's exclusive properties, beyond the

Private International Law

in the Philippines and begot 2 children; Alicia filed for divorce in Nevada; the parties were divorced in Nevada, US, in 1982; and petitioner has remarried also in Nevada, this time to Theodore Van Dorn. On June 18, 1983 Upton filed a suit against petitioner in the RTC-Pasay, stating that petitioner's business in Ermita, Manila (the Galleon Shop), is conjugal property and asking that petitioner be ordered to render an accounting of that business, and that Upton be declared as having the right to manage the conjugal property.

Held:

ISSUE:

Whether or not the properties in question are conjugal?

Petitioner Alice Reyes Van Dorn is a citizen of the Philippines while private respondent Richard Upton is a US citizen; they were married in Hong Kong in 1972; after the marriage, they established their residence

Owing to embodied Philippine the policy

the nationality principle in Art. 13, NCC, only nationals are covered by against absolute divorce

the same being considered contrary to our concept of public policy and morality. However, aliens may obtain divorce abroad, which may be recognized in the Philippines provided they are valid according to their national law. In this case, the divorce in Nevada released private respondents from the marriage from the standards of American law, under which divorce dissolves the marriage.



Court said that “Ours is not only a court of law but also a court of equity.” The Court could not turn its back on its citizen when the foreign national itself benefited from such divorce decree;



Thus, pursuant to his national law, Upton is no longer the husband of petitioner. He would have no standing to sue in the case below as petitioner's



husband who assets. is entitled to exercise control over conjugal To maintain, as Upton does, that under our laws, petitioner has to be considered still married to him and still subject to a wife's obligations under the NCC cannot be just. Petitioner should not be obliged to live together with, observe respect and fidelity, and render support to private respondent. The latter should not continue to be one of her heirs w/ possible rights to conjugal properties. She should not be discriminated against in her own country if the ends of justice are to be observed.

PILAPIL V. IBAY-SOMERA

Page 50

174 SCRA 653 •









Petitioner Imelda Manalaysay Pilapil, a Filipino citizen, and private respondent Erich Ekkehard Geiling, a German national, were married in the Federal Republic of Germany. The marriage started auspiciously enough, and the couple lived together for some time in Malate, Manila where their only child, Isabella Pilapil Geiling, was born on April 20, 1980. Thereafter, marital discord set in, with mutual recriminations between the spouses, followed by a separation de facto between them. After about three and a half years of marriage, private respondent initiated a divorce proceeding against petitioner in Germany. He claimed that there was failure of their marriage and that they had been living apart since April, 1982. Petitioner, on the other hand, filed an action for lega l separation, support and separation of property before the Regional Trial Court of Manila. Thereafter a decree of divorce was promulgated. The records show that under German law said court was locally and internationally competent for the divorce proceeding and that the dissolution of said marriage was legally founded on and authorized by the applicable law of that foreign jurisdiction. More than five months after the issuance of the divorce decree , private respondent filed two complaints for adultery before the City Fiscal of Manila alleging that, while still married to said respondent, petitioner "had an affair with a certain William Chua as early as 1982 and with yet another man named Jesus Chua sometime in 1983".

Issue: WON the adultery case be sustained even though there has already been a finality of a divorce decree. Held: •

The law specifically provides that in prosecutions for adultery and concubinage the person who becan the complaint should the legally offendedfile spouse, and nobody else.



Corollary to such exclusive grant of power to the offended spouse to institute the action, it necessarily follows that such initiator must have the status, capacity or legal representation to do so at the time of the filing of the criminal action. This is a familiar and express rule in civil actions; in fact, lack of legal capacity to sue, as a ground for a motion to dismiss in civil cases, is determined as of the filing of the complaint or petition. In these cases, therefore, it is indispensable that the status and capacity of the complainant to commence the action be definitely

Private International Law

established and, as already demonstrated, such status or capacity must indubitably exist as of the time he initiates the action. •



In the present case, the fact that private respondent obtained a valid divorce in his country, the Federal Republic of Germany, it is deemed admitted. Said divorce and its legal effects may be recognized in the Philippines insofar as private respondent is concerned in view of the nationality principle in our civil law on the matter of status of persons. There fore, the priva te responden t, ioner being, no longer husband of petit had no legal standing to commence the adultery case under the imposture that he was the offen ded spou se at the time he filed suit.

LLORENTE V. COURT OF APPEALS GR No. 124371, November 23, 2000

FACTS:

Lorenzo Llorente and petitioner Paula Llorente were married in 1937 in the Philippines. Lorenzo was an enlisted serviceman of the US Navy. Soon after, he left fo r the US where through nat ura lizvisit ation, he wife, became a US Citize Upon his to his he discovered thatn. she was living with his brother and a child was born. The child was registered as illegitimate but the name of the father was left blank. Llorente filed a divorce in California in which Paula was represented by counsel, John Riley, and actively participated in the proceedings, which later on became final. He married Alicia and they lived together for 25 years bringing 3 children. He made his last will and testament stating that all his properties will be given to his second marriage. He filed a petition of probate that made or appointed Alicia his special administrator of his estate. Before the proceeding could be terminated, Lorenzo died. Paula filed a letter of administration over Llorente’s estate. The trial granted the letter and denied the motion for reconsideration. An appeal was made to the Court of Appeals, which affirmed and modified

the judgment of the Courtproperties, that she be declared co-owner of Trial whatever she and the deceased, may have acquired in their 25 years of cohabitation. ISSUE:

Whether or not national law shall apply? RULING: “Art. 15. Laws relating to family rights and duties, or to the status, condition and legal capacity of persons are binding upon citizens of the Philippines , even though living abroad.

Page 51

“Art. 16. Real property as well as personal property is subject to the law of the country where it is situated. First, there is no such thing as one American law. The "national law" indicated in Article 16 of the Civil Code cannot possibly apply to general American law. There is no such law governing the validity of testamentary provisions in the United States. Each State of the unio n has its own law applicable to its citizens and in forc e only within the State. It can therefore refer to no other than the law of the StateSecond, of which theis no decedent was a there showing that resident. the application of the renvoi doctrine is called for or required by New York State law. “However, intestate and testamentary succession, both with respect to the order of succession and to the amount of successional rights and to the intrinsic validity of testamentary provisions, shall be regulated by the national law of the person whose succession is under consideration , whatever may be the nature of the property and regardless of the country wherein said property may be found.” (emphasis ours)

Likewise, Lorenzo Llorente was already an American citizen when he divorced Paula . Such was also the situation when he married Alicia and executed his will. As stated in Article 15 of the civil code, alien s may obtain divorces abroad, provided that they are valid in their National Law . Thus the divorce obtained by Llorente is valid because the law that governs him is not Philippine Law but his National Law since the divorce was contracted after he became an American citizen. Furthermore, his National Law allowed divorce.

The case was remanded to the court of srcin for determination of the intrinsic validity of Lorenzo Llorente’s will and determination of the parties’ successional rights allowing proof of foreign law. GARCIA V. RECIO October 2, 2001

Rederick Recio, a Filipino, married Editha Samson, an Australian in Malabon Rizal. However, on 1989, they got divorced in an Australian family court. On 1992, Rederick became an Australian Citizen. He later married Petitioner in 1994 in Cabanatuan City. Thereafter, the two separated and petitioner filed a complaint for Declaration of Nullity of Marriage on the ground of bigamy. While the suit was pending, Rederick was able to obtain a divorce decree in Australia. Trial Court declared the marriage dissolved based on the subsequent divorce decree obtained by the respondent. •







Whether the divorce between respondent and Editha Samson was proven; Whether respondent was proven to be legally capacitated to marry petitioner; Held: A divorce obtained abroad by an alien may be recognized in our jurisdiction, provided such decree is valid according to the national law of the foreigner. However, the divorce decree and the governing personal law of the alien spouse who obtained the divorce must be proven . Our courts do not take judicial notice of foreign laws and judgment; hence, like any other facts, both the divorce decree and the national law of the alien must be alleged and proven according to our law on evidence. Was the first divorce validly obtained and binding?

At the outset, the Court lays the following basic legal principles; Philippine law does not provide for absolute divorce; hence, Philippine courts cannot grant it. A marriage between two Filipinos cannot be dissolved even by a divorce obtained abroad, because of Articles 1522 and 1723 of the Civil Code . In mixed marriages involving a Filipino and a foreigner, Article 26 of the Family Code allows the former to contract a subsequent marriage in case the divorce is "validly obtained abroad by the alien spouse capacitating him or her to remarry." A divorce obtained abroad by a couple, who are both aliens, may be recognized in the Philippines, provided it is consistent with their respective national laws. Before a foreign divorce decree can be recognized by our courts, the party pleading it must prove the divorce as a fact and demonstrate its conformity to the foreign law allow ing it . Presentation solely of the divorce decree is insufficient. In the case at bar, Respondent only presented the divorce decree; Likewise, before a foreign judgment is given presumptive evidentiary value, the document must first be presented and admitted in evidence. A divorce obtained abroad is proven by the divorce decree itself.

Indeed the best evidence of a judgment is the judgment itself. The decree purports to be a written act or record of an act of an officially body or tribunal of a foreign country. However, under Sections 24 and 25 of Rule 132, a writing or document may be proven as a public or official record of a foreign country by either (1) an official publi cation or (2) a copy thereof attested by the officer having legal custody of the document. If the record is not kept in the Philippines, such copy must be (a) accompanied by a certificate issued by the proper diplomatic or consular officer in the Philippine Foreign Service stationed in the foreign country in which the record is

Issues:

Private International Law

Page 52

kept and (b) authenticated by the seal of his office.

The divorce decree between respondent and Editha Samson appears to be an authentic one issued by an Australian family court. However, appearance is not sufficient; compliance with the aforementioned rules on evidence must be demonstrated. Fortunately for respondent, this matter was not objected to by the petitioner, thus by virtue of such waiver, is deemed admitted as evidence. Who has the burden of proving a foreign law? Respondent has the burden of proof; The burden of proof lies with "the party who alleges the exist ence of a fact or thing necessary in the prosecution or defense of an action." It is a well-settled that courts cannot take judicial notice of foreign laws. Like any other facts, they must be alleged and proved. Australian marital laws are not among those matters that judges are supposed to know by reason of their judicial function. The power of judicial notice must be exercised with caution, and every reasonable doubt upon the subject should be resolved in the negative. 2nd Issue: Is Respondent capacitated to remarry?

legally

Divorce means the legal dissolution of a lawful union for a cause arising after marriage. But divorces are of different types. The two basic ones are (1) absolute divorce or a vinculo matrimonii and (2) limited divorce or a mensa et thoro. The first kind terminates the marriage, while the second suspends it and leaves the bond in full force. There is no showing in the case at bar which type of divorce was procured by respondent. Respondent presented a decree nisi or an interlocutory decree – a conditional or provisional judgment of divorce. It is in effect the same as a separation from bed and board, although an absolute divorce may follow after the lapse of the prescribed period during which no reconciliation is effected. On its face, the herein Australian divorce

concerned. The certificate mentioned in Article 21 of the Family Code would have been sufficient to establish the legal capacity of respondent, had he duly presented it in court. A duly authenticated and admitted certificate is prima facie evidence of legal capacity to marry on the part of the alien applicant for a marriage license. In the case at bar, there is absolutely no evidence that proves respondent's legal capacity to marry petitioner.

Based on the above records, the Court cannot conclude that respondent, who was then a naturalized Australian citizen, was legally capacitated to marry petitioner on January 12, 1994. The court a quo erred in finding that the divorce decree ipso facto clothed respondent with the legal capacity to remarry without requiring him to adduce sufficient evidence to show the Australian personal law governing his status; or at the very least, to prove his legal capacity to contract the second marriage. Neither can the Court grant petition er's prayer to declare her marriage to respondent null and void on the ground of bigamy. After all, it may turn out that under Australian law, he was really capacitated to marry petitioner as a direct result of the divorce decree.

Hence, case was remanded to the court a quo for further determination of legal capacity and to receive evidence to determine if bigamy has been committed; REPUBLIC V. ORBECIDO GR NO. 154380, October 5, 2005

Facts: On May 24, 1981, Cipriano Orbecido III married Lady Myros M. Villanueva in the Philippines in Lam-an, Ozamis City. Their marriage was blessed with a son and a daughter. In 1986, Cipriano’s wife left for the United States bringing along their son Kristoffer. A few years later, Cipriano discovered that his wife had been naturalized as an American citizen.

decree contains a restriction that reads: "1. A party to a marriage who marries again before this decree becomes absolute (unless the other party has died) commits the offence of bigamy."

This quotation bolsters the Court’s contention that the divorce obtained by respondent may have been restricted. It did not absolutely establish his legal capacity to remarry according to his national law. Significance of the Certificate of Legal Capacity

Legal capacity to contract marriage is determined by the national law of the party

Private International Law

Sometime in 2000, Cipriano learned from his son that his wife had obtained a divorce decree and then married a certain Innocent Stanley. She, Stanley and her child by him currently live in San Gabriel, California. Cipriano thereafter filed with the trial court a petition for authority to remarry invoking Paragraph 2 of Article 26 of the Family Code. No opposition was filed. Finding merit in the petition, the court granted the same. The Republic, herein petitioner, through the Office of the Solicitor General (OSG), sought reconsideration but it was denied.

Page 53

The OSG contends that Paragraph 2 of Article 26 of the Family Code is not applicable to the instant case because it only applies to a valid mixed marriage; that is, a marriage celebrated between a Filipino citizen and an alien. Furthermore, the OSG argues there is no law that governs respondent’s situation. The OSG posits that this is a matter of legislation and not of judicial determination.

Art. 99.

No person shall be entitled to a legal separation who has not resided in the Philippines for one year prior to the filing of the petition, unless the cause for the legal separation has taken place within the territory of this Republic. (Sec. 2a, Act No. 2710) TITLE II

Held: Taking into consideration the legislative intent and applying the rule of reason, we hold that Paragraph 2 of Article 26 should be interpreted to include cases involving parties who, at the time of the celebration of the marriage were Filipino citizens, but later on, one of them becomes naturalized as a foreign citizen and obtains a divorce decree. The Filipino spouse should likewise be allowed to remarry as if the other party were a foreigner at the time of the solemnization of the marriage . To rule otherwise would be to sanction absurdity and injustice. Where the interpretation of a statute according to its exact and literal import would lead to mischievous results or contravene the clear purpose of the legislature, it should be construed according to its spirit and reason, disregarding as far as necessary the letter of the law. A statute may therefore be extended to cases not within the literal meaning of its terms, so long as they come

LEGAL SEPARATION Art. 55. A petition for legal separation may be filed on any of the following grounds: (1) Repeated physical violence or grossly abusive conduct directed against the petitioner, a common child, or a child of the petitioner; (2) Physical violence or moral pressure to compel the petitioner to change religious or political affiliation; (3) Attempt of respondent to corrupt or induce the petitioner, a common child, or a child of the petitioner, to engage in prostitution, or connivance in such corruption or inducement; (4) Final judgment sentencing the respondent to imprisonment of more than six years, even if pardoned;

within its spirit or intent. In view of the fore going, we state the twin elements for the application of Paragraph 2 of Article 26 as follows: 3. There is a vali d mar riage tha t has been celebrated between a Filipino citizen and a foreigner; and 4. A vali d div orce is obt ained ab road by the alien spouse capacitating him or her to remarry.

(5) Drug addiction or habitual alcoholism of the respondent; (6) Lesbianism or homosexuality of the respondent; (7) Contracting by the respondent of a subsequent bigamous marriage, whether in the Philippines or abroad; (8) Sexual infidelity or perversion;

The reckoning point is not the citizenship of the parties at the time of the celebration of the marriage, but their citizenship at the time a valid divorce is obtained abroad by the alien spouse capacitating the latter to remarry. 5.

(9) Attempt by the respondent against the life of the petitioner; or (10) Abandonment of petitioner by respondent without justifiable cause for more than one year.

Legal Separation

Civil Code: Art. 97. A separation may be filed: 1.

2.

petition

for

legal

For adultery on t he part of the wife and for concubinage on the part of the husband as defined in the Penal Code; or An attempt by one spouse against the life of the other. (n)

Private International Law

For purposes of this Article, the term "child" shall include a child by nature or by adoption. (9a) Art. 56. The petition for legal separation shall be denied on any of the following grounds: (1) Where the aggrieved party has condoned the offense or act complained of; (2) Where the aggrieved party has consented to the commission of the offense or act complained of;

Page 54

(3) Where there is connivance between the parties in the commi ssion of the offense or act constituting the ground for legal separation;

absolute community or the conjugal partnership, which shall be forfeited in accordance with the provisions of Article 43(2);

(4) Where both parties have given ground for legal separation;

(3) The custody of the minor children shall be awarded to the innocent spouse, subject to the provisions of Article 213 of this Code; and

(5) Where there is collusion between the parties to obtain decree of legal separation; or (6) Where the action is barred by prescription. (100a) Art. 57. An action for legal separation shall be filed within five years from the time of the occurrence of the cause. (102) Art. 58. An action for legal separation shall in no case be tried before six months shall have elapsed since the filing of the petition. (103) Art. 59. No legal separation may be decreed unless the Court has taken steps toward the reconciliation of the spouses and is fully satisfied, despite such efforts, that reconciliation is highly improbable. (n) Art. 60. No decree of legal separation shall be based upon a stipulation of facts or a confession of judgment. In any case, the Court shall order the prosecuting attorney or fiscal assigned to it to take steps to prevent collusion between the parties and to take care that the evidence is not fabricated or suppressed. (101a) Art. 61. After the filing of the petition for legal separation, the spouses shall be entitled to live separately from each other. The court, in the absence of a written agreement between the spouses, shall designate either of them or a third person to administer the absolute community or conjugal partnership property. The administrator appointed by the court shall have the same powers and duties as those of a guardian under the Rules of Court. (104a) Art. 62. During thethe pendency of the action for legal separation, provisions of Article 49 shall likewise apply to the support of the spouses and the custody and support of the common children. (105a) Art. 63. The decree of legal separation shall have the following effects:

(4) The offending spouse shall be disqualified from inheriting from the innocent spouse by intestate succession. Moreover, provisions in favor of the offending spouse made in the will of the innocent spouse shall be revoked by operation of law. (106a) Art. 64. After the finality of the decree of legal separation, the innocent spouse may revoke the donations made by him or by her in favor of the offending spouse, as well as the designation of the latter as beneficiary in any insurance policy, even if such designation be stipulated as irrevocable. The revocation of the donations shall be recorded in the registries of property in the places where the properties are located. Alienations, liens and encumbrances registered in good faith before the recording of the complaint for revocation in the registries of property shall be respected. The revocation of or change in the designation of the insurance beneficiary shall take effect upon written notification thereof to the insured. The action to revoke the donation under this Article must be brought within five years from the time the decree of legal separation become final. (107a) Art. 65. If the spous es should reconcile, a corresponding joint manifestation under oath duly signed by them shall be filed with the court in the same proceeding for legal separation. (n) Art. 66. The reconciliation referred to in the preceding Articles shall have the following consequences: (1) The legal separation proceedings, if still pending, shall thereby be terminated at whatever stage; and (2) The final decree of legal separation shall be set aside, but the separation of property and any forfeiture of the share of the guilty spouse already effected shall subsist, unless the spouses agree to revive their former property regime.

(1) The spouses shall be entitled to live separately from each other, but the marriage bonds shall not be severed;

The court's order containing the foregoing shall be recorded in the proper civil registries. (108a)

(2) The absolute community or the conjugal partnership shall be dissolved and liquidated but the offending spouse shall have no right to any share of the net profits earned by the

Art. 67. The agreement to revive the former property regime referre d to in the preceding Article shall be executed under oath and shall specify:

Private International Law

Page 55

(1) The properties to be contributed anew to the restored regime; (2) Those to be retained as separated properties of each spouse; and

Civil Code: Art. 15, supra Art. 335.

The following cannot adopt:

(3) The names of all their known creditors, their addresses and the amounts owing to each.

1. Those who have legitimate, legitimated, acknowledged natural children, or natural children by legal fiction; 2. The guardian, with respect to the ward, before the final approval of his accounts; 3. A married person, without the consent of the other spouse; 4. Non-resident aliens; 5. Resident aliens with whose government the Republic of the Philippines has broken diplomatic relations; 6. Any person who has been convicted of a crime involving moral turpitude, when the penalty imposed was six months' imprisonment or more. (174a)

The agreement of revival and the motion for its approval shall be filed with the cour t in the same proceeding for legal separation, with copies of both furnished to the creditors named therein. After due hearing, the court shall, in its order, take measure to protect the interest of creditors and such order shall be recorded in the proper registries of properties. The recording of the ordering in the registries of property shall not prejudice any creditor not listed or not notified, unless the debtor-spouse has sufficient separate properties to satisfy the creditor's claim. (195a, 108a) Capacity to Remarry All marriages solemnized outside the Philippines, in accordance with the laws in force in the country where they were solemnized, and valid there as such, shall also be

Art. 26.

Art. 339. adopted:

valid this country, except those in prohibited under Articles 35 (1), (4), (5) and (6), 3637 and 38. (17a) Where a marriage between a Filipino citizen and a foreigner is validly celebrated and a divorce is thereafter validly obtained abroad by the alien spouse capacitating him or her to remarry, the Filipino spouse shall have capacity to remarry under Philippine law. (As amended by Executive Order 227)

validity



law of the place of

celebration; lex loci contractus rule Substantial validity – governed by:

1. Personal law: status/legal capacity – National law of the parties

2. lex loci celebrationis: 

exceptions: Family Code Art. 26, paragraph 1 Consular Marriages – consul granted by the accepting country where such consular office was located to solemnize marriage •



VIII. Parents Relationship)

and

Children

Private International Law

(Parental

following

cannot

be

1. A married person, without the written consent of the other spouse; 2. An alien with whose government the Republic of the Philippines has broken diplomatic relations; 3. A person who has already been adopted. (n) Muslim Code: TITLE V Parental Authority CHAPTER I Nature and Effects

Nota bene: Formal

The

Art. 71.

Who exercises. — (1) The father and the mother shall jointly exercise just and reasonable parental authority and fulfill their responsibility over their legitimate and acknowledged children. In case of disagreement, the father's decision shall prevail unless there is a judicial order to the contrary. (2) The mother shall exercise parental authority over her children born out of wedlock, but the court

Page 56

may, when the best interests of the children so require, appoint a general guardian. .chan robles virtual law library Art. 72.

renounced nor transferred except as otherwise provided in this Code and the general principles of Islamic law. Art. 77. authority. —

Duty to parents. — (1) Children shall respect, revere, and obey their parents always unless the latter cast them into disbelief.

Duty to children. — Every parent and every person exercising parental authority shall see to it that the rights of the children are respected, and their duties complied with, and shall particularly by precept and example, imbue them with religious and civic attachment to the ideal of permanent world peace.

Art. 74.

Effects upon person of children. — The parents have, with respect to their unemancipated children:

Art. 78.

property

Care and custody. — (1) The care and cus tody of children below seven years of age whose parents are divorced shall belong to the mother or, in her absence, to the maternal grandmother, the paternal grandmother, the sister and aunts. In their default, it shall devolve upon the father and the nearest paternal relatives. The minor above seven years of age but below the age of puberty may choose the parent with whom he wants to stay.

(b) The power to cor rect, discipline, and punish them moderately. Effects upon children. —

parental

CHAPTER II Custody and Guardianship

(a) The duty to support them, have them in their company, educate and instruct them in keeping with their means and represent them in all actions which shall redound to their benefits; and

Art. 75.

of

(1) Parental authority terminates upon the death of the parents or the child, or upon emancipation. (2) Subject to Article 78, the widowed mother who contracts a subsequent marriage shall lose parental authority and custody over all children by the deceased husband, unless the second husband is related to them within the prohibited degrees of consanguinity. (3) The court may deprive a person of parental authority or suspend the exercise thereof if he treats his children with excessive harshness, gives then corrupting or immoral orders and counsel, or abandons them.

(2) Grandparents are likewise entitled to respect and reverence, and shall be consulted whenever practicable by all members of the family on all important questions. Art. 73.

Extinguishment

of

(1) The father, or in his absence the mother, shall be the legal administrator of the property of the child under parental authority. If the property is worth more than five thousand pesos, the father or the mother shall give a bond to be approved by the court. (2) The court may appoint a guardian (wasi) in the absence of one who is natural or testamentary. Art. 76.

Parental authority nontransferable. — Parental authority can neither be

Private International Law

(2) The unmarried daughter who has reached the age of puberty shall stay with the father; the son, under the same circumstances, shall stay with the mother. Art. 79.

Guardian for marriage (wali). — The following persons shall have authority to act as guardian for marriage (wali) in the order of precedence: (a) Father (b) Paternal grandfather; (c) Brother and other paternal relatives;

Page 57

(d) Paternal grandfather's executor or nominee; or .chan robles virtual law library (e) The court. Art. 80.

Guardian of minor's property. — The following persons shall exercise guardianship over the property of minors in the order of precedence: (a) Father; (b) Father's executor or nominee; (c) Paternal grandfather; (d) Paternal grandfather's nominee; or (e) The court.

Art. 163. The filiation of children may be by nature or by adoption. Natural filiation may be legitimate or illegitimate. (n) Art. 164. Children conceived or born during the marriage of the parents are legitimate. Children conceived as a result of artificial insemination of the wife with the sperm of the husband or that of a donor or both are likewise legitimate children of the husband and his wife, provided, that both of them authorized or ratified such insemination in a written instrument executed by them before the birth of the and child.signed The instrument shall be recorded in the civil registry together with the birth certificate of the child. (55a, 258a) Art. 165. Children conceived and born outside a valid marriage are illegitimate, unless otherwise provided in this Code. (n) Art. 166. Legitimacy of a child may be impugned only on the following grounds: (1) That it was physically impossible for the husband to have sexual intercourse with his wife within the first 120 days of the 300 days which immediately preceded the birth of the child because of:

provided in the second paragra ph of Article 164; or (3) That in case of children conceived through artificial insemination, the written authorization or ratification of either parent was obtained through mistake, fraud, violence, intimidation, or undue influence. (255a) Art. 167. The child shall be considered legitimate although the mother may have declared against its legitimacy or may have been sentenced as an adulteress. (256a) Art. 168. If the marriage is terminated and the mother contracted another marriage within three hundred days after such termination of the former marriage, these rules shall govern in the absence of proof to the contrary: (1) A child born before one hundred eighty days after the solemnization of the subsequent marriage is considered to have been conceived during the former marriage, provided it be born within three hundred days after the termination of the former marriage; (2) A child born after one hundred eighty days following the celebration of the subsequent marriage is considered to have been conceived during such marriage, even hundred though days it be born within the three after the termination of the former marriage. (259a) Art. 169. The legitimacy or illegitimacy of a child born after three hundred days following the termination of the marriage shall be proved by whoever alleges such legitimacy or illegitimacy. (261a) Art. 170. The action to impugn the legitimacy of the child shall be brought within one year from the knowledge of the birth or its recording in the civil regi ster, if the husband or, in a proper case, any of his heirs, should reside in the city or municipality where the birth took place or was recorded. If the husband or, in his default, all of his heirs

(a) the physical of the husband to incapacity have sexual intercourse with his wife; (b) the fact that the husband and wife were living separately in such a way that sexual intercourse was not possible; or (c) serious illness of the husband, which absolutely prevented sexual intercourse; (2) That it is proved that for biological or other scientific reasons, the child could not have been that of the husband, except in the instance

Private International Law

do not at the or place of birth asrecorded, defined in the firstreside paragraph where it was the period shall be two years if they should reside in the Philippines; and three years if abroad. If the birth of the child has been concealed from or was unknown to the husband or his heirs, the period shall be counted from the discovery or knowledge of the birth of the child or of the fact of registration of said birth, whichever is earlier. (263a) Art. 171. The heirs of the husband may impugn the filiation of the child within the period prescribed in the preceding article only in the following cases:

Page 58

(1) If the husband should died before the expiration of the period fixed for bringing his action; (2) If he should die after the filing of the complaint without having desisted therefrom; or (3) If the child was born after the death of the husband. (262a) Chapter 2. Proof of Filiation Art. 172. The filiation of legitimate children is established by any of the following: (1) The record of birth appearing in the civil register or a final judgment; or (2) An admission of legitimate filiation in a public document or a private handwritten instrument and signed by the parent concerned. In the absence of the foregoing evidence, the legitimate filiation shall be proved by: (1) The open and continuous possession of the status of a legitimate child; or

The action must be brought within the same period specified in Article 173, except when the action is based on the secon d paragraph of Article 172, in which case the action may be brought during the lifetime of the alleged parent. (289a) Art. 176. Illegitimate children shall use the surname and shall be under the parental authority of their mother, and shall be entitled to support in conformity with this Code. The legitime of each illegitimate child shall consist of one-half of the legitime of a legitimate child. Except for this modification, all other provisions in the Civil Code governing successional rights shall remain in force. (287a)

Chapter 4. Legitimated Children Art. 177. Only children conceived and born outside of wedlock of parents who, at the time of the conception of the former, were not disqualified by any impediment to marry each other may be legitimated. (269a) Art. 178. Legitimation shall take place by a subsequent valid marriage between parents. The annulment of a voidable marriage shall not affect the legitimation. (270a)chan robles virtual law library Art. 179. Legitimated children shall enjoy the same rights as legitimate children. (272a)

(2) Any other and means allowed the Rules of Court special laws.by (265a, 266a, 267a) Art. 173. The action to claim legitimacy may be brought by the child during his or her lifetime and shall be transmitted to the heirs should the child die during minority or in a state of insanity. In these cases, the heirs shall have a period of five years within which to institute the action. Art. 174. Legitimate children shall have the right: (1) To bear the surnames of the father and the mother, in conformity with the provisions of the Civil Code on Surnames; (2) To receive support from their parents, their ascendants, and in proper cases, their brothers and sisters, in conformity with the provisions of this Code on Support; and (3) To be entitled to the legitimate and other successional rights granted to them by the Civil Code. (264a) Chapter 3. Illegitimate Children Art. 175. Illegitimate children may establish their illegitimate filiation in the same way and on the same evidence as legitimate children.

Private International Law

Art. 180. The effects of legitimation shall retroact to the time of the child's birth. (273a) Art. 181. The legitimation of children who died before the celebration of the marriage shall benefit their descendants. (274) Art. 182. Legitimation may be impugned only by those who are prejudiced in their rights, within five years from the time their cause of action accrues. (275a) Art. 209. Pursuant to the natural right and duty of parents over the person and property of their unemancipated children, parental authority and responsibility shall include the caring for and rearing them for civic consciousness and efficiency and the development of their moral, mental physical character and well-being. (n)

and

Art. 210. Parental authority and responsibility may not be renounced or transferred except in the cases authorized by law. (313a) Art. 211. The father and the mother shall jointly exercise parental authority over the persons of their common children. In case of disagreement, the father's decision shall prevail, unless there is a judicial order to the contrary. Children shall always observe respect and reverence towards their parents and are obliged to obey them as long as the children

Page 59

are under parental authority. (311a) robles virtual law library

chan

Art. 212. In case of absence or death of either parent, the parent present shall continue exercising parental authority. The remarriage of the surviving parent shall not affect the parental authority over the children, unless the court appoints another person to be the guardian of the person or property of the children. (n)

Art. 218. The school, its administrators and teachers, or the individual, entity or institution engaged in child are shall have special parental authority and responsibility over the minor child while under their supervision, instruction or custody. Authority and responsibility shall apply to all authorized activities whether inside or outside the premises of the school, entity or institution. (349a)

Art. 213. In case of separation of the parents,

Art. 129. Those given the authority and

parental authority shall be exercised by the parent designated by the Court. The Court shall take into account all relevant considerations, especially the choice of the child over seven years of age, unless the parent chosen is unfit. (n)

responsibility under the preceding Article shall be principally and solidarily liable for damages caused by the acts or omissions of the unemancipated minor. The parents, judicial guardians or the persons exercising substitute parental authority over said minor shall be subsidiarily liable.

Art. 214. In case of death, absence or unsuitability of the parents, substitute parental authority shall be exercised by the surviving grandparent. In case several survive, the one designated by the court, taking into account the same consideration mentioned in the preceding article, shall exercise the authority. (355a) Art. 215. No descendant shall be compelled, in a criminal case, to testify against his parents and grandparents, except when such testimony is indispensable in a crime against the descendant or by one parent against the other. (315a)

Chapter 2. Substitute and Special Parental Authority Art. 216. In default of parents or a judicially appointed guardian, the following person shall exercise substitute parental authority over the child in the order indicated: (1) The surviving grandparent, as provided in Art. 214; (2) The oldest brother or sister, over twenty-one years of age, unless unfit or disqualified; and (3) The child's actual custod ian, over twenty-one years of age, unless unfit or disqualified. Whenever the appointment or a judicial guardian over the property of the child becomes necessary, the same order of preference shall be observed. (349a, 351a, 354a) Art. 217. In case of foundlings, abandoned neglected or abused children and other children similarly situated, parental authority shall be entrusted in summary judicial proceedings to heads of children's homes, orphanages and similar institutions duly accredited by the proper government agency. (314a)

Private International Law

The respective liabilities of those referred to in the preceding paragraph shall not apply if it is proved that they exercised the proper diligence required under the particular circumstances. All othe r cases not covered by this and the preceding articles shall be governed by the provisions of the Civil Code on quasi-delicts. (n)

Chapter 3. Effect of Parental Authority Upon the Persons of the Children Art. 220. The parents and those exercising parental authority shall have with the respect to their unemancipated children on wards the following rights and duties: (1) To keep them in their company, to support, educate and instruct them by right precept and good example, and to provide for their upbringing in keeping with their means; (2) To give them love and affe ction, advice and counsel, companionship and understanding; (3) To provide them with moral and spiritual guidance, inculcate in them honesty, integrity, self-discipline, selfreliance, industry and thrift, stimulate their interest in civic affairs, and inspire in them compliance with the duties of citizenship; (4) To furnish them with good and wholesome educational materials, supervise their activities, recreation and association with others, protect them from bad company, and prevent them from acquiring habits detrimental to their health, studies and morals; (5) To represent them in all matte rs affecting their interests; (6) To demand from them respect and obedience;

Page 60

(7) To impose disc ipline on them as may be required under the circumstances; and (8) To perform such other duties as are imposed by law upon parents and guardians. (316a) Art. 221. Parents and other persons exercising parental authority shall be civilly liable for the injuries and damages caused by the acts or omissions of their unemancipated children living in their company and under their parental authority subject to the appropriate defenses provided by law. (2180(2)a and (4)a ) Art. 222. The courts may appoint a guardian of the child's property or a guardian ad litem when the best interests of the child so requires. (317) Art. 223. The parents or, in their absence or incapacity, the individual, entity or institution exercising parental authority, may petition the proper court of the place where the child resides, for an order providing for disciplinary measures over the child. The child shall be entitled to the assistance of counsel, either of his choice or appointed by the court, and a summary hearing shall be conducted wherein the petitioner and the child shall be heard.

may determine, but not less than ten per centum (10%) of the value of the property or annual income, to guarantee the performance of the obligations prescribed for general guardians. A verified petition for approval of the bond shall be filed in the proper court of the place where the child resides, or, if the child resides in a foreign country, in the proper court of the place where the property or any part thereof is situated. The petition shall be docketed as a summary special proceeding in which all incidents and issues regarding the performance of the obligations referred to in the second paragraph of this Article shall be heard and resolved. The ordinary rules on guardianship shall be merely suppletory except when the child is under substitute parental authority, or the guardian is a stranger, or a parent has remarried, in which case the ordinary rules on guardianship shall apply. (320a)

However, if in the same proceeding the court

Art. 226. The property of the unemancipated child earned or acquired with his work or industry or by onerous or gratuitous title shall belong to the child in ownership and shall be devoted exclusively to the latter's support and education, unless the title or transfer provides otherwise.

finds petitioner at fault, irrespective the meritstheof the petition, or when ofthe circumstances so warrant, the court may also order the deprivation or suspension of parental authority or adopt such other measures as it may deem just and proper. (318a)

The right of the parents over the frui ts and income of the child's property shall be limited primarily to the child's support and secondarily to the collective daily needs of the family. (321a, 323a)

Art. 224. The measures referred to in the preceding article may include the commitment of the child for not more than thirty days in entities or institutions engaged in child care or in children's homes duly accredited by the proper government agency.

Art. 227. If the parents entrust the management or administration of any of their properties to an unemancipated child, the net proceeds of such property shall belong to the owner. The child shall be given a reasonable monthly allowance in an amount not less than that which the owner would have paid if the administrator were a stranger, unless the owner, grants the entire proceeds to the child. In any case, the proceeds thus give in whole or in part shall not be charged to the child's legitime. (322a)

The parent exercising parental authority shall not interfere with the care of the child whenever committed but shall provide for his support. Upon proper petition or at its own instance, the court may terminate the commitment of the child whenever just and proper. (391a)

Chapter 5. Suspension or Termination of Parental Authority Chapter 4. Effect of Parental Authority Upon the Property of the Children Art. 225. The father and the mother shall jointly exercise legal guardianship over the property of the unemancipated common child without the necessity of a court appointment. In case of disagreement, the father's decision shall prevail, unless there is a judicial order to the contrary. Where the market value of the property or the annual income of the child exceeds P50,000, the parent concerned shall be required to furnish a bond in such amount as the court

Private International Law

Art. 228. Parental authority terminates permanently: (1) Upon the death of the parents; (2) Upon the death of the child; or (3) Upon emancipation of the child. (327a) Art. 229. Unless subsequently revived by a final judgment, parental authority also terminates: (1) Upon adoption of the child;

Page 61

(2) Upon appointment of a general guardian;

exercising special parental authority inflict corporal punishment upon the child. (n)

(3) Upon judicial declaration of abandonment of the child in a case filed for the purpose;

Ching Leng v. Galang 57 OG 2312

(4) Upon final judgment of a competent court divesting the party concerned of parental authority; or (5) Upon judicial declaration of absence or incapacity of the person exercising parental authority. (327a) Art. 230. Parental authority is suspended upon conviction of the parent or the person exercising the same of a crime which carries with it the penalty of civil interdiction. The authority is automatically reinstated upon service of the penalty or upon pardon or amnesty of the offender. (330a) Art. 231. The court in an action filed for the purpose in a related case may also suspend parental authority if the parent or the person exercising the same: (1) Treats the child with excessive harshness or cruelty; (2) Gives the child corrupting orders, counsel or example; (3) Compels the child to beg; or (4) Subjects the child or allows him to be subjected to acts of lasciviousness. The grounds enumerated above are deemed to include cases which have resulted from culpable negligence of the parent or the person exercising parental authority. If the degree of seriousness so warrants, or the welfare of the child so demands, the court shall deprive the guilty party of parental authority or adopt such other measures as may be proper under the circumstances. The suspension or deprivation may be revoked and the parental authority revived in a case filed for the purpose or in the same proceeding if the court finds that the cause therefor has ceased and will not be repeated. (33a) Art. 232. If the person exercising parental authority has subjected the child or allowed him to be subjected to sexual abuse, such person shall be permanently deprived by the court of such authority. (n) Art. 233. The person exercising substitute parental authority shall have the same authority over the person of the child as the parents. In no case shall the school administrator, teacher of individual engaged in child care

Private International Law

“In Ching Leng the provision in the 1935 Constitution stating "those whose fathers are citizens of the Philippines" refers only to legitimate children. When the 1973 and 1987 Constitutions were drafted, the framers did not attempt to change the intent of this provision, even as they were presumably aware of the Ching Leng doctrine. Nevertheless, I believe that it is now time to abandon the Ching Leng doctrine. (Tecson V. Comelec)” The prevailing doctrine today is that an illegitimate child of a Filipino father and an alien mother follows the citizenship of the alien mother as the only legally known parent. The illegitimate child, even if acknowledged and legally adopted by the Filipino father, cannot acquire the citizenship of the father. The Court made this definitive doctrinal ruling in Ching Leng v. Galang which involved the illegitimate minor children of a naturalized Filipino of Chinese descent with a Chinese woman, Sy An. The illegitimate children were later on jointly adopted by the naturalized Filipino and his legal wife, So Buan Ty. The facts in Ching Leng as quoted by the Court from the trial court’s decision are as follows: After the petitioner Ching Leng Alias Ching Ban Lee obtained judgment in this Court dated May 2, 1950 granting his petition for naturalization, he together with his wife So Buan Ty filed another petition also in this Court in Special Proc. No. 1216 for the adoption of Ching Tiong Seng, Ching Liong Ding, Victoria Ching Liong Yam, Sydney Ching and Ching Tiong An, all minors and admittedly the illegitimate children of petitioner Ching Leng with one Sy An , a Chinese citizen. Finding the petition for adoption proper, this Court granted the same in a decision dated September 12, 1950, declaring the said minors free from all legal obligations of obedience and maintenance with

respect to their mother Sy An and to all legal intents and purposes the children of the adopter Ching Leng alias Ching Ban Lee and So Buan Ty with all the legal rights and obligations provided by law. On September 29, 1955, Ching Leng took his oath of allegiance and became therefore a full pledge (sic) Filipino citizen . Believing now that his adopted illegitimate children became Filipino citizens by virtue of his naturalization, petitioner Ching Leng addressed a communication to the respondent Commissioner of Immigration requesting that the alien certificate of registration of the said minors be cancelled. (Bold underscoring supplied)

Page 62

In Ching Leng, the Cour t made a definitive ruling on the meaning of "minor child or children" in Section 15 of the Naturalization Law, as well as the meaning of children "whose parents are citizens of the Philippines" under the Constitution. The Court categorically ruled that these children refer to legitimate children only, and not to illegitimate children. Thus, the Court held:







It is claimed that the phrases "minor children" and "minor child" , used in these provisions, include adopted children. The argument is p red cadtedchilup rytent ths at an an ad opite d on is, th foer th aleo l in d purposes, a legitimate child. Whenever, the word "children" or "child" is used in statutes, it is generally understood, however, to refer to legitimate childre n, unless the context of the law and its spirit indicate clearly the contrary. Thus, for instance, when the Constitution provides that "those whose parents are citizens of the Philippines, "and "those whose mothers are citizens of the Philippines," who shall elect Philippine citizenship "upon reaching the age of majority", are citizens of the Philippines (Article IV, Section 1, subdivisions 3 and 4), our fundamental law clearly refers to legitimate children (Chiong Bian vs. De Leon, 46 Off. Gaz., 3652-3654; Serra v. Republic, L-4223, May 12, 1952).













Similarly, the children alluded to in said section 15 are those begotten in lawful wedlock, when the adopter, at least is the father. In fact, illegitimate children are under the parental authority of the mother and follow her nationality, not that of the illegitimate father (U.S. vs. Ong Tianse, 29 Phil. 332, 335336; Santos Co vs. Gov’t of the Philippines, 52 Phil. 543, 544; Serra v. Republic, supra; Gallofin v. Ordoñez, 70 Phil. 287; Quimsuan vs. Republic, L-4693, Feb. 16, 1953). Although, adoption gives "to the adopted person the same rights and duties as if he were a legitimate child of the adopter", pursuant to said Article 341 of our Civil Code, we have already seen that the rights therein alluded to are merely those enumerated in Article 264, and do not include the acquisition of the nationality of the adopter. Moreover, as used in said section 15 of the Naturalization Law, the term "children" could not possibly refer to those whose relation to the naturaliz ed person is one created by legal fiction, as, for instance, by adoption, for, otherwise, the place and time of birth of the child would be immaterial. The fact that the adopted persons involved in the case at bar are illegitimate children of appellant Ching Leng does not affect substantially the legal situation before us, for, by legal fiction, they are now being sought to be given the status of legitimate children of said appellant, despite the circumstance that the Civil Code of the Philippine does not permit their legitimation. Ng Hian v. Collector 34 Phil 248

Private International Law







On 30th of October, 1915 the steamship Tian arrived at the port of Manila. A woman, Marcosa S. Dy Jiongco, together with two children, Ng Tio a female of the age of 9 years, and Ng Hian a boy of 16 years of age (the petitioner herein) were on board. Marcosa S. Dy Jiongco, a Filipina born of a Filipina mother and a Chinese father was married to a Chinaman by the name of (Filipino name) Juan Uy Tue, (Chinese name) Ng Chion Tue. That Juan Uy with Tue Marcosa (Ng Chion before his marriage S. Tue), Dy Jiongco, had been married to a Chinese woman with whom he had some children, the petitioner herein and also one called Ng Guan. It appears that Ng Guan was residing in the Philippine Islands at the time of the presentation of the present petition; The Chinese wife of Juan Uy Tue died while the petitioner herein, Ng Hian, was a very small child; Juan Uy Tue, after the death of his Chinese wife, married Marcosa S. Dy Jiongco. Ng Tio was the daughter of the brother of the said Juan Uy Tue, born of a Chinese father and mother; that the father of the little girl had given her to Marcosa S. Dy Jiongco; Marcosa S. Dy Jiongco, being the stepmother of the said Ng Hian, adopted him and was bringing him to the Philippine Islands to study. The Board of Special Inquiry refused the right of each child to enter the Philippine Islands; However, after a rehearing, Ng Tio was admitted but Ng Hian was not. Petitioner petitioned for habeas corpus in the CFI, CFI granted entry to petitioner, Collector of Customs appealed; Thus the present petition;

Issue: Whether or not the minor children of a deceased resident Chinese merchant have a right to enter the territory of the Philippine Islands; Held: It is true that the petitioner, Ng Hian, had never been in the Philippine Islands before. It is also true that the said Marcosa S. Dy Jingco was his stepmother. She swore positively that she had adopted him. That fact is not denied of record. Until the fact is denied we must accept it. There is nothing in the record which shows or tends to show that she had not adopted him in good faith. The question whether or not Marcosa S. Dy Jiongco could bring Ng Hian into the territory of the Philippine Islands as her adopted son has been discussed by the Federal Courts of the United States. In the case of Ex parte Fong Yim (134 Fed. Rep., 938), the court held that: A Chinese merchant domiciled in the United States has the right to bring into

Page 63

this country with his wife minor children legally adopted by him in China, where it is shown that the adoption was bona fide, and that the children have lived as members of his family and have been supported by him for several years.

The court further said: Of course, the question whether the adoption is a genuine one is a question of fact, open to investigation . . . . The evidence shows that the practice of adopting children in China is very common, that it takes place substantially without legal formalities, but that the rights and obligati ons of children adopted and recognized as such are similar to those of natural children. Under these circumstances I can see no difference between the legal status of adopted children and of natural children. The Supreme Court (of the United States) having decided that a Chinese merchant domiciled in this country has the right to bring into it his natural children, I think that the same decision is authority for the proposition that he has the right to introduce his adopted children; ROEHR VS. RODRIGUEZ

Facts:

Petitioner Wolfgang Roehr, a German citizen, married a Filipina, Respondent Carmen Rodriguez in Germany. The marriage was ratified in Tayasan, Negros Oriental. They had two children. Private respondent filed a petition for the declaration of nullity of marriage before the RTC of Makati. Petitioner filed a motion to dismiss but was denied by the trial court. However, The petitioner obtained a decree of divorce from the Court of First Instance of Hamburg - Blankenese with the custody of the children granted to the father.

such decree is valid according to the national law of the foreigner. Relevant to the present case is Pilapil v. Ibay-Somera, where the Court specifically recognized the validity of a divorce obtained by a German citizen in his country, the Federal Republic of Germany. The Court held in Pilapil that a foreign divorce and its legal effects may be recognized in the Philippines insofar as respondent is concerned in view of the nationality principle in our civil law on the status of persons.

In case, decree issued by thethis Ger man the coudivorce rt dated Decemb er 16, 1997 has not been challenged by either of the parties. In fact, save for the issue of parental custody, even the trial court recognized said decree to be valid and binding, thereby endowing private respondent the capacity to remarry. Thus, the present controversy mainly relates to the award of the custody of their two children, Carolynne and Alexandra Kristine, to petitioner. As a general rule, divorce decrees obtained by foreigners in other countries are recogniza ble in our jurisdic tion, but the legal effects thereof, e.g. on custody, care and support of the children, must still be determined by our courts. Before our courts can give the effect of res judicata to a foreign judgment, such as the award of custody to petitioner by the German court, it must be shown that the parties opposed to the judgm ent had been given ample opportunity to do so on grounds allowed under Rule 39, Section 50 of the Rules of Court (now Rule 39, Section 48, 1997 Rules of Civil Procedure), to wit: SEC. 5 0. Effect of foreign judgments. - The effect of a judgment of a tribunal of a foreign country, having jurisdiction to pronounce the judgment is as follows:

Issue:

Whether or not the legal effects of a divorce obtained from a foreign country such as support and custody of the children can be determined in our courts? Held:

Yes. In order to take effect, a foreign judgement must clearly show that the opposing party has been given ample opportunity to do so under the Rules of Civil Procedure. Accordingly, the respondent was not given the opportunity to challenge the judgment of the German Court, therefore, legal effects of divorce must be determined in our courts. The court held that the trial court has jurisdiction over the issue between the parties as to who has the parental custody.

In Garcia v. Recio ,19 Van Dorn v. Romillo, Jr.,20 and Llorente v. Court of Appeals , the Court consistently held that a divorce obtained abroad by an alien may be recognized in our jurisdiction, provided

Private International Law

(a) In case of a judgment upon a specific thing, the judgment is conclusive upon the title to the thing; (b) In case of a judgment against a person, the judgment is presumptive evidence of a right as be tw s uc ceee ssnorsthein pairt ntie esresan t d bytheiar subsequent title; but the judgment may be repelled by evidence of a want of jurisdiction, want of notice to the party, collusion, fraud, or clear mistake of law or fact.

It is essential that there should be an opportunity to challenge the foreign judgment, in order for the court in this jurisdiction to properly determine its efficacy. In this jurisdiction, our Rules of Court clearly provide that with respect to actions in personam, as distinguished from actions in rem , a foreign judgment merely constitutes prima facie evidence of the

Page 64

justness of the claim of a party and, as such, is subject to proof to the contrary.

In the pres ent case , it cannot be said that private respondent was given the opportunity to challenge the judgment of the German court so that there is basis for declaring that judgment as res judicata with regard to the rights of petitioner to have parental custody of their two children. The proceedings in the German court were summary. Exam is on Wednesday 630 to 830 Coverage is from start to parents and children

Private International Law

Page 65

View more...

Comments

Copyright ©2017 KUPDF Inc.
SUPPORT KUPDF